The question bank may take some time to load… Just enough time to stretch, blink a few times, and question your life choices — but not too long, we promise!
We recommend going Full Screen for the best experience. Have Fun !
Report a question
Endo – Pathology
Compiled Topical Questions of Endo – Pathology
Think about which hormone plays a crucial role in maintaining sodium and water balance, and what happens to blood volume when that hormone is deficient.
1 / 232
Tags:
2018
A patient was diagnosed with Addison’s disease. Which of the following would explain the symptom of orthostatic hypotension in this patient?
To understand the cause of orthostatic hypotension in Addison’s disease , let’s walk through the pathophysiology of the disease step-by-step.
🔬 What is Addison’s Disease?
Addison’s disease is primary adrenal insufficiency —a condition in which the adrenal cortex is destroyed or dysfunctional. As a result, the gland fails to produce adequate amounts of:
Cortisol (from zona fasciculata)
Aldosterone (from zona glomerulosa)
Androgens (from zona reticularis)
💉 What is Orthostatic Hypotension?
Orthostatic hypotension is a drop in blood pressure that occurs when standing up , often due to:
🧪 Aldosterone’s Role in Blood Pressure Regulation:
Aldosterone:
Promotes Na⁺ and water reabsorption in the distal nephron
Leads to expansion of plasma volume
Helps maintain blood pressure and electrolyte balance
✅ Why Hypoaldosteronism Causes Orthostatic Hypotension:
In Addison’s disease:
The zona glomerulosa is damaged → decreased aldosterone production
↓ Na⁺ reabsorption → ↓ water retention → hypovolemia
↓ Plasma volume → drop in BP , especially upon standing
This leads to orthostatic hypotension
❌ Why the Other Options Are Incorrect:
B. Hypercortisolism – ❌
Not seen in Addison’s; cortisol is low , not high.
Excess cortisol causes hypertension , not hypotension.
C. Hypoandrogenism – ❌
Addison’s can cause low adrenal androgens.
But androgens have minimal effect on BP —especially in adults.
Doesn’t explain orthostatic hypotension .
D. Hyperaldosteronism – ❌
This would cause hypertension , not hypotension.
It’s the opposite of what occurs in Addison’s disease.
E. None of these – ❌
Think about which condition actively stimulates the thyroid gland rather than simply releasing stored hormone. The immune system isn’t destroying the gland here — it’s tricking it into working overtime.
2 / 232
Tags:
2018
Which of the following is an autoimmune disorder resulting in hyperthyroidism?
🔬 Graves Disease – Overview:
Graves disease is the most common cause of hyperthyroidism and is a classic example of an autoimmune disorder .
Key Mechanism:
Histology:
✅ Thus, Graves disease is an autoimmune disease that causes hyperthyroidism .
❌ Why the Other Options Are Incorrect:
B. Hashimoto thyroiditis – ❌ Incorrect
Autoimmune, but causes hypothyroidism , not hyperthyroidism.
Characterized by anti-TPO and anti-thyroglobulin antibodies .
Histology: lymphoid aggregates, Hürthle cells , follicular destruction.
C. Subacute lymphocytic thyroiditis – ❌ Incorrect
Autoimmune, painless , and may have a brief phase of mild hyperthyroidism , but the end result is usually hypothyroidism .
No stimulating antibodies like in Graves.
D. De Quervain thyroiditis (Subacute granulomatous) – ❌ Incorrect
Follows a viral infection .
Not autoimmune.
Presents with painful thyroid , often after a URI.
Initial thyrotoxic phase due to follicular rupture, but it’s transient and not antibody-mediated.
E. Postpartum thyroiditis – ❌ Incorrect
Variant of subacute lymphocytic thyroiditis.
Occurs within 1 year of delivery.
May have brief hyperthyroid phase , but it’s due to release of stored hormone , not overproduction.
Also ends in hypothyroidism , often transient.
Chronic inflammation can lead to cellular transformation. Think about which thyroid disorder involves long-standing autoimmune attack, promoting cellular metaplasia and an increase in mitochondrial content within follicular cells.
3 / 232
Tags:
2018
Hürthle cells are present in which of the following?
🔬 What Are Hürthle Cells?
🦠 Hürthle Cells in Hashimoto Thyroiditis:
Hashimoto thyroiditis is the most common cause of hypothyroidism in iodine-sufficient areas.
It is an autoimmune thyroiditis , with:
Lymphoid aggregates with germinal centers
Plasma cell infiltration
Destruction of thyroid follicles
Prominent Hürthle cell change due to chronic injury
✅ Thus, Hürthle cells are a hallmark feature of Hashimoto thyroiditis .
❌ Why the Other Options Are Incorrect:
A. Graves disease – ❌ Incorrect
Autoimmune hyperthyroidism .
Histology: Hyperplastic, tall columnar follicular epithelium , scalloped colloid.
No Hürthle cells present.
B. De Quervain thyroiditis (Subacute granulomatous thyroiditis) – ❌ Incorrect
Typically follows a viral infection .
Histology: Granulomatous inflammation , multinucleated giant cells , disrupted follicles.
Hürthle cells are not a feature .
C. Subacute lymphocytic thyroiditis – ❌ Incorrect
Mild, painless autoimmune thyroiditis.
Lymphocytic infiltration is present, but Hürthle cell change is minimal or absent .
Often seen postpartum or in silent thyroiditis.
E. Postpartum thyroiditis – ❌ Incorrect
A form of subacute lymphocytic thyroiditis that occurs after delivery.
Lymphocytic infiltrates, but no prominent Hürthle cells .
Consider which pancreatic cells would be targeted in a disease where the body can no longer produce the hormone responsible for lowering blood glucose. Think about the consequences of this loss and which hormone therapies are required for treatment.
4 / 232
Tags:
2018
Which of the following cells is destroyed in type 1 diabetes mellitus?
T1DM is an autoimmune disease .
It typically presents in childhood or adolescence , but can occur at any age.
The immune system mistakenly attacks the pancreatic β-cells in the islets of Langerhans , leading to insulin deficiency .
Without insulin, the body can’t take up glucose effectively, causing hyperglycemia , ketoacidosis , and long-term complications if untreated.
🧬 Pancreatic Islet Cell Types:
Let’s review the key islet cells and their hormones:
Cell Type
Hormone Secreted
Function
Alpha (α)
Glucagon
Raises blood glucose by glycogenolysis and gluconeogenesis
Beta (β)
Insulin
Lowers blood glucose by promoting uptake into cells
Delta (δ)
Somatostatin
Inhibits insulin, glucagon, and GH release
PP Cells
Pancreatic polypeptide
Regulates GI activity, inhibits pancreatic exocrine secretion
🔍 In T1DM , β-cells are selectively destroyed by cytotoxic T lymphocytes (CD8⁺) , sometimes accompanied by autoantibodies against:
❌ Why the other options are incorrect:
A. Alpha cells – ❌ Incorrect
These produce glucagon , not insulin.
They remain intact in T1DM and may even become hyperactive due to the loss of insulin’s regulatory inhibition.
B. Pancreatic polypeptide cells (PP cells) – ❌ Incorrect
C. Delta cells – ❌ Incorrect
E. None of these – ❌ Incorrect
When the adrenal cortex fails to deliver, other glands step up to compensate. Think about which hormone might go into overdrive, and consider what it shares with the molecule that darkens the skin. The answer lies in their shared origin.
5 / 232
Tags:
2018
Which of the following is responsible for hyperpigmentation in Addisons disease?
Addison’s disease is primary adrenal insufficiency , meaning the adrenal cortex is damaged and cannot produce sufficient amounts of:
This leads to a loss of negative feedback to the hypothalamus and anterior pituitary, resulting in:
Increased CRH
Increased ACTH
🎨 Why does hyperpigmentation occur?
ACTH is derived from a larger precursor molecule : 🧬 Pro-opiomelanocortin (POMC) POMC is cleaved into several products:
⬇️ Now here’s the key: When ACTH production increases massively (as in Addison’s), so does MSH .
❌ Why the other options are incorrect:
B. Angiotensin – ❌ Incorrect
Angiotensin II stimulates aldosterone release , and causes vasoconstriction .
It plays no role in skin pigmentation.
C. Corticotropin-releasing hormone (CRH) – ❌ Incorrect
CRH is secreted by the hypothalamus , and it stimulates the anterior pituitary to release ACTH .
CRH itself does not directly affect pigmentation , and it does not elevate enough to cause hyperpigmentation.
D. Aldosterone – ❌ Incorrect
Produced by the zona glomerulosa of the adrenal cortex.
Regulates sodium and water balance .
Has no direct effect on melanocytes or pigmentation.
E. Cortisol – ❌ Incorrect
Cortisol, when deficient , causes ACTH to increase (due to lack of negative feedback).
But cortisol itself does not cause pigmentation —its absence indirectly contributes by increasing ACTH .
When the nerves are too easily triggered, sometimes the reason lies in the bloodstream’s missing mineral — not in the thyroid’s behavior.
6 / 232
Tags:
2018
Chvostek sign is seen in which of the following?
👋 What is Chvostek Sign?
It’s a clinical test for neuromuscular excitability , often due to low calcium (hypocalcemia) .
Tap over the facial nerve (just anterior to the ear).
A positive sign : Twitching of the facial muscles (esp. upper lip, cheek, or eyelid).
🧪 Why does it happen in Hypoparathyroidism?
➡️ In hypoparathyroidism , PTH is deficient → hypocalcemia develops → increased neuromuscular excitability → positive Chvostek sign
❌ Why the Other Options Are Incorrect:
Hyperparathyroidism ❌ → Causes hypercalcemia , not hypocalcemia. Would not produce Chvostek sign.
Hyperthyroidism ❌ → May have other signs like tremor, heat intolerance, tachycardia — not Chvostek sign.
Hypercalcemia ❌ → Calcium levels are high → reduces neuromuscular excitability → Chvostek sign is absent .
Both hyperthyroidism and hypothyroidism ❌ → Neither is directly associated with Chvostek sign. Only hypoparathyroidism causes the specific hypocalcemia that leads to it.
Even essentials, when over-supplied, can shut down the system they support — especially when the body tries to protect itself from overstimulation.
7 / 232
Tags:
2018
What does highly excessive iodine cause?
Although iodine is essential for thyroid hormone synthesis (T3 and T4) , excessive iodine can actually inhibit thyroid hormone production — a phenomenon known as the Wolff–Chaikoff effect .
🌊 The Wolff–Chaikoff Effect:
When the thyroid is exposed to very high levels of iodine , it temporarily stops producing thyroid hormone .
This is a protective autoregulatory mechanism to prevent excessive hormone synthesis.
If the effect persists (especially in people with underlying thyroid disease), it can lead to hypothyroidism .
✅ Therefore, excessive iodine can cause hypothyroidism , especially in:
People with autoimmune thyroiditis (e.g., Hashimoto’s)
Neonates
Patients receiving iodine-containing drugs (like amiodarone)
❌ Why the Other Options Are Incorrect:
Cushing syndrome ❌ → Caused by excess cortisol , not related to iodine levels.
Exophthalmos ❌ → Seen in Graves’ disease (autoimmune hyperthyroidism) , which is not caused by excess iodine .
Hyperthyroidism ❌ → Excess iodine can rarely cause hyperthyroidism in patients with nodular thyroid (Jod-Basedow phenomenon), but hypothyroidism is more common due to Wolff–Chaikoff effect.
None of these ❌ → Incorrect, because hypothyroidism is a well-documented effect of iodine excess.
When a hormone responsible for brain and bone development is missing at life’s earliest stages, the consequences can be tragically permanent.
8 / 232
Tags:
2018
🔍 What is Cretinism ?
Cretinism is the severe form of congenital or early-onset hypothyroidism , and it leads to:
It is now referred to as congenital hypothyroidism , and is often screened for at birth through neonatal TSH/T4 testing .
🧬 Why is it so dangerous?
Thyroid hormone (especially T3/T4) is essential for brain development and myelination , particularly in the first 2–3 years of life .
Without early diagnosis and treatment (levothyroxine), the damage is irreversible .
❌ Why the Other Options Are Incorrect:
Hyperparathyroidism during adult life ❌ → Causes calcium-related issues (e.g., stones, bones, groans), not cretinism.
Hyperthyroidism during childhood ❌ → Causes increased metabolism, not growth delay or cognitive impairment.
None of them ❌ → Incorrect because hypothyroidism in early life is a well-known cause of cretinism.
Increased growth hormone ❌ → Leads to gigantism (in children) or acromegaly (in adults), not cretinism.
When an immune army sits in one place too long, it may eventually forget its mission — and start multiplying in ways it shouldn’t.
9 / 232
Tags:
2018
Hashimoto thyroiditis can lead to:
🔬 Hashimoto thyroiditis (Chronic lymphocytic thyroiditis):
An autoimmune destruction of the thyroid gland
Common in middle-aged women
Involves anti-thyroid peroxidase (anti-TPO) and anti-thyroglobulin antibodies
Leads to gradual hypothyroidism
Over time, the chronic inflammation results in:
Dense lymphocytic infiltration
Formation of germinal centers in the thyroid
And in rare cases, malignant transformation
🔥 Most Important Complication:
✅ Primary thyroid lymphoma — especially B-cell non-Hodgkin lymphoma — is a rare but well-established complication of long-standing Hashimoto thyroiditis.
❌ Why the Other Options Are Incorrect:
Pretibial myxedema ❌ → Occurs in Graves’ disease , not Hashimoto’s. It’s due to TSH receptor antibodies stimulating fibroblasts in the skin.
Pancytopenia ❌ → Not a typical feature of Hashimoto thyroiditis. May occur in aplastic anemia or marrow failure, not autoimmune thyroiditis.
Leukemia ❌ → No direct association. Hashimoto’s is associated with lymphoma, not leukemia.
All of them ❌ → Incorrect because only thyroid lymphoma is the true established complication among the choices.
When a cell chronically exposed to injury or autoimmune attack changes its identity to adapt — it’s not growing, shrinking, or mutating. It’s transforming.
10 / 232
Tags:
2018
🔬 What are Hurthle cells?
Also known as oncocytic cells , Hurthle cells are:
Large epithelial cells with abundant granular eosinophilic cytoplasm
Have prominent nucleoli
Contain numerous mitochondria
Typically arise in the thyroid gland (especially in Hashimoto’s thyroiditis, but also in neoplasms)
🧬 Why “Metaplastic change”?
Hurthle cells are considered to result from a metaplastic transformation of normal follicular epithelial cells in response to chronic injury or inflammation — most commonly in autoimmune thyroiditis (Hashimoto’s disease).
So, they don’t arise due to overgrowth (hyperplasia) or abnormal size increase (hypertrophy), but rather due to cell type transformation , which is the definition of metaplasia .
❌ Why the Other Options Are Incorrect:
Atrophy ❌ → Refers to shrinking or wasting of tissue — Hurthle cells are actually larger than normal.
Dysplasia ❌ → Refers to abnormal cellular architecture , often precancerous — Hurthle cells are not dysplastic by default.
Hyperplasia ❌ → Means increase in number of normal cells — not necessarily involving cell type change.
Hypertrophy ❌ → Means increase in cell size , but Hurthle cells are not just enlarged — they’re transformed into a different cell type with different cytoplasmic features.
🧪 Where are Hurthle cells seen?
Hashimoto’s thyroiditis ✅ (most common)
Hurthle cell adenoma/carcinoma
Sometimes in Graves’ disease and other thyroid disorders
When the thermostat in the body is malfunctioning, the best way to confirm it is to check the signal that controls the temperature — not the temperature itself.
11 / 232
Tags:
2018
A patient presented to the clinic with the symptoms of hyperthyroidism. Which of the following should be assessed to confirm the diagnosis?
When a patient presents with signs and symptoms of hyperthyroidism (e.g., weight loss, heat intolerance, palpitations, tremors, irritability), the first and best screening test is to measure TSH (Thyroid-Stimulating Hormone) .
🔬 Why TSH?
TSH is secreted by the anterior pituitary .
It is extremely sensitive to changes in thyroid hormone levels.
It shows inversely related changes:
🔎 In hyperthyroidism , TSH is typically:
Low in primary hyperthyroidism (e.g., Graves’ disease)
Could be high or normal in secondary/central hyperthyroidism (rare — pituitary TSH-secreting adenoma)
After TSH, if abnormal:
❌ Why the Other Options Are Less Preferred or Incorrect:
Thyroglobulin ❌ → Used as a tumor marker in thyroid cancer , not for hyperthyroidism diagnosis.
TRH ❌ → Hypothalamic hormone, rarely measured clinically . Not useful as an initial test.
Free T3 ❌ → Elevated in hyperthyroidism, but not the first test . T3 can be normal in early/mild disease.
T4 ❌ → Also increases in hyperthyroidism, but TSH is more sensitive and specific for diagnosis.
🩺 Clinical Flow for Suspected Hyperthyroidism:
TSH (First-line screening)
If TSH is low → check Free T4 and Free T3
Consider antibody tests (e.g., TSI, TRAb) if Graves’ suspected
RAIU scan if nodular disease suspected
When the body finishes using its stress hormones, it sends the leftovers out through a route that doctors can easily measure — especially when a tumor is making too much of them.
12 / 232
Tags:
2018
Which of the following is used to check catecholamine degradation?
Catecholamines like epinephrine , norepinephrine , and dopamine are synthesized in the adrenal medulla and sympathetic nervous system. After they are used, the body metabolizes (breaks down) them into inactive products , which are then excreted in the urine.
🔄 Key Degradation Pathway:
🧪 Clinical Use:
❌ Why the Other Options Are Wrong:
Glucose tolerance test ❌ → Used to diagnose diabetes mellitus , not catecholamine metabolism.
Complete blood count (CBC) ❌ → Measures red and white blood cells and platelets — no relation to catecholamine breakdown.
Lactate threshold test ❌ → Used in sports medicine/physiology to evaluate endurance and aerobic capacity — unrelated to adrenal function.
None of these ❌ → Incorrect, because VMA in urine is the correct and widely used test.
When the blood pressure refuses to behave and the glands above the kidneys are both large — ask yourself: what stress hormone factory lives there and loves to cause chaos?
13 / 232
Tags:
2018
A 34-year-old female comes to the outpatient department with uncontrolled hypertension. A computed tomography (CT) scan reveals bilateral enlargement of the suprarenal gland. What is the most probable cause?
👩⚕️ Clinical Presentation Summary:
This is highly suggestive of a catecholamine-secreting tumor — the most classic one being:
⚡ Pheochromocytoma
Arises from the adrenal medulla
Secretes excessive catecholamines (epinephrine, norepinephrine, dopamine)
Causes:
🔎 It may be unilateral or bilateral — bilateral forms are often seen in familial syndromes (like MEN 2A/2B, von Hippel–Lindau, NF1).
❌ Why the Other Options Are Incorrect:
Renal cyst ❌ → Benign and usually asymptomatic . Doesn’t involve adrenal glands or cause hypertension directly.
Renal tumor ❌ → Could cause hypertension, but the question mentions suprarenal gland enlargement , not renal (kidney) mass.
Hyperplasia of suprarenal cortex ❌ → Causes Cushing’s syndrome (cortisol excess) or Conn’s syndrome (aldosterone excess). These cause gradual onset hypertension, not typically sudden, and are rarely bilateral with acute symptoms .
Hyperplasia of suprarenal medulla ❌ → Not a commonly used term in clinical diagnosis. Medullary overgrowth usually refers to a pheochromocytoma , which is a tumor , not “hyperplasia” per se.
🧪 Confirmatory Tests for Pheochromocytoma:
Plasma free metanephrines
24-hour urine catecholamines
MIBG scan (to localize catecholamine-secreting tumors)
When checking whether the command system is working properly, would you first look at the worker (hormone levels) or the boss (the signal that controls them)? Start with the signal.
14 / 232
Tags:
2018
Which of the following is the very first test to be done for hypothyroidism?
When a patient is suspected to have hypothyroidism (e.g., weight gain, cold intolerance, fatigue, constipation, etc.), the first and most important test you should order is TSH .
Why?
Because TSH is the most sensitive and reliable screening test to detect thyroid dysfunction. Here’s how:
🔄 How the Thyroid Axis Works:
The hypothalamus releases TRH
TRH stimulates the pituitary to release TSH
TSH stimulates the thyroid gland to make T3 and T4
🧪 What Happens in Hypothyroidism?
In Primary Hypothyroidism (most common):
In Central Hypothyroidism (pituitary or hypothalamus problem):
So, measuring TSH tells you:
❌ Why the Other Options Are Wrong (or Secondary Tests):
Total T4 and T3 ❌ → Affected by thyroid-binding globulin levels; not the first or best screening test.
Free T3 ❌ → Less reliable in hypothyroidism (T3 often stays normal in early hypothyroidism).
T3 ❌ → Poor marker for hypothyroidism. Levels fluctuate and can be misleading.
T4 ❌ → Helpful, but it should be tested after TSH . Used to confirm diagnosis if TSH is abnormal .
🩺 Clinical Flow:
Start with TSH
If TSH is abnormal → then check Free T4 (and sometimes T3) to classify the type and severity
Only one of these conditions involves an immune system “trick” that pushes the gland into overdrive. Others either fail to make enough hormone — or can’t make it at all.
15 / 232
Tags:
2018
Which of the following conditions causes hyperthyroidism?
🔥 Hyperthyroidism = A state where the thyroid gland produces too much thyroid hormone (T3 and T4) .
This leads to symptoms like:
Weight loss despite normal/increased appetite
Heat intolerance
Tachycardia, anxiety, tremors
Diarrhea, hyperreflexia
Menstrual irregularities
👑 Graves disease is the most common cause of hyperthyroidism.
It is an autoimmune condition where the body produces TSH receptor antibodies (TRAb) . These antibodies:
Also causes exophthalmos (eye bulging) and pretibial myxedema , which are unique to Graves.
❌ Why the Other Options Are Incorrect:
Congenital hypothyroidism ❌ → This causes low thyroid hormone , not excess. Leads to mental retardation and stunted growth if untreated.
Hashimoto thyroiditis ❌ → It is also autoimmune , but it causes destruction of the thyroid gland , leading to hypothyroidism .
Iodine deficiency ❌ → Iodine is needed to make T3 and T4. Deficiency leads to decreased production → hypothyroidism , not hyperthyroidism.
All of them ❌ → Incorrect because only Graves disease actually causes hyperthyroidism as a primary pathology .
What if a machine isn’t broken, but no one presses the “start” button? Think of a healthy organ waiting for a signal that never comes.
16 / 232
Tags:
2018
Which of the following is the cause of central hypothyroidism?
Central hypothyroidism means the problem is not in the thyroid gland itself , but rather in the central command system — the hypothalamus or pituitary .
Let’s recall the hormone cascade:
🧠 Hypothalamus → releases TRH 🧠 Pituitary → releases TSH 🦋 Thyroid gland → makes T3 and T4
In central hypothyroidism , there’s a problem with either:
🧪 Result: The thyroid is not being stimulated enough , so it doesn’t produce enough T3/T4 — even though the gland itself is healthy.
❌ Why the Other Options Are Wrong:
Defect in thyroid gland ❌ → That’s primary hypothyroidism , not central. Here the thyroid itself is damaged (e.g., Hashimoto’s).
Decreased free T3 ❌ → That’s an effect , not a cause. Many conditions can lower free T3. Doesn’t tell you where the problem is.
None of them ❌ → Incorrect, because we do have a correct choice: insufficient TSH stimulation .
Decreased T4 ❌ → Also an effect of central hypothyroidism, not the cause. It’s what happens because of low TSH.
🧩 Recap Mnemonic:
Primary hypothyroidism = Problem in the gland
Secondary hypothyroidism = Problem in the pituitary
Tertiary hypothyroidism = Problem in the hypothalamus
All of these are central causes , because they’re “upstream”
Sometimes, inflammation in an endocrine gland follows a more general illness like a cold or flu. Think about which type of common insult could cause a painful thyroid without needing antibiotics or surgery.
17 / 232
Tags:
2018
Thyroiditis is the inflammation of the thyroid gland encompassing a number of disorders that have some elements of inflammation. Subacute thyroiditis is believed to be triggered by which of the following insults?
🔥 Subacute Thyroiditis (also called De Quervain’s thyroiditis or granulomatous thyroiditis )
🔹 Cause:
It is **most commonly triggered by a viral infection or follows a viral upper respiratory illness (like coxsackievirus, mumps, adenovirus, or echovirus).
Often affects middle-aged women.
🔹 Clinical Picture:
Painful, tender thyroid gland
May have fever, sore throat, and neck pain that can radiate to the jaw or ears.
Patients usually go through phases:
Hyperthyroid phase (due to leakage of preformed thyroid hormone)
Hypothyroid phase (transient)
Recovery phase
🔹 Labs:
❌ Why the Other Options Are Incorrect:
Iodine deficiency ❌ → Causes goiter or hypothyroidism, not subacute thyroiditis .
Bacterial infection ❌ → Can cause acute (suppurative) thyroiditis , which is rare and usually seen in immunocompromised patients or those with anatomical defects like a piriform sinus fistula.
Trauma ❌ → Rarely causes thyroid inflammation directly; may cause hemorrhage into a thyroid nodule but not classical subacute thyroiditis.
Fungal infection ❌ → Extremely rare and mostly seen in immunocompromised patients — not a typical cause of subacute thyroiditis.
Some health problems come from damage that’s deep and small — others from damage that’s wide and big. Think about what kinds of blood vessels feed organs like nerves versus the heart.
18 / 232
Tags:
2018
Which of the following is not a microvascular complication of long-standing diabetes?
In diabetes, when blood sugar stays high for many years, it damages blood vessels in the body. There are two types of blood vessels:
1. Microvascular (very small blood vessels)
These tiny vessels supply important organs like:
Eyes → leading to retinopathy (eye damage)
Kidneys → leading to nephropathy (kidney damage)
Nerves → leading to neuropathy (nerve damage)
These three are the classic microvascular complications — they happen because high sugar damages the walls of these small vessels.
2. Macrovascular (larger blood vessels)
These are the big blood vessels that supply your:
If these get damaged, you get big problems like:
So, while a heart attack is a dangerous complication of diabetes, it’s not a microvascular one — it’s from damage to big arteries .
❌ Why the Other Options Are Incorrect:
Diabetic neuropathy = ✅ Happens due to damage to small nerve blood vessels.
Retinopathy = ✅ Eye damage from small vessel injury.
Nephropathy = ✅ Kidney damage from tiny capillaries in the kidney.
None of these = ❌ Wrong, because we found one that is not a microvascular complication — the heart attack.
This benign tumor is the leading cause of primary hyperparathyroidism .
19 / 232
Tags:
2018
Which of the following is the most common parathyroid tumor?
Parathyroid adenoma is the most common cause of primary hyperparathyroidism , responsible for about 85–90% of cases.
It involves a benign clonal proliferation of one parathyroid gland, leading to excess parathyroid hormone (PTH) secretion.
This excess PTH causes hypercalcemia , often detected incidentally or via symptoms like kidney stones , bone pain , or neuropsychiatric changes .
❌ Explanation of Incorrect Options:
Parathyroid hyperplasia: ❌ Less common (~10–15%). Often involves all four glands and is more likely associated with MEN syndromes .
Parathyroid carcinoma: ❌ Extremely rare (<1%). Presents with very high calcium and PTH levels , and more severe symptoms.
Multiple endocrine neoplasia type 1 (MEN 1): ❌ Genetic syndrome involving parathyroid hyperplasia , pancreatic tumors , and pituitary tumors .
Multiple endocrine neoplasia type 2 (MEN 2): ❌ Usually involves medullary thyroid carcinoma , pheochromocytoma , and occasionally parathyroid hyperplasia—not adenoma.
Which option describes a wasting state typically linked with cancer or chronic infections — not hormone deficiency?
20 / 232
Tags:
2018
Which of the following is not seen in hypopituitarism?
Hypopituitarism refers to decreased secretion of one or more of the hormones produced by the anterior or posterior pituitary. Common features result from deficiency of specific pituitary hormones , including:
Dwarfism → due to growth hormone (GH) deficiency.
Amenorrhea and decreased libido → from gonadotropin (FSH/LH) deficiency.
Pallor → from MSH (melanocyte-stimulating hormone) deficiency, leading to reduced skin pigmentation.
However, cachexia , which is extreme weight loss and muscle wasting typically seen in chronic diseases or cancer , is not a typical feature of hypopituitarism. Patients may have reduced muscle mass or fatigue, but not the profound wasting seen in cachexia.
❌ Breakdown of Incorrect Options:
Dwarfism ✅ – GH deficiency in childhood.
Decreased libido ✅ – Gonadotropin (FSH/LH) deficiency.
Pallor ✅ – Decreased MSH secretion.
Amenorrhea ✅ – Gonadotropin (FSH/LH) deficiency.
Cachexia ❌ – Not a standard feature of hypopituitarism; usually related to cancer, AIDS, or end-stage organ failure.
Think of the thyroid’s visible reaction to not getting enough iodine — it tries to grow bigger to compensate.
21 / 232
Tags:
2018
Which of the following is the most obvious clinical manifestation of long-standing iodine deficiency?
The most obvious and visible clinical sign of long-standing iodine deficiency is the development of a goiter , which is an enlargement of the thyroid gland .
When iodine is deficient, the thyroid cannot make enough thyroid hormone. In response, the pituitary gland secretes more TSH , which causes the thyroid to grow — leading to a goiter .
❌ Why the other options are incorrect:
Reproductive failure – Possible, but not the most obvious or visible sign.
Mental retardation – Can occur in severe congenital iodine deficiency (as in cretinism), but it’s a later or severe consequence .
Endemic cretinism – A serious complication in newborns due to maternal deficiency, not the most common or early sign in the general population.
Abortion – Can happen, but it is not the most visible or common sign in a population.
This condition involves antibodies that don’t destroy the cells but overstimulate a receptor on them. Which type of hypersensitivity fits that role?
22 / 232
Tags:
2018
Graves disease is an example of which type of hypersensitivity?
Graves disease is an autoimmune disorder in which the body produces autoantibodies that stimulate the TSH receptor on thyroid follicular cells. This leads to increased thyroid hormone production (hyperthyroidism).
This is classified as a Type II hypersensitivity reaction — but with a twist.
📌 Why Type II is Correct:
Type II hypersensitivity involves antibodies (usually IgG or IgM) directed against antigens on cell surfaces or tissues.
In Graves disease , the TSH receptor is the target.
The body produces stimulating autoantibodies (TSI – thyroid-stimulating immunoglobulins), which mimic TSH and overstimulate the thyroid .
Although Type II is commonly cytotoxic , this is an example of receptor stimulation , a non-cytotoxic Type II mechanism .
❌ Why the other options are incorrect:
Type I ❌ Involves IgE and immediate allergic reactions (e.g., anaphylaxis, asthma). Graves is not IgE-mediated .
Type III ❌ Caused by immune complex deposition (e.g., in SLE, post-streptococcal glomerulonephritis). Graves involves cell-surface receptor stimulation , not complex deposition.
Type IV ❌ This is delayed-type hypersensitivity mediated by T cells , such as in contact dermatitis or TB skin test. Graves is antibody-mediated .
None of them ❌ Incorrect because Graves disease clearly fits into the non-cytotoxic subtype of Type II hypersensitivity .
This crisis results from a condition where the immune system tricks the body into overproducing thyroid hormones — consider which disease causes the most dramatic overactivity of the thyroid.
23 / 232
Tags:
2018
Thyroid storm is seen in which of the following diseases?
Thyroid storm is a life-threatening medical emergency caused by a sudden and extreme overproduction of thyroid hormones . It often occurs in patients with untreated or poorly controlled hyperthyroidism , most commonly due to Grave’s disease , especially when triggered by:
Infection
Surgery
Trauma
Childbirth
In thyroid storm, patients present with:
Grave’s disease is an autoimmune condition where antibodies (TSI – thyroid-stimulating immunoglobulins) mimic TSH and overstimulate the thyroid, leading to excess T3 and T4 .
When this system goes into overdrive — such as during stress or illness — it may trigger thyroid storm .
❌ Why the other options are incorrect:
Wolff-Chaikoff effect ❌ This is a protective response where excess iodine actually suppresses thyroid hormone synthesis. It is not a disease, and it doesn’t cause thyroid storm.
Myxedema coma ❌ This is the opposite of thyroid storm — it occurs in severe hypothyroidism , leading to bradycardia, hypothermia, and coma.
Hashimoto’s thyroiditis ❌ An autoimmune disease that causes chronic hypothyroidism , not hyperthyroidism. No link to thyroid storm.
Hypothyroidism ❌ By definition, thyroid hormone levels are low — no excess T3/T4 to trigger thyroid storm.
Think about how we test a gland’s ability to respond to stimulation, not just resting hormone levels.
24 / 232
Tags:
2018
Which of the following is the confirmation test for Addison’s disease?
Addison’s disease is a primary adrenal insufficiency , meaning the adrenal cortex is damaged and cannot produce adequate cortisol (and sometimes aldosterone).
While measuring cortisol , ACTH , and electrolytes can suggest adrenal insufficiency, the gold standard test for confirming the diagnosis is the ACTH stimulation test , also known as the cosyntropin stimulation test .
In this test:
Synthetic ACTH (cosyntropin) is administered.
Cortisol levels are measured before and after (usually at 30 and 60 minutes).
In normal individuals , cortisol levels should rise significantly.
In Addison’s disease , cortisol levels remain low , confirming adrenal gland failure.
❌ Breakdown of Incorrect Options:
Glucose levels ❌ → Hypoglycemia can occur in Addison’s, but it’s nonspecific and not diagnostic.
Adrenocorticotropic hormone levels ❌ → ACTH is typically elevated in Addison’s disease due to lack of negative feedback, but this only suggests the condition — it’s not confirmatory.
Cortisol levels ❌ → Cortisol may be low , but a single low value isn’t conclusive; it can vary by time of day, stress, or illness. Needs dynamic testing.
Aldosterone levels ❌ → May be low in Addison’s, especially if mineralocorticoid function is impaired, but again, this is not confirmatory .
Think about what happens when a hormone that normally only increases after childbirth is chronically elevated in people who are not pregnant or breastfeeding.
25 / 232
Tags:
2018
What are the main clinical features of a prolactinoma?
A prolactinoma is a benign pituitary adenoma that overproduces prolactin , the hormone responsible for milk production. It is the most common type of functioning pituitary tumor .
Excess prolactin leads to suppression of gonadotropin-releasing hormone (GnRH) from the hypothalamus. This causes low FSH and LH , which disrupts reproductive function.
🔍 Main clinical features:
Galactorrhea (milk discharge from the breast, even in non-lactating women)
Amenorrhea (absence of menstruation)
Infertility (due to anovulation or hypogonadism)
In men , it can cause:
Decreased libido
Erectile dysfunction
Infertility
❌ Why the other options are incorrect:
Acne and weight gain ❌ → More common with Cushing’s syndrome or PCOS , not classic for prolactinomas.
Acromegaly, hyperostosis, and prognathism ❌ → Seen in GH-secreting tumors , not prolactinomas.
Vision disturbance, headaches, and mood swings ❌ → Can occur in large tumors due to mass effect, but these are non-specific . Not the primary hormone-related symptoms.
Hyperpigmentation and decreased libido ❌ → Suggests Addison’s disease (high ACTH), not prolactin excess.
Think about the hormone that directly affects your height as you grow through childhood and adolescence — and what happens if it’s missing.
26 / 232
Tags:
2018
Growth retardation occurs due to deficiency of which hormone?
Growth hormone (GH) is the primary hormone responsible for linear growth in children . It stimulates:
Liver and peripheral tissues to produce IGF-1 (insulin-like growth factor-1)
Cell proliferation and bone growth , particularly at the epiphyseal growth plates
When GH is deficient — either due to pituitary dysfunction or genetic causes — the result is growth retardation , often presenting as short stature in children. This condition is called dwarfism if the deficiency is severe and prolonged.
❌ Why the other options are incorrect:
Follicle stimulating hormone (FSH) ❌ → Regulates spermatogenesis and ovarian follicle development , not body growth.
Adrenocorticotropic hormone (ACTH) ❌ → Stimulates adrenal cortex to release cortisol. While important, ACTH deficiency alone doesn’t directly cause growth retardation .
Prolactin ❌ → Involved in milk production , not growth.
None of them ❌ → Incorrect, since GH clearly causes growth retardation when deficient .
Focus on the physical and metabolic changes caused by too much growth hormone after growth plates have fused. What doesn’t fit with this picture?
27 / 232
Tags:
2018
Which of the following is not associated with acromegaly?
Acromegaly is a condition caused by excess secretion of growth hormone (GH) in adults , most often due to a pituitary adenoma . Since epiphyseal plates have already closed in adults, the excess GH leads to soft tissue and bone overgrowth , rather than increased height.
Here’s what is commonly seen in acromegaly:
Frontal bossing ✅ → Thickened skull bones and prominent forehead are classic signs.
Insulin resistance ✅ → GH antagonizes insulin, often leading to hyperglycemia or even diabetes mellitus .
Hypertension ✅ → Seen frequently due to fluid retention and vascular changes.
Large tongue and hands ✅ → Macroglossia and enlarged extremities are hallmark features.
But:
Think about which hormone helps the body mobilize fat — without it, fat just piles up.
28 / 232
Tags:
2018
Deficiency of which hormone causes fatty liver?
Growth hormone (GH) plays a critical role in lipid metabolism , especially in the mobilization of fats and prevention of fat accumulation in the liver .
When there’s a deficiency of GH , the body’s ability to burn fat is reduced. This leads to:
Increased fat storage , particularly in the liver
Development of non-alcoholic fatty liver disease (NAFLD)
Decreased stimulation of IGF-1 , which also helps regulate metabolism
So, in GH deficiency , the liver starts accumulating fat, resulting in fatty liver .
❌ Why the other options are incorrect:
Oxytocin ❌ → Involved in uterine contraction and milk ejection . No role in liver metabolism or fat accumulation.
Somatostatin ❌ → Inhibits many hormones (including GH and insulin), but its deficiency does not cause fatty liver .
Prolactin ❌ → Main role is lactation . It doesn’t influence lipid metabolism to the extent that GH does.
None of them ❌ → Incorrect, because GH deficiency clearly leads to fatty liver .
Think about which symptom results from irreversible brain development issues early in life, not just hormone loss from the pituitary.
29 / 232
Tags:
2018
Which of the following does not occur in panhypopituitarism?
Panhypopituitarism is a condition where all (or nearly all) anterior pituitary hormones are deficient . This leads to multiple hormone deficiencies , causing widespread effects:
Key features of panhypopituitarism:
↓ GH → Reduced growth (especially in children)
↓ TSH → Reduced thyroxine (T4) → Symptoms of hypothyroidism
↓ LH/FSH → Hypogonadism → Infertility, delayed puberty, decreased libido
However…
❌ Mental retardation does not occur in panhypopituitarism
In panhypopituitarism , if thyroid hormone deficiency occurs after birth and is treated promptly , intellectual development remains normal.
So, mental retardation is not a typical feature of panhypopituitarism.
❌ Why other options are incorrect choices :
Hypogonadism ✅ Occurs due to ↓ LH and FSH
Reduced growth ✅ Due to ↓ GH
Reduced thyroxine ✅ Due to ↓ TSH
None of them ❌ Incorrect because mental retardation does NOT occur —so “none” isn’t the right answer.
Think about a situation where the body is failing to produce growth hormone , not where it’s producing too much. That’s when you’d consider replacing it.
30 / 232
Tags:
2018
In which of the following conditions is human growth hormone given?
Human Growth Hormone (hGH) is used only when there’s a deficiency of GH — most often due to panhypopituitarism , a condition where all or most anterior pituitary hormones are deficient .
In children, this leads to growth failure (dwarfism) , and in adults, it causes low energy, decreased muscle mass, and poor quality of life . Giving recombinant hGH in these patients replaces the missing hormone .
❌ Why the Other Options Are Incorrect:
Hyperpituitarism ❌ → This means too much pituitary hormone (like excess GH in acromegaly/gigantism). Giving GH here would worsen the condition .
Gigantism ❌ → Caused by excess GH in children before epiphyseal fusion. hGH is not given — treatment focuses on reducing GH (e.g., surgery, meds).
Acromegaly ❌ → Caused by excess GH in adults . Giving GH would make symptoms worse , not better.
All of them ❌ → Only panhypopituitarism involves GH deficiency that warrants treatment.
Think about what happens when a hormone that promotes growth keeps acting even after your bones have stopped growing longer . What kind of growth would you see?
31 / 232
Tags:
2018
A 40-year-old man comes to the OPD complaining of muscle weakness. He is 6 feet tall and has a protruding lower jaw, along with enlarged hands and feet. What is your diagnosis of the patient and what will you expect his lab tests to reveal?
This patient’s symptoms tell a clear clinical story:
👨⚕️ Clues from the case:
Age: 40 years old
Protruding jaw (prognathism)
Enlarged hands and feet
Muscle weakness
Tall stature — though he’s already an adult, this doesn’t mean he’s still growing.
These are classic features of acromegaly , which is caused by excess growth hormone (GH) after epiphyseal fusion (i.e., in adulthood). The GH acts on the liver to increase IGF-1 (Insulin-like Growth Factor 1), which causes soft tissue overgrowth, bone remodeling, and metabolic disturbances.
🧪 Expected lab results in acromegaly:
❌ Why the other options are incorrect:
Gigantism, with increased GH and IGF-1 ❌ → Gigantism occurs before puberty (before epiphyseal closure), causing excessive linear height gain , not in a 40-year-old adult.
Hypopituitarism, with increased GH ❌ → Hypopituitarism = decreased secretion of one or more pituitary hormones, not increased GH.
Hashitoxicosis, with increased T3 and T4, and low TSH ❌ → This is a transient hyperthyroid phase of Hashimoto’s thyroiditis. The symptoms don’t fit — no thyroid-related signs like tremor, palpitations, weight loss, or goiter.
Cushing syndrome, with increased midnight plasma cortisol ❌ → While muscle weakness may overlap, moon face, truncal obesity, and skin changes are hallmark Cushing signs — not prognathism and enlarged hands/feet.
In hormone-secreting tumors, think upstream: which mutation would lead to constant activation of a second messenger pathway without needing a receptor?
32 / 232
Tags:
2018
Genetic abnormalities are associated with pituitary adenomas. Which of the following is the most common genetic abnormality in pituitary adenomas?
The most common genetic abnormality in pituitary adenomas , especially somatotroph adenomas (which secrete growth hormone), involves:
✅ Mutations in the GNAS gene , which codes for the Gs alpha subunit of the G-protein
These mutations lead to constitutive activation of the G-protein
This keeps adenylyl cyclase active → ↑ cAMP → uncontrolled cell growth and hormone secretion
This is why it’s especially implicated in GH-secreting tumors (acromegaly)
So while cAMP is involved downstream, the actual mutation occurs in the G-protein .
❌ Why the Other Options Are Incorrect:
When thyroid hormone production is blocked due to environmental lack of an essential mineral, what visible compensatory sign shows up in the population?
33 / 232
Tags:
2018
Which of the following is the most common disease of the thyroid in mountainous areas?
In mountainous areas , soils and water often lack iodine , a key mineral needed for the synthesis of thyroid hormones (T3 and T4) .
Low iodine → ↓ thyroid hormone synthesis
The pituitary responds by releasing more TSH (thyroid-stimulating hormone)
TSH causes the thyroid gland to enlarge , forming a goiter
When this occurs in a large portion of the population in a region, it’s called:
✅ Endemic goiter – the most common thyroid disease in iodine-deficient (mountainous) areas
It is a preventable public health issue and has largely declined in areas with iodized salt programs .
❌ Why the Other Options Are Incorrect:
Myxedema ❌ Refers to severe hypothyroidism , usually due to autoimmune or surgical causes — not common geographically.
Hashimoto’s thyroiditis ❌ Autoimmune hypothyroidism; not linked to iodine deficiency or geography.
Graves disease ❌ Autoimmune hyperthyroidism — occurs worldwide but isn’t associated with mountainous regions specifically.
Iatrogenic goiter ❌ Caused by medical treatment , such as lithium or amiodarone — not related to iodine-poor environments.
Think about which endocrine gland regulates both calcium and phosphate levels inversely. If a tumor damages that gland instead of overactivating it, what changes would you expect in these electrolytes?
34 / 232
Tags:
2018
A 56-year-old man presents to the outpatient department with pain in his phalanges. He is found to have high levels of parathyroid hormone and hypercalcemia. What is the most likely diagnosis?
Let’s interpret this question systematically by focusing on the clinical and biochemical picture :
🔍 Key findings:
Lightheadedness and bone pain: suggest possible electrolyte imbalances and/or bone resorption abnormalities .
Low calcium levels → hypocalcemia .
High phosphate levels → suggests loss of parathyroid hormone (PTH) function.
Low calcitonin → rules out medullary thyroid carcinoma , which secretes calcitonin .
🧠 Now, step-by-step:
Parathyroid hormone (PTH) :
Normally increases serum calcium and decreases serum phosphate by increasing renal excretion of phosphate.
So, low calcium + high phosphate = low PTH effect.
Parathyroid carcinoma :
Usually causes hypercalcemia due to excessive PTH secretion.
But advanced or destructive lesions may impair PTH secretion , leading to functional hypoparathyroidism and the presented lab findings.
Bone pain and lightheadedness may also reflect chronic mineral imbalance or osteodystrophy.
The key clue is that none of the thyroid carcinomas (A, B, C, E) are associated with low calcium and high phosphate . They do not fit the endocrine profile.
❌ Why the Other Options Are Incorrect:
A) Anaplastic carcinoma : Highly aggressive, rapid growth, local invasion—no specific endocrine features or calcitonin involvement. Doesn’t explain calcium/phosphate imbalance.
B) Follicular thyroid carcinoma : May metastasize hematogenously, but no parathyroid or calcitonin involvement . No hormonal secretion.
C) Papillary thyroid carcinoma : Most common thyroid cancer, slow growing, spreads via lymphatics. Does not affect calcium or phosphate metabolism .
E) Medullary thyroid carcinoma : Arises from parafollicular C-cells → produces calcitonin → typically calcitonin is high , not low. So this is ruled out due to low calcitonin levels in this patient.
Think about which endocrine gland regulates both calcium and phosphate levels inversely. If a tumor damages that gland instead of overactivating it, what changes would you expect in these electrolytes?
35 / 232
Tags:
2018
A 30-year-old woman suffering from diabetes presents to the outpatient department with lightheadedness and bone pain. Her serum calcitonin and calcium levels are low and phosphate levels are high. What is the most likely diagnosis?
Let’s interpret this question systematically by focusing on the clinical and biochemical picture :
🔍 Key findings:
Lightheadedness and bone pain: suggest possible electrolyte imbalances and/or bone resorption abnormalities .
Low calcium levels → hypocalcemia .
High phosphate levels → suggests loss of parathyroid hormone (PTH) function.
Low calcitonin → rules out medullary thyroid carcinoma , which secretes calcitonin .
🧠 Now, step-by-step:
Parathyroid hormone (PTH) :
Normally increases serum calcium and decreases serum phosphate by increasing renal excretion of phosphate.
So, low calcium + high phosphate = low PTH effect.
Parathyroid carcinoma :
Usually causes hypercalcemia due to excessive PTH secretion.
But advanced or destructive lesions may impair PTH secretion , leading to functional hypoparathyroidism and the presented lab findings.
Bone pain and lightheadedness may also reflect chronic mineral imbalance or osteodystrophy.
The key clue is that none of the thyroid carcinomas (A, B, C, E) are associated with low calcium and high phosphate . They do not fit the endocrine profile.
❌ Why the Other Options Are Incorrect:
A) Anaplastic carcinoma : Highly aggressive, rapid growth, local invasion—no specific endocrine features or calcitonin involvement. Doesn’t explain calcium/phosphate imbalance.
B) Follicular thyroid carcinoma : May metastasize hematogenously, but no parathyroid or calcitonin involvement . No hormonal secretion.
C) Papillary thyroid carcinoma : Most common thyroid cancer, slow growing, spreads via lymphatics. Does not affect calcium or phosphate metabolism .
E) Medullary thyroid carcinoma : Arises from parafollicular C-cells → produces calcitonin → typically calcitonin is high , not low. So this is ruled out due to low calcitonin levels in this patient.
Consider what happens when a rapidly expanding mass in a tightly confined anatomical space suddenly loses its vascular integrity. How would such an event manifest clinically, and what term best describes this acute presentation?
36 / 232
Tags:
2018
Hypopituitarism can result from hemorrhage into a pituitary adenoma. What is this condition known as?
To understand this question, we need to examine the pathophysiological basis of hypopituitarism in the context of acute vascular events affecting the pituitary gland .
📌 What is Pituitary Apoplexy?
Pituitary apoplexy is a rare but life-threatening endocrine emergency that involves sudden hemorrhage or infarction (or both) within a pituitary adenoma . It leads to rapid pituitary gland dysfunction , causing acute hypopituitarism , along with compressive symptoms due to the gland’s proximity to the optic chiasm and cavernous sinus.
Patients typically present with:
Sudden severe headache (due to meningeal irritation)
Visual disturbances (like bitemporal hemianopia from optic chiasm compression)
Ophthalmoplegia (if cranial nerves III, IV, VI are affected)
Nausea/vomiting
Altered consciousness in severe cases
❌ Why the Other Options Are Incorrect:
A) None of these: This is incorrect because pituitary apoplexy is a well-defined and known condition associated with hemorrhage into a pituitary adenoma.
B) Sheehan syndrome: This is also a cause of hypopituitarism , but it specifically refers to ischemic necrosis of the pituitary gland following severe postpartum hemorrhage . It is not related to an adenoma or hemorrhage into a tumor.
C) Craniopharyngioma: This is a benign congenital tumor of the pituitary region that can cause chronic compressive hypopituitarism , but not acute hemorrhagic events . It does not result from bleeding into a tumor.
D) Empty sella syndrome: This refers to a radiological finding where the sella turcica appears empty due to herniation of the subarachnoid space. It may be asymptomatic or associated with chronic pituitary insufficiency, not acute hemorrhage.
Imagine a vital mineral that regulates the rhythm of the heart being poured into the bloodstream too quickly. What’s the first thing you’d expect to malfunction — the structure, the circulation, or the beat?
37 / 232
Tags:
2018
Rapid calcium infusion leads to:
Let’s break down the effects of rapid calcium infusion step-by-step.
🧠 Basic Physiology:
Calcium plays a vital role in:
Cardiac muscle contraction
Electrical conduction in the heart (via voltage-gated calcium channels)
Neuromuscular transmission
Because calcium directly influences myocyte depolarization and repolarization , a sudden increase in serum calcium levels — such as from rapid IV infusion — can dangerously affect cardiac rhythm .
⚠️ What happens during rapid calcium infusion ?
This is why calcium is administered slowly , especially in emergency settings like hyperkalemia or hypocalcemia , under ECG monitoring.
❌ Why the Other Options Are Incorrect:
Myocardial infarction : Not directly caused by calcium infusion. Infarction results from ischemia , not electrolyte imbalance.
Cardiac failure : While excess calcium can affect cardiac output , a single rapid infusion usually does not cause outright heart failure , though it may worsen pre-existing dysfunction.
Neurogenic shock : This involves autonomic nervous system disruption , not electrolyte changes. Not related to calcium infusion.
Hypercalcemia : Technically correct in a long-term sense , but not the best acute effect of a rapid infusion. Also, mild hypercalcemia doesn’t explain the immediate danger posed by rapid infusion — arrhythmia does.
In which condition does the body already produce too much of what adrenocortical therapy would provide—making such therapy not just unnecessary, but harmful?
38 / 232
Tags:
2018
Adrenocortical therapy is not given in:
To answer this correctly, we must understand what adrenocortical therapy is , and when it is indicated .
👉 What is Adrenocortical Therapy?
Adrenocortical therapy refers to the administration of corticosteroids (usually glucocorticoids like hydrocortisone, prednisone, or dexamethasone ) to replace deficient hormones in adrenal insufficiency or as anti-inflammatory/immunosuppressive agents .
Now, let’s examine each option:
✅ Cushing Disease (Correct Answer)
Cushing disease is caused by excess ACTH secretion , usually from a pituitary adenoma , which stimulates the adrenal glands to produce excess cortisol .
Therefore, adrenocortical therapy is contraindicated in this condition because cortisol levels are already abnormally high .
Giving more corticosteroids would worsen the hypercortisolism , leading to further metabolic complications (e.g., muscle wasting, hyperglycemia, hypertension).
❌ Addison’s Disease (Incorrect)
This is a classic form of primary adrenal insufficiency .
The adrenal cortex is destroyed or dysfunctional , leading to deficient cortisol and aldosterone production.
Adrenocortical therapy is essential to replace these missing hormones.
Without treatment, it can be life-threatening .
❌ Primary Adrenal Insufficiency (Incorrect)
This includes Addison’s disease and other conditions where the adrenal cortex fails.
Again, adrenocortical therapy is life-saving and absolutely indicated.
❌ Secondary Adrenal Insufficiency (Incorrect)
In this case, the pituitary fails to secrete ACTH , leading to low cortisol but often normal aldosterone (since it’s controlled by the renin-angiotensin system).
Still, glucocorticoid replacement (like hydrocortisone or prednisone) is necessary.
So, adrenocortical therapy is definitely given .
❌ None of them (Incorrect)
Consider which cause of hypoglycemia shows high insulin levels but low levels of the natural byproduct of insulin secretion, indicating external insulin use.
39 / 232
Tags:
2019
An elderly man is brought to the emergency room with altered levels of consciousness. His initial blood glucose measurement revealed severe hypoglycemia. There is no history of significant weight loss or gain, but he complains of body aches. His insulin levels are high, and his C-peptide levels are low. Family history reveals a 20-year-old son with type-1 diabetes mellitus. What is the most likely cause of hypoglycemia in this elderly patient with altered consciousness?
Key Clinical and Laboratory Findings:
Severe hypoglycemia with altered consciousness : urgent condition needing diagnosis.
High insulin levels but low C-peptide : this indicates exogenous insulin administration because C-peptide is produced endogenously alongside insulin.
No weight changes , but body aches reported (non-specific).
Family history of type 1 diabetes in son may suggest access to insulin or medications.
Understanding the Options:
1. Factitious hypoglycemia
Occurs when a person self-administers insulin or sulfonylureas covertly.
Exogenous insulin raises insulin levels but does not raise C-peptide , since C-peptide is secreted only by pancreatic beta cells.
Fits perfectly with high insulin + low C-peptide .
Common in healthcare workers, or those with access to insulin.
❌ Important to confirm, but in this scenario, most likely diagnosis .
2. Inadequate dietary intake
3. Sulfonylurea ingestion
Sulfonylureas stimulate endogenous insulin secretion, so both insulin and C-peptide would be elevated.
❌ Not compatible with low C-peptide.
4. Insulinoma
A pancreatic beta-cell tumor causing excess endogenous insulin secretion.
Both insulin and C-peptide levels are high .
❌ Not consistent with low C-peptide.
5. Alcohol excess
Summary:
The combination of high insulin with low C-peptide during hypoglycemia indicates exogenous insulin administration (factitious hypoglycemia) .
Final Answer:
✅ Factitious hypoglycemia
If one area of the thyroid is lighting up while the rest is quiet, consider what might make that one part act independently — and excessively — compared to the rest of the gland.
40 / 232
Tags:
2019
There is increased uptake of radioactive iodine in a solitary area of the thyroid gland. What is most likely present in that area?
🧬 What is a Toxic Adenoma?
A toxic adenoma is a benign, autonomously functioning thyroid nodule .
It produces thyroid hormones independently of TSH regulation.
This leads to:
This is classic for a solitary “hot” nodule — high iodine uptake limited to a single hyperfunctioning region , with suppression of the rest of the gland.
❌ Explanation of Incorrect Options:
Thyroglossal cyst 🔴 This is a developmental cyst from remnants of the thyroglossal duct. It does not take up iodine and appears cold on scans.
Pheochromocytoma 🔴 This is a catecholamine-secreting tumor of the adrenal medulla — unrelated to the thyroid gland.
Colloid nodule 🔴 Common benign thyroid lesion, but typically non-functioning → shows as a “cold” nodule (no increased uptake).
Follicular carcinoma 🔴 Malignant thyroid tumor. Usually non-functioning and appears cold on thyroid scans. Though some follicular neoplasms can uptake iodine, solitary hot nodules are almost never malignant .
Consider which condition causes excessive growth hormone after puberty, leading to enlargement of hands and feet rather than increased height.
41 / 232
Tags:
2019
A 40-year-old patient presents to the clinic with complaints of headache, decreased libido, and frequent episodes of hypoglycemia. He mentions that he has had to increase hat, ring, and shoe sizes twice now. Which of the following is most likely his diagnosis?
Key Clinical Features:
Headache and decreased libido suggest possible pituitary involvement.
Frequent hypoglycemia can occur due to excessive insulin-like growth factor 1 (IGF-1) action increasing glucose utilization.
Increase in hat, ring, and shoe size indicates enlargement of bones and soft tissues , especially in hands, feet, and skull.
Understanding Each Option:
1. Acromegaly
✅ Correct answer
Occurs due to excess growth hormone (GH) secretion after epiphyseal plate closure (adult onset) .
Characterized by enlargement of hands, feet, and facial bones (leading to increased hat, ring, and shoe size).
Headache and hypopituitarism symptoms (decreased libido) occur due to pituitary adenoma mass effect.
Frequent hypoglycemia can result from GH excess-induced insulin resistance followed by compensatory hyperinsulinemia or IGF-1 effects.
2. Hurler syndrome (mucopolysaccharidosis type I)
A genetic lysosomal storage disorder presenting in childhood with coarse facial features, developmental delay, and organomegaly.
No adult onset or increase in hat/shoe size.
❌ Incorrect.
3. Hunter syndrome (mucopolysaccharidosis type II)
4. β Thalassemia major
A severe hemoglobinopathy causing anemia, bone deformities (due to marrow expansion), and growth retardation.
No increase in hat/shoe size, and hypoglycemia is not typical.
❌ Incorrect.
5. Gigantism
Summary:
The combination of adult onset of bone enlargement , headache , hypopituitarism signs , and metabolic symptoms like hypoglycemia is characteristic of acromegaly .
Final Answer:
✅ Acromegaly
Consider which thyroid disorder causes pain due to inflammation and shows decreased iodine uptake despite hyperthyroidism.
42 / 232
Tags:
2019
A 35-year-old female presents to the clinic with complaints of palpitations, heat intolerance, unintentional weight loss, and a painful mass in her neck. The pain is increased by swallowing and coughing and radiated to the jaw. On examination, the mass moves upon swallowing and is tender. Her hands are clammy, her heart rate is 98 beats per minute and her blood pressure is 140/110 mmHg. Her serum TSH is low, free T3 and T4 levels are markedly increased, and ESR is increased. A Radioiodine uptake (RAIU) study shows decreased iodine uptake. What is the most likely diagnosis of this patient?
Key Clinical Features to Note:
Painful, tender thyroid swelling moving on swallowing, pain radiating to jaw, worsened by swallowing and coughing.
Symptoms of hyperthyroidism : palpitations, heat intolerance, weight loss.
Signs of hyperthyroidism : clammy hands, tachycardia, hypertension.
Increased ESR (inflammatory marker) suggests inflammation.
Radioiodine uptake (RAIU) is decreased , meaning the thyroid is not actively trapping iodine.
Low TSH, high free T3/T4 consistent with hyperthyroidism.
Interpretation of findings:
The painful thyroid and elevated ESR strongly suggest an inflammatory thyroiditis , not a primary hyperthyroid state caused by overproduction of thyroid hormone.
Decreased RAIU means thyroid hormone is released due to inflammation-induced follicular destruction (release of preformed hormone), not increased synthesis.
Grave’s disease would show diffusely increased RAIU .
Hashimoto’s thyroiditis is typically painless and presents with hypothyroidism or transient hyperthyroidism (Hashitoxicosis), with variable RAIU.
Postpartum thyroiditis and subacute lymphocytic thyroiditis are painless forms.
Analysis of Options:
1. Subacute lymphocytic thyroiditis
Painless thyroiditis, usually postpartum or autoimmune.
Tenderness and pain are absent or minimal .
❌ Less likely because this patient has a painful thyroid.
2. Postpartum thyroiditis
3. Subacute granulomatous thyroiditis (De Quervain thyroiditis)
✅ Correct answer
Characterized by a painful, tender thyroid gland , often following a viral illness.
Symptoms include hyperthyroidism due to release of preformed hormone.
ESR elevated due to inflammation.
RAIU is low because the gland is inflamed and hormone is released from damaged follicles, not synthesized.
Pain radiating to jaw is classic.
4. Grave’s disease
Hyperthyroidism with diffuse goiter , usually painless .
RAIU is increased .
Eye signs, pretibial myxedema may be present.
❌ Not consistent with painful thyroid and low RAIU.
5. Hashimoto thyroiditis
Summary:
This clinical picture of painful, tender thyroid, raised ESR, hyperthyroidism, and low RAIU is classic for subacute granulomatous thyroiditis (De Quervain thyroiditis) .
Final Answer:
✅ Subacute granulomatous thyroiditis (De Quervain thyroiditis)
Which neuropathy is most commonly associated with diabetes and presents with symmetric, distal sensory symptoms often described as “stocking-glove”?
43 / 232
Tags:
2019
A 72-year-old patient, with diabetes mellitus and hypertension, presents to the clinic with complaints of symmetric loss of sensation in distal extremities. The loss of sensation was initially only in the lower extremities but has now also started in the upper extremities. She sometimes also experiences a burning sensation in her feet, especially at night. She is currently taking metformin, lisinopril, and atorvastatin, but she tells you that she frequently forgets to take her medications. Her most recent laboratory reports show: fasting blood sugar level = 160 mg/dL and HbA1C= 7.6% What is the most likely diagnosis for the complaint the patient has presented with?
Key clinical features:
Symmetric sensory loss starting distally in the feet and progressing proximally and to upper extremities.
Burning pain especially at night.
Patient has diabetes with poor control (HbA1c = 7.6%).
Gradual progression , sensory > motor symptoms.
These are typical features of a length-dependent neuropathy .
Analysis of Options:
1. Mononeuritis multiplex
Usually asymmetric, affects multiple individual nerves.
Presents with acute or subacute nerve deficits.
❌ Not typical for symmetric distal sensory loss.
2. Diabetic lumbosacral plexopathy
Presents with asymmetric pain, weakness, and muscle wasting predominantly in the thigh, and is often painful.
Usually affects one leg.
❌ Not symmetric or distal.
3. Peripheral mononeuropathy
Involves a single nerve (e.g., median nerve in carpal tunnel syndrome).
❌ The patient has polyneuropathy with multiple nerves involved bilaterally.
4. Chronic inflammatory demyelinating polyneuropathy (CIDP)
An immune-mediated neuropathy with both motor and sensory symptoms , often progressive.
Can mimic diabetic neuropathy but typically presents with weakness and elevated CSF protein.
❌ Less likely given classic diabetic history and symptoms.
5. Distal symmetric polyneuropathy
✅ Correct answer
Most common form of diabetic neuropathy.
Characterized by symmetric, distal sensory loss (“stocking-glove” pattern), burning pain, especially nocturnal.
Sensory symptoms precede motor symptoms.
Fits well with the patient’s presentation and diabetic history.
Summary:
The patient’s symptoms are classic for diabetic distal symmetric polyneuropathy .
Final Answer:
✅ Distal symmetric polyneuropathy
Think about which organ is particularly sensitive to high levels of circulating fats and can become inflamed suddenly when triglycerides are extremely elevated.
44 / 232
Tags:
2019
A 34-year-old woman presents to the outpatient department for a check-up 1 year after she had been prescribed fenofibrate for a medical condition that causes deranged lipid profiles. She tells you that she often forgets to take her medication. Furthermore, she tells you that her maternal uncles have deranged lipid profiles for which they take medications. On physical examination, her height is 5’4″ (163 cm), weight is 82 kg (180 pounds) and BMI is 31.6. She has bilateral yellow plaques on her eyelids and nodular, firm, painless swellings on the extensor surfaces of her hands. Laboratory studies are significant for triglycerides = 1100 mg/dL (12.43 mmol/L). Which of the following conditions is this patient most likely at risk of developing?
Key Clinical Points:
Very high triglycerides (1100 mg/dL) — normal is less than 150 mg/dL.
Physical signs:
Family history of lipid disorders (maternal uncles).
BMI indicates obesity , a risk factor for lipid abnormalities.
What are the major risks associated with very high triglycerides?
Severe hypertriglyceridemia (usually >1000 mg/dL) is a well-known risk factor for acute pancreatitis .
Elevated LDL cholesterol and familial hypercholesterolemia are strongly linked to coronary heart disease , but extremely high triglycerides increase pancreatitis risk more dramatically.
Analysis of Options:
1. Pancreatitis
2. Coronary heart disease
3. Hepatitis
4. Appendicitis
5. Cholelithiasis
Summary:
With severe hypertriglyceridemia (>1000 mg/dL), the most immediate and dangerous risk is acute pancreatitis .
Final Answer:
✅ Pancreatitis
Think about the effects of severe blood loss during childbirth on a gland critical for hormone production, especially one controlling milk production.
45 / 232
Tags:
2019
A 32-year-old female, P1+0, who recently gave birth presents to the clinic with the inability to lactate, headaches, dizziness, and tachycardia. Laboratory investigations show low serum prolactin levels. Low to normal levels of prolactin levels are seen in which of the following conditions?
Clinical Context:
A woman who recently delivered is unable to lactate, which suggests a problem with prolactin secretion .
She has additional symptoms (headache, dizziness, tachycardia), which may indicate pituitary pathology.
Lab shows low to normal serum prolactin , which is abnormal post-partum, as prolactin is usually elevated to stimulate lactation.
Understanding Each Option:
1. Pregnancy
❌ Prolactin levels are high during pregnancy and lactation to prepare and maintain milk production.
So, low prolactin is not seen in pregnancy.
2. Multiple endocrine neoplasia type 1 (MEN-1)
❌ MEN-1 can cause pituitary adenomas , commonly prolactinomas , which increase prolactin levels.
Low prolactin is not typical.
3. Pituitary infarct
✅ Correct answer
This condition refers to Sheehan syndrome in postpartum women: ischemic necrosis of the pituitary due to blood loss during delivery.
Leads to hypopituitarism , including decreased prolactin secretion, causing inability to lactate .
Other hormones may also be low; symptoms include fatigue, dizziness, and hypotension/tachycardia.
Prolactin is low or low-normal .
4. Pituitary adenoma
❌ Pituitary adenomas may cause either hypersecretion (e.g., prolactinoma → high prolactin) or hyposecretion if large and compressive.
But most prolactin-secreting adenomas cause high prolactin , not low.
5. Base of Skull Fracture
❌ May cause pituitary stalk injury, potentially altering hormone secretion, but this is not the classical cause of low prolactin postpartum.
Less specific and less common in this clinical context.
Summary:
Pituitary infarct (Sheehan syndrome) is the classic cause of low prolactin levels in a postpartum woman, leading to inability to lactate.
Consider which vascular complication involves large arteries and leads to heart attacks and strokes, rather than problems with small vessels or the eyes alone.
46 / 232
Tags:
2019
What is the marked macrovascular complication of type 2 diabetes mellitus?
Macrovascular vs. Microvascular Complications in Diabetes:
Evaluating Each Option:
1. Accelerated arthritis
2. Diminishing vision
3. Tunnel vision
4. Accelerated atherosclerosis
✅ Correct answer
Diabetes accelerates the process of atherosclerosis in large arteries, leading to macrovascular complications such as myocardial infarction, stroke, and peripheral artery disease.
5. Retinal vein occlusion
Summary:
The marked macrovascular complication of type 2 diabetes mellitus is accelerated atherosclerosis .
Final Answer:
✅ Accelerated atherosclerosis
Which intervention directly targets cholesterol production at its source, with the strongest evidence of reducing heart attacks and strokes?
47 / 232
Tags:
2019
A person with cardiovascular disease wants to decrease his low-density lipoprotein (LDL) cholesterol level. What would be the preferred suggestion?
Understanding LDL and Cardiovascular Risk:
Evaluation of Each Option:
1. Niacin
Niacin (vitamin B3) can lower LDL moderately and raise HDL, but its use has decreased due to side effects and limited evidence on cardiovascular outcome benefit.
Not first-line for LDL lowering nowadays.
2. Weight loss
Weight loss can improve lipid profile including LDL , but its effect on LDL lowering is modest and more variable.
Still important for overall health, but not the most powerful LDL-lowering strategy.
3. Fibrates
4. Statins
Gold standard for lowering LDL cholesterol.
Statins inhibit HMG-CoA reductase, reducing cholesterol synthesis and increasing LDL receptor expression in the liver, effectively lowering LDL levels.
Proven to reduce cardiovascular events and mortality.
Recommended as first-line therapy in patients with established CVD to reduce LDL.
5. Exercise
Exercise improves overall cardiovascular health and raises HDL, but has limited direct effect on lowering LDL cholesterol .
Important as adjunctive lifestyle modification but not primary therapy for LDL lowering.
Summary:
Statins are the preferred, most effective, and evidence-based therapy for lowering LDL cholesterol in patients with cardiovascular disease.
Final Answer:
✅ Statins
If a disease causes tumors in multiple hormone-producing organs — including the parathyroids — and runs in families, what genetic syndrome could be the unifying diagnosis?
48 / 232
Tags:
2019
Parathyroid adenoma is most commonly associated with which of the following?
🔍 What is a Parathyroid Adenoma ?
A benign tumor of one (sometimes more) of the parathyroid glands.
It causes primary hyperparathyroidism , leading to:
Now, Let’s Analyze Each Option:
1. MEN-1 (Multiple Endocrine Neoplasia Type 1)
✅ Correct
2. Parathyroid carcinoma
❌ Incorrect
Rare cause of primary hyperparathyroidism .
Much less common than adenomas.
Parathyroid carcinoma is typically more aggressive and associated with extremely high calcium levels.
3. Hypoparathyroidism
❌ Incorrect
Opposite condition — low PTH levels, often due to surgical removal or autoimmune destruction.
Not associated with adenomas.
4. MEN-2 (Multiple Endocrine Neoplasia Type 2)
❌ Incorrect
5. Secondary hyperparathyroidism
❌ Incorrect
Summary:
The most common association of parathyroid adenoma is with MEN-1 syndrome , a hereditary condition affecting multiple endocrine organs.
✅ Correct Answer: MEN-1
Consider what kind of internal error could cause the body to attack the very cells that help regulate glucose — a process unrelated to lifestyle, but deeply tied to self-recognition.
49 / 232
Tags:
2019
Type I diabetes is associated with which of the following?
T1DM is a chronic autoimmune disease characterized by:
Destruction of pancreatic β-cells in the islets of Langerhans
Absolute insulin deficiency
Early onset (typically in children and adolescents, but can occur in adults)
The body mistakenly produces autoantibodies (e.g., anti-GAD, anti-insulin, anti-IA2) that target and destroy β-cells, leading to:
🔍 Key Pathologic Features:
HLA association (especially HLA-DR3 and HLA-DR4)
Often associated with other autoimmune diseases , like Hashimoto’s thyroiditis or celiac disease
Not primarily driven by lifestyle factors
❌ Explanation of Incorrect Options:
Metabolic syndrome
A cluster of conditions: insulin resistance, central obesity, hypertension, dyslipidemia, and impaired glucose tolerance .
Closely associated with Type II diabetes , not Type I.
Smoking
While it contributes to insulin resistance and cardiovascular disease , it’s not causally linked to T1DM .
More relevant in Type II diabetes and vascular complications .
Obesity
Major risk factor for Type II diabetes , due to insulin resistance .
T1DM patients are often lean at diagnosis , although some may gain weight later with insulin therapy.
Excess growth hormone
Seen in conditions like acromegaly , where GH excess leads to insulin resistance — again, relevant to Type II or secondary diabetes , not Type I.
Which condition results from a failure of the entire adrenal cortex, not just a tumor or hormone excess? Consider which gland must stop functioning to lose both cortisol and aldosterone.
50 / 232
Tags:
2019
Which of the following diseases affects both mineralocorticoid and glucocorticoid secretion?
🔍 What are mineralocorticoids and glucocorticoids?
Mineralocorticoids (like aldosterone ) — regulate sodium, potassium, and water balance.
Glucocorticoids (like cortisol ) — regulate metabolism, stress response, and immune function.
Both are secreted by the adrenal cortex :
Let’s evaluate each option:
1. Pheochromocytoma
❌ Incorrect
Tumor of the adrenal medulla , which secretes catecholamines (epinephrine/norepinephrine).
It does not affect mineralocorticoid or glucocorticoid secretion directly.
2. Waterhouse-Friderichsen syndrome
✅ Can affect both , but it’s a rare, acute cause of adrenal failure.
Caused by adrenal hemorrhage , often due to Neisseria meningitidis sepsis .
Results in sudden loss of both mineralocorticoids and glucocorticoids — similar to Addison’s but acute and fulminant .
Technically correct, but Addison’s is the more classic and chronic answer.
3. Cushing syndrome
❌ Incorrect
Caused by excess cortisol (glucocorticoid), due to tumors or exogenous steroids.
Mineralocorticoid secretion is usually normal or mildly affected.
4. Conn syndrome
❌ Incorrect
Caused by excess aldosterone (primary hyperaldosteronism).
Only affects mineralocorticoids , not glucocorticoids.
5. Addison disease
✅ Correct
Also known as primary adrenal insufficiency .
The adrenal cortex is destroyed or fails , leading to low cortisol and low aldosterone .
Classic symptoms: fatigue, hypotension, hyperpigmentation, hyponatremia, hyperkalemia.
Affects both glucocorticoids and mineralocorticoids.
Final Answer:
✅ Addison disease
What condition leads to a global slowing of metabolism, causes fluid retention in tissues without pitting, and affects physical appearance like hair, skin, and facial muscles?
51 / 232
Tags:
2019
A middle-aged man presents to the outpatient department with mild hypothermia. A physical examination reveals goiter, sparse hair, ptosis, and non-pitting edema. What is the most likely diagnosis?
Step-by-Step Analysis:
Patient Presentation Includes:
Mild hypothermia: suggests reduced metabolic activity
Goiter: enlarged thyroid, seen in both hypo- and hyperthyroid conditions
Sparse hair: common in hypothyroidism
Ptosis: can be due to myxedema affecting muscles around the eyes
Non-pitting edema (especially of face and limbs): classic sign of myxedema
Understanding Each Option:
1. Acromegaly
Caused by excess growth hormone in adults
Features: coarse facial features, enlarged hands/feet, prognathism, deep voice
Does not cause hypothermia or non-pitting edema
Incorrect
2. Myxedema
Severe hypothyroidism in adults
Symptoms: fatigue, weight gain, cold intolerance, hypothermia, bradycardia, non-pitting edema , goiter , dry/sparse hair, ptosis , slow reflexes
Myxedema can also present with altered mental status in severe cases
Correct
3. Multinodular goiter
Structural condition with multiple thyroid nodules
May be euthyroid, hyperthyroid, or hypothyroid depending on function
Does not directly explain the constellation of systemic hypothyroid symptoms
Incorrect
4. Hyperthyroidism
Presents with heat intolerance , weight loss , tachycardia , hyperreflexia , warm skin , sweating
Opposite symptoms of what’s described
Incorrect
5. Cretinism
Congenital hypothyroidism in infants/children
Causes growth retardation, developmental delay, coarse facial features
Not applicable to a middle-aged man
Incorrect
Summary:
The most likely diagnosis is Myxedema (Option 2) — a severe form of adult hypothyroidism that explains all clinical signs given in the question.
When too much of a bone-degrading hormone is circulating, the skeleton suffers, the blood shows signs of mineral imbalance, and the kidneys may be left to deal with the aftermath. Consider the root cause behind the triad of bone pain, stones, and hypercalcemia.
52 / 232
Tags:
2019
A 30-year-old woman presents to the outpatient department with pain in the lower abdomen and has passed three stones in the urine. She also has high calcium and low phosphate levels in her blood. What is the diagnosis?
This case describes a young woman with:
This classic biochemical pattern points to primary hyperparathyroidism , and the bone manifestation of that condition is osteitis fibrosa cystica .
📌 What is Osteitis Fibrosa Cystica?
A skeletal disorder caused by excess parathyroid hormone (PTH) .
PTH:
Increases osteoclastic bone resorption → weak bones and bone pain.
Increases calcium reabsorption in kidneys and increases phosphate excretion → resulting in hypercalcemia and hypophosphatemia .
Increases activation of vitamin D , indirectly promoting calcium absorption from the gut.
Bones become weakened, and cystic lesions may appear due to excessive resorption → sometimes called “brown tumors” on imaging due to hemosiderin deposition.
Associated symptoms: bone pain, kidney stones, abdominal pain, psychiatric symptoms — remembered by the classic mnemonic:
🧠 “Stones, bones, groans, and psychiatric overtones”
🔍 Why the Other Options Are Incorrect:
Chronic renal failure : ❌ Typically causes secondary hyperparathyroidism , with low to normal calcium and high phosphate due to poor phosphate excretion. The biochemical pattern doesn’t fit .
Osteoporosis : ❌ This is a silent disease — no kidney stones or hypercalcemia; lab values are usually normal .
Osteogenesis imperfecta : ❌ A genetic collagen disorder leading to brittle bones, blue sclerae, and fractures, but no hypercalcemia or kidney stones .
Paget’s disease : ❌ Characterized by abnormal bone remodeling , can cause bone pain, deformities, and high ALP, but calcium and phosphate levels are usually normal .
When evaluating whether a gland is overactive, consider the brain’s first response to rising hormone levels—is the upstream signal turned off, or is the message still being sent?
53 / 232
Tags:
2019
A patient presented to the clinic with the symptoms of hyperthyroidism. Which of the following should be assessed to confirm the diagnosis?
🔍 Why TSH is the Best Initial Test:
Thyroid-Stimulating Hormone (TSH) is secreted by the anterior pituitary in response to TRH from the hypothalamus and is regulated by negative feedback from circulating levels of T3 and T4 .
TSH is extremely sensitive to small changes in thyroid hormone levels and is the first-line screening test in evaluating thyroid function.
In primary hyperthyroidism (where the thyroid itself is overactive), TSH is suppressed (low), often below the detectable range , due to negative feedback from elevated T3 and T4.
In secondary or tertiary hyperthyroidism (rare), TSH may be normal or high, so further evaluation is needed.
✅ Typical Pattern in Primary Hyperthyroidism:
↓ TSH
↑ Free T4
↑ Free T3
🔍 Why the Other Options Are Incorrect:
Thyroglobulin : ❌ This is a precursor protein stored in thyroid follicles. It’s not used to confirm hyperthyroidism . It’s more helpful in monitoring thyroid cancer recurrence or evaluating thyroiditis .
T4 : ❌ Total T4 includes protein-bound hormone, which may be influenced by thyroxine-binding globulin (TBG) levels. While elevated in hyperthyroidism, total T4 alone is less specific than TSH or free T4/T3.
Free T3 : ❌ While elevated in T3 toxicosis and some cases of hyperthyroidism, it may still be normal early in the disease. It’s usually ordered after TSH is found to be low .
TRH (Thyrotropin-Releasing Hormone) : ❌ Rarely used clinically. TRH stimulation tests were used historically to assess pituitary function but have largely been replaced by TSH measurements .
When inflammation flares in a hormone-secreting gland, ask yourself—what condition combines destruction, leakage, and discomfort all in one clinical picture?
54 / 232
Tags:
2019
A 40-year-old lady comes to the outpatient department with a painful swelling on the neck. She has high T3 and T4 and her TSH is diminished. The thyroid gland is tender and palpable. What is the most likely cause of her thyrotoxicosis?
Let’s decode this question using clinical clues and physiological reasoning .
🔬 Key Clues in the Question:
These clues point strongly toward subacute granulomatous thyroiditis , also called de Quervain thyroiditis .
📌 De Quervain (Subacute Granulomatous) Thyroiditis:
Often follows a viral upper respiratory infection .
The thyroid becomes inflamed , leading to destruction of follicular cells and release of preformed thyroid hormones → transient thyrotoxicosis .
Painful and tender thyroid is a hallmark (unlike most other causes of thyrotoxicosis).
Labs: ↑T3/T4, ↓TSH. May later progress to hypothyroidism.
Usually self-limited and resolves in weeks to months.
🔍 Why the Other Options Are Incorrect:
Hashimoto’s thyroiditis : ❌ Usually painless and leads to hypothyroidism , although there may be a transient thyrotoxic phase (“Hashitoxicosis”). However, tenderness is not typical , and it’s chronic , not acute.
Multinodular goiter : ❌ Typically causes a painless , enlarged thyroid with possible nodules. It may cause hyperthyroidism but is not associated with pain or tenderness .
Graves disease : ❌ The most common cause of thyrotoxicosis , but it presents with a diffusely enlarged, non-tender thyroid , along with exophthalmos and pretibial myxedema . No pain is present.
Iodine-induced hyperthyroidism : ❌ Seen in individuals with underlying thyroid disease exposed to excess iodine (e.g., contrast media or amiodarone). Presents as painless thyrotoxicosis .
When evaluating hormone-related disorders, ask: is the problem due to lack of hormone , or ineffective response to a hormone that is present? This distinction is key in endocrine pathology.
55 / 232
Tags:
2019
Decreased tissue response to the parathyroid hormone is seen in which of the following condition?
Pseudohypoparathyroidism is a rare genetic disorder in which the body produces normal or elevated levels of parathyroid hormone (PTH) , but the target tissues (like bone and kidney) are resistant to its effects. This is in contrast to true hypoparathyroidism, where the hormone itself is deficient.
🔬 Key Features of Pseudohypoparathyroidism:
PTH is elevated , but serum calcium remains low and phosphate is high .
Due to defects in the PTH receptor or downstream signaling pathways , particularly involving Gs alpha subunit mutations .
Often associated with Albright’s hereditary osteodystrophy , which includes:
❌ Why the Other Options Are Incorrect:
Congenital absence of parathyroid gland:
Primary hypoparathyroidism:
Secondary hypoparathyroidism:
Acquired hypoparathyroidism:
When evaluating a patient with episodic hypertension and sympathetic overactivity , ask: what lab test would best reflect excessive activity of the adrenal medulla ?
56 / 232
If you’re investigating a tumor that secretes stress hormones, think about how the body metabolizes catecholamines —what are their breakdown products, and where would you expect to find them elevated?
57 / 232
Consider which enzyme plays a central role in the synthesis of thyroid hormones and might be a key target when the immune system turns against the thyroid gland.
58 / 232
Tags:
2019
A 35-year-old lady is diagnosed with autoimmune hypothyroidism. Her physician is concerned about thyroid autoantibodies. Which of the following is the most appropriate antibody to detect autoimmune thyroid disease?
Autoimmune hypothyroidism, particularly Hashimoto’s thyroiditis , results from the immune system attacking the thyroid gland. The two most clinically relevant antibodies found in these patients are:
Anti-thyroid peroxidase (TPO) antibodies , also known as antiperoxidase antibodies
Anti-thyroglobulin antibodies (TgAb)
Among these, TPO antibodies are the most specific and sensitive for diagnosing autoimmune thyroid disease . These antibodies target the thyroid peroxidase enzyme , which is essential for thyroid hormone synthesis (it catalyzes the iodination of tyrosine residues in thyroglobulin).
TPO antibodies are detected in more than 90% of patients with Hashimoto’s thyroiditis and are used routinely in practice for confirmation.
❌ Why the Other Options Are Incorrect
Anticytoplasmic Associated with vasculitic diseases like granulomatosis with polyangiitis (c-ANCA, p-ANCA). Not relevant in thyroid autoimmunity.
Antifollicular Nonspecific; may occasionally be seen but not routinely used or reliable in diagnosing autoimmune thyroid disease.
Anti-membrane Vague and not a recognized test in the context of thyroid pathology. Sometimes seen in nephrology (anti-GBM), but not applicable here .
Antinuclear Used primarily to detect systemic autoimmune diseases like lupus. May be positive in some thyroid cases, but not diagnostic or specific for thyroid conditions.
When analyzing the cause of endocrine insufficiency, consider whether the damage is due to destruction , infiltration , infection , or congenital absence . Then ask: what mechanism would be most common in settings with modern healthcare systems and low infectious disease burden ?
59 / 232
Tags:
2019
What is the most common cause of Addison’s disease in the West?
Addison’s disease , or primary adrenal insufficiency , is a disorder where the adrenal cortex is destroyed or dysfunctional , leading to a deficiency of glucocorticoids, mineralocorticoids, and androgens . The causes can vary geographically, and it’s important to distinguish between them to understand the pathogenesis and prevalence of the disease.
🔬 Why Autoimmune Adrenalitis is Correct:
In Western countries , the most common cause is autoimmune destruction of the adrenal cortex, called autoimmune adrenalitis .
This is part of autoimmune polyglandular syndromes (APS type 1 and 2).
Autoantibodies target 21-hydroxylase and other adrenal enzymes.
The condition is insidious , often presenting with:
Fatigue , hypotension , hyperpigmentation , hyponatremia , and hyperkalemia
Diagnosis involves elevated ACTH with low cortisol , and the presence of adrenal autoantibodies .
❌ Why the Other Options Are Incorrect:
If the anterior pituitary starts making too much of one hormone—or pushing on the optic chiasm—it’s often because of this common benign growth.
60 / 232
Tags:
2019
Which of the following is the most common pituitary disease?
A pituitary adenoma is by far the most common pituitary disease , accounting for approximately 10–15% of all intracranial tumors . These are benign tumors of the anterior pituitary that may be:
Functioning (hormone-secreting) e.g., prolactinomas (most common subtype), somatotroph adenomas (GH), corticotroph adenomas (ACTH)
Non-functioning (silent) Often present due to mass effect (e.g., visual field defects via optic chiasm compression)
❌ Why the Other Options Are Incorrect:
Empty sella syndrome ❌ Can cause pituitary dysfunction but is less common than adenomas; may be idiopathic or secondary to pituitary infarct or surgery.
Sarcoidosis ❌ Can involve the hypothalamus/pituitary (neurosarcoidosis), but is rare as a primary pituitary pathology.
Pituitary metastatic tumors ❌ Rare; breast and lung cancers are most likely to metastasize to the pituitary, but it’s still uncommon overall .
Pituitary apoplexy ❌ Acute hemorrhage or infarction of a pituitary adenoma. It’s a complication of adenoma , not the most common primary disease itself.
When thinking about cortisol excess, remember: it turns your body into a sugar-making, fat-shifting, salt-holding machine — not one that runs out of glucose.
61 / 232
Tags:
2019
Which of the following is not a feature of Cushing syndrome?
Cushing syndrome refers to the clinical state of hypercortisolism , whether due to:
Exogenous steroids (most common)
Pituitary ACTH-secreting tumor (Cushing disease)
Adrenal tumor
Ectopic ACTH production (e.g., small-cell lung cancer)
Cortisol has powerful metabolic and electrolyte effects , leading to a classic pattern:
🔹 Features of Cushing Syndrome:
✅ Hypernatremia Cortisol has mineralocorticoid activity , increasing sodium reabsorption in the kidney → high serum sodium
✅ Hypokalemia Due to enhanced potassium excretion in distal tubules
✅ Increased plasma protein levels (mainly hepatic acute-phase proteins) Cortisol stimulates hepatic protein synthesis (e.g., fibrinogen, clotting factors)
✅ Increased mobilization of fat from lower body Cortisol causes central fat deposition (moon face, buffalo hump) but breakdown of peripheral/subcutaneous fat in limbs and buttocks
❌ Hypoglycemia WRONG — Cortisol raises blood glucose by:
Stimulating gluconeogenesis
Decreasing glucose uptake in tissues Therefore, hyperglycemia , not hypoglycemia, is seen in Cushing syndrome.
When a question involves multiple endocrine tumors or early-onset parathyroid disease, think about tumor suppressor genes that affect more than one endocrine organ.
62 / 232
Tags:
2019
Which of the following is correct regarding gene mutations responsible for parathyroid neoplasia?
Parathyroid tumors , especially parathyroid adenomas , are commonly seen in the context of Multiple Endocrine Neoplasia type 1 (MEN1) .
The MEN1 gene is a tumor suppressor gene located on chromosome 11q13
It encodes menin , a nuclear protein that regulates gene transcription
Loss-of-function mutations in MEN1 lead to uncontrolled cell growth and endocrine tumors , including:
So, MEN1 mutations are well-established in both familial and sporadic parathyroid tumors .
❌ Why the Other Options Are Incorrect:
GNAS mutations are responsible for familial parathyroid tumors ❌ GNAS mutations are classically seen in McCune-Albright syndrome and some pituitary adenomas , not parathyroid neoplasms .
The most important susceptibility locus is the HLA gene cluster (6p21) ❌ This refers to autoimmune diseases , not parathyroid adenomas.
The mutations causing parathyroid adenomas are rarely sporadic ❌ Actually, most parathyroid adenomas are sporadic , not familial. About 85–90% are sporadic .
Cyclin D1 gene deletion causes over-expression ❌ 🚫 This is incorrect at the mechanistic level : Cyclin D1 overexpression occurs due to a parathyroid-specific chromosomal inversion that places Cyclin D1 under control of the PTH gene promoter — not due to deletion .
When postpartum women present with lactation failure, fatigue, and hormonal symptoms — ask yourself if they had any major blood loss. The anterior pituitary is delicate, and hypoperfusion can silence it.
63 / 232
Tags:
2019
A 26-year-old woman presents to the clinic due to fatigue and difficulty in breastfeeding her 5-week-old baby. She also notices a slower growth of pubic hair, constipation, weight gain, and severe fatigue. Antenatal history is unremarkable, however, she has a history of severe bleeding with blood transfusion right after her delivery. What is the diagnosis of this patient?
🔍 Clue 1: Fatigue, weight gain, constipation
These are signs of hypothyroidism — possibly secondary (due to pituitary failure).
🔍 Clue 2: Difficulty breastfeeding & pubic hair thinning
Suggests low prolactin and low gonadotropins (FSH/LH), indicating anterior pituitary failure .
🔍 Clue 3: Severe postpartum hemorrhage (PPH)
This is the key clue . In Sheehan’s syndrome , severe blood loss during delivery causes ischemic necrosis of the anterior pituitary .
💡 The anterior pituitary controls:
So, this woman is showing panhypopituitarism after postpartum hemorrhage — classic Sheehan’s syndrome .
❌ Why the Other Options Are Incorrect:
Thyroid dysfunction ❌ May explain fatigue, weight gain, and constipation — but not lactation failure or pubic hair loss . Also doesn’t tie into the PPH history.
Prolactinoma ❌ Causes excess prolactin (leading to galactorrhea , not failure of lactation), and usually suppresses gonadotropins , but isn’t tied to PPH.
MEN syndrome ❌ Multiple endocrine neoplasia involves tumors in endocrine glands (e.g., parathyroid, pancreas, pituitary), but not postpartum necrosis or this pattern of deficiency.
Ineffective latching ❌ Might cause breastfeeding trouble, but won’t cause pubic hair thinning, fatigue, or constipation .
When a hormone deficiency shows up abruptly in a young adult, think about what could have physically removed or damaged the gland—especially in areas close to common surgical sites.
64 / 232
Tags:
2019
A 20-year-old man was brought to the emergency room presenting with seizures, paresthesia, cramps, and tetany. After managing his symptoms, his blood calcium levels were investigated and were found to be lower than normal. The doctor explained to the patient’s attendant that he had a disease that was caused by the deficiency of a specific hormone. What is the most common cause of this condition?
The symptoms — seizures, paresthesia, cramps, and tetany — are all hallmarks of hypocalcemia . Low calcium levels in a young adult, especially with neuromuscular irritability (tetany), point to hypoparathyroidism .
🦴 Parathyroid hormone (PTH) is responsible for:
Increasing serum calcium (via bone resorption, kidney reabsorption, and activating vitamin D)
Decreasing serum phosphate
When PTH is deficient , serum calcium drops, causing the symptoms described.
So the question becomes: What is the most common cause of hypoparathyroidism?
✅ Surgical removal or damage to the parathyroid glands during neck surgeries , especially thyroidectomy , is by far the most common cause — especially in adults .
Even if surgery was done years ago, hypoparathyroidism can persist.
❌ Why the Other Options Are Incorrect:
Congenital ❌ Rare. Seen in conditions like DiGeorge syndrome (22q11 deletion) — more common in infants , not a typical cause in a 20-year-old.
Neoplasia ❌ Tumors of the parathyroid are more likely to cause hyperparathyroidism , not deficiency.
Autoimmune ❌ Can occur (especially in Autoimmune Polyendocrine Syndrome type 1 ) but less common than postsurgical causes .
Idiopathic ❌ Means cause is unknown. Always a diagnosis of exclusion , and less common than surgical causes .
When identifying the most common type of a malignancy, consider those associated with good prognosis, lymph node spread, and classic histologic features—not necessarily the ones that sound the most dangerous.
65 / 232
Tags:
2019
Which of the following is the most common neoplasm of the thyroid?
Papillary carcinoma is the most common thyroid malignancy , accounting for ~80% of all thyroid cancers. It usually affects:
Key features :
❌ Why the Other Options Are Incorrect:
Follicular carcinoma ❌ Second most common, but less common than papillary . More likely to spread hematogenously (bloodstream) to lungs/bones, unlike papillary which spreads via lymphatics .
Medullary carcinoma ❌ Arises from parafollicular C cells (not follicular cells), secretes calcitonin . Associated with MEN 2A/2B syndromes. Rare—only ~5%.
Anaplastic (undifferentiated) carcinoma ❌ Very aggressive, poor prognosis. Occurs in older patients, least common type. Often presents with rapidly enlarging neck mass and airway compromise.
Adenoma ❌ Refers to a benign tumor , often follicular adenoma . Not a malignancy—does not invade or metastasize.
Consider which electrolyte is critical for generating action potentials in muscle fibers. When it drops, muscles may still look fine structurally — but functionally, they’re too “quiet” to contract.
66 / 232
Tags:
2018
Which of the following causes lead to muscle weakness in Conn syndrome?
Let’s examine what Conn syndrome is, understand its mechanism , and explore why hypokalemia is the correct answer — and why the others are not the primary cause of muscle weakness in this condition.
🔬 What is Conn Syndrome?
Conn syndrome (also known as primary hyperaldosteronism ) is a condition caused by excess secretion of aldosterone , typically from an adrenal adenoma.
Aldosterone is a hormone produced by the zona glomerulosa of the adrenal cortex.
Its main actions are:
Sodium retention
Potassium excretion
Hydrogen ion excretion
⚙️ How Does It Cause Muscle Weakness?
Excess aldosterone → ↑ Na⁺ retention & ↑ K⁺ loss →Hypokalemia →Muscle weakness, fatigue, and even paralysis
Potassium is essential for normal muscle contraction and nerve conduction . When potassium levels drop too low:
The resting membrane potential becomes more negative.
Muscles and neurons become less excitable .
This results in muscle weakness , cramps, and sometimes cardiac arrhythmias.
❌ Why the Other Options Are Incorrect:
Option
Why It’s Incorrect
Hypertension
Common in Conn syndrome due to sodium retention, but it doesn’t cause muscle weakness .
Hypercalcemia
Causes weakness in other disorders (like hyperparathyroidism), not in Conn syndrome .
Hypernatremia
Sodium may be elevated mildly, but not enough to cause direct muscle weakness in Conn syndrome.
Neuropathy
Not a feature of Conn syndrome; muscle weakness here is due to electrolyte disturbance , not nerve damage.
Consider the effect of a hormone that mobilizes calcium from bone and reduces phosphate. What would happen to muscles and nerves if calcium levels are excessively high rather than low?
67 / 232
Tags:
2018
Which of the following is not associated with hyperparathyroidism?
This question tests your understanding of parathyroid hormone (PTH) physiology and the clinical features of hyperparathyroidism .
🔬 What is Hyperparathyroidism?
Hyperparathyroidism is a condition characterized by excess secretion of parathyroid hormone (PTH) from the parathyroid glands. PTH plays a crucial role in calcium and phosphate balance by acting on the bones, kidneys, and intestines .
📈 What Does PTH Do?
Target
Effect of PTH
Bone
Stimulates osteoclasts → bone resorption → ↑ serum calcium
Kidney
Increases calcium reabsorption, phosphate excretion
Intestine
Indirectly increases calcium absorption via activation of Vitamin D
✅ Let’s Analyze Each Option:
❌ Tetany → NOT associated with hyperparathyroidism
Tetany is a neuromuscular irritability caused by low serum calcium .
It is a classic feature of hypoparathyroidism , not hyperparathyroidism .
Symptoms: muscle cramps, Chvostek’s sign, Trousseau’s sign, etc.
Since hyperparathyroidism raises calcium levels, tetany does not occur.
✔️ Correct answer!
✅ Calcium present in the urine
Despite high PTH increasing calcium reabsorption in the kidneys, the overload of calcium in blood leads to hypercalciuria .
Patients can develop renal stones (nephrolithiasis) due to calcium excretion.
✅ Osteitis fibrosa cystica
A bone complication of severe hyperparathyroidism.
High PTH → Excess bone resorption → Fibrous tissue and cystic bone lesions.
Also called “brown tumors ” (not true neoplasms).
✅ Hypophosphatemia
✅ Hypercalcemia
The hallmark of primary hyperparathyroidism.
“Bones, stones, groans, and psychic overtones” — refers to skeletal pain, kidney stones, abdominal pain, and neuropsychiatric symptoms due to high calcium.
In a condition where the body’s salt and stress hormones are both missing, would you expect energy, pressure, and balance to rise or fall? Think about what happens when both the regulators of sugar and sodium are absent.
68 / 232
Tags:
2018
Which of the following is seen in Addison’s disease?
🧠 What is Addison’s Disease?
Addison’s disease is primary adrenal insufficiency , meaning the adrenal cortex itself is damaged and cannot produce its hormones properly.
The adrenal cortex normally produces:
Aldosterone (zona glomerulosa)
Cortisol (zona fasciculata)
Androgens (zona reticularis)
In Addison’s disease, all three may be decreased, but cortisol and aldosterone are the most clinically significant.
🔬 Pathophysiology Recap:
Cortisol deficiency → fatigue, hypoglycemia, weight loss, hypotension
Aldosterone deficiency → hyponatremia, hyperkalemia, dehydration, hypotension
ACTH levels increase due to lack of negative feedback → leads to hyperpigmentation
✅ Correct Option: Decrease in both aldosterone and cortisol
This is the hallmark of Addison’s disease .
Both hormones are made in the adrenal cortex.
Since Addison’s is primary failure of the adrenal glands, both are reduced .
This explains the salt-wasting , low blood pressure , and fatigue seen in these patients.
❌ Why the Other Options Are Incorrect:
❌ Decrease in cortisol (only)
While cortisol is decreased, aldosterone is also usually decreased in primary adrenal insufficiency.
This option is incomplete .
❌ Increase in cortisol
The opposite is true.
Addison’s = cortisol deficiency , not excess.
Increased cortisol is seen in Cushing’s syndrome , not Addison’s.
❌ Hypertension
Addison’s causes hypotension , not hypertension.
Due to low aldosterone , the kidneys lose sodium and water → blood volume drops → low blood pressure
❌ None of them
When the fight-or-flight system goes into overdrive, which hormone floods the bloodstream, causing the heart to race, pressure to rise, and sweat to pour—courtesy of a small but mighty center of stress deep inside the gland?
70 / 232
Tags:
2018
Pheochromocytoma is a neoplasm that secretes which of the following?
Pheochromocytoma is a catecholamine-secreting tumor that arises from chromaffin cells of the adrenal medulla .
These chromaffin cells are part of the sympathetic nervous system and secrete catecholamines , mainly:
📌 Therefore, the hallmark of pheochromocytoma is excessive secretion of catecholamines , especially adrenaline .
⚠️ Clinical Features of Pheochromocytoma:
Due to excess catecholamines, patients may experience:
Paroxysmal hypertension
Palpitations
Sweating
Tremors
Anxiety or panic attacks
🩸 Diagnostic tests include elevated plasma metanephrines , urinary VMA (vanillylmandelic acid) , or plasma catecholamines .
❌ Why the Other Options Are Incorrect:
❌ Cortisol
Secreted by the adrenal cortex (zona fasciculata)
Not a product of the adrenal medulla
Associated with Cushing syndrome , not pheochromocytoma
❌ Aldosterone
Secreted by the zona glomerulosa of the adrenal cortex
Involved in sodium and water retention (e.g., in Conn’s syndrome )
Not secreted in pheochromocytoma
❌ Sex hormones
❌ All of them
Only adrenaline (catecholamines) are secreted by pheochromocytoma.
Others are secreted by different zones of the adrenal cortex , not the medulla .
Therefore, this is incorrect .
Consider which part of the chain fails to react even when properly signaled from above. The issue isn’t in the command or the target—it lies in the middle link of the chain.
71 / 232
Tags:
2018
In which of the following conditions thyroid-stimulating hormone (TSH) is not responsive to thyroid-releasing hormone (TRH)?
To answer this question, you need a solid understanding of how the hypothalamic-pituitary-thyroid (HPT) axis works:
🔁 Normal HPT Axis:
Hypothalamus secretes TRH (Thyrotropin-releasing hormone)
TRH stimulates the anterior pituitary to secrete TSH
TSH acts on the thyroid gland to produce T3 and T4
T3 and T4 then negatively feedback to the hypothalamus and pituitary to regulate the system.
🧪 TRH Stimulation Test:
Used to determine the cause of hypothyroidism , especially in central (secondary or tertiary) cases:
If TSH increases after TRH → pituitary is responsive → problem is in the hypothalamus (tertiary)
If TSH does not increase after TRH → pituitary is non-functional → problem is in the pituitary (secondary)
⚠️ Secondary Hypothyroidism:
Cause: Pituitary dysfunction
TSH is low or inappropriately normal , and it does not increase in response to TRH .
This is because the pituitary is damaged or non-responsive .
✅ Therefore, in secondary hypothyroidism, TSH is not responsive to TRH.
❌ Why the Other Options Are Incorrect:
❌ Tertiary hypothyroidism
Problem is in the hypothalamus → ↓ TRH
But the pituitary is intact , so if you administer TRH , TSH increases .
TSH is responsive to TRH → Incorrect.
❌ Primary hypothyroidism
Problem is in the thyroid gland , so TSH is high (loss of negative feedback).
TRH can still increase TSH levels.
TSH is responsive → Incorrect.
❌ Hyperparathyroidism
Unrelated to thyroid axis
It affects calcium and PTH , not TSH or TRH.
TSH responsiveness is normal → Incorrect.
❌ None of them
When certain minerals are excessively filtered or reabsorbed abnormally, the body’s filter system may become clogged with crystals. Think of where these mineral build-ups would most logically occur.
72 / 232
Tags:
2018
What is nephrocalcinosis?
Nephrocalcinosis refers to diffuse, abnormal deposition of calcium salts in the renal parenchyma , particularly in the tubules or interstitium of the kidney.
This is not the same as kidney stones (nephrolithiasis), which are discrete stone formations. Nephrocalcinosis is microscopic to macroscopic calcification of the kidney tissue itself.
📈 Causes:
It is commonly associated with conditions that raise calcium levels in the blood or urine , such as:
🔍 Clinical Consequences:
May be asymptomatic or lead to progressive kidney dysfunction .
Detected by imaging : e.g., ultrasound or CT shows increased echogenicity or density.
❌ Why the Other Options Are Incorrect:
❌ Calcification of lower respiratory tract
❌ Lipid deposits in renal structure
This describes lipoid nephrosis (a.k.a. minimal change disease), not nephrocalcinosis.
Nephrocalcinosis involves calcium , not lipids.
❌ Calcification of myocardium
❌ Lyonisation of DNA
Consider what happens when the body’s natural “height regulator” is turned up to maximum during a period when bones are still open to growth. What would the result look like years later if left unchecked?
73 / 232
Tags:
2018
An excess of growth hormone in childhood leads to what pathology?
Growth hormone is secreted by the anterior pituitary gland .
It acts indirectly through insulin-like growth factor 1 (IGF-1) to stimulate growth of bones and tissues.
The effect of GH depends on the timing of its excess—before or after epiphyseal plate closure (which occurs after puberty).
👶 In Childhood: Gigantism
In children and adolescents before epiphyseal plate closure , excess GH leads to linear bone growth .
Result: Abnormally tall stature , proportionate enlargement of body parts.
This condition is called Gigantism .
It’s usually caused by a pituitary adenoma producing excess GH.
🧔 In Adulthood: Acromegaly
After epiphyseal plates have fused, GH cannot cause height increase.
Instead, it leads to abnormal growth of soft tissues and bones of the face, hands, feet , etc.
This is called Acromegaly .
❌ Why the Other Options Are Incorrect:
❌ Acromegaly
❌ Retinal detachment
❌ Myxedema coma
This is a severe form of hypothyroidism , usually in elderly patients .
It involves low thyroid hormones , not GH.
❌ Cushing disease
This results from excess ACTH , usually from a pituitary tumor , leading to cortisol excess , not GH.
Features: central obesity, moon face, striae — not gigantism .
In normal physiology, glucose suppresses growth hormone. Which test checks if this suppression fails?
74 / 232
Tags:
2019
Which of the following is the confirmatory test for acromegaly?
The oral glucose tolerance test (OGTT) is the confirmatory test for acromegaly . Here’s how and why:
In a normal person , oral glucose suppresses growth hormone (GH) secretion.
In acromegaly , GH fails to suppress (or may even increase) after glucose intake.
Therefore, a lack of suppression of GH during OGTT confirms the diagnosis.
🔬 Role of Other Options:
IGF-1 levels ✅ Elevated IGF-1 is screening for acromegaly (good first step), but not confirmatory .
Growth hormone levels ❌ GH is secreted pulsatilely , so random levels are unreliable .
Somatostatin levels ❌ Not routinely measured for diagnosis.
Insulin levels ❌ Not directly relevant to acromegaly diagnosis.
📝 Summary:
When evaluating endocrine insufficiency, consider not just the hormone levels at rest, but how the gland responds to stimulation. Ask yourself: How can we test the functional reserve of the adrenal glands?
75 / 232
Tags:
2019
Which of the following is the confirmation test for Addison’s disease?
Addison’s disease , or primary adrenal insufficiency , is a condition where the adrenal cortex fails to produce adequate levels of cortisol (and often aldosterone).
To confirm the diagnosis, we use the ACTH stimulation test (also called the Synacthen test) . Here’s how it works:
A synthetic form of ACTH is administered.
In a healthy person , this causes the adrenal glands to produce a significant rise in cortisol .
In Addison’s disease , the adrenal glands are damaged and cannot respond properly, so cortisol levels remain low even after stimulation.
This test is critical because basal hormone levels alone (like cortisol or ACTH) can sometimes be inconclusive due to diurnal variation or stress.
❌ Incorrect Answer Breakdown:
Aldosterone levels ✘ Can be affected in Addison’s disease, but they are not sufficient alone to confirm the diagnosis.
Cortisol levels ✘ May be low in Addison’s disease, but low levels alone are not definitive — they can be low in secondary causes or due to other factors (e.g., time of day, stress).
Adrenocorticotropic hormone levels ✘ High ACTH is typical in primary Addison’s disease, but levels can overlap and don’t test the gland’s functional capacity .
Glucose levels ✘ Hypoglycemia may occur in Addison’s, but it is a non-specific finding and not a diagnostic test.
Think about what essential micronutrient might be lacking in remote, high-altitude regions and how the body’s compensatory mechanisms respond when thyroid hormone synthesis is impaired by that deficiency.
76 / 232
Tags:
2019
Which of the following is the most common disease of the thyroid in mountainous areas?
Endemic goiter is the most common thyroid disorder seen in mountainous areas , particularly due to iodine deficiency in the local diet.
Why It Happens:
Iodine is a key component of thyroid hormones (T3 and T4).
In iodine-deficient regions , such as mountainous areas where the soil and water may lack iodine, the thyroid enlarges (goiter formation) to try to capture more iodine and produce sufficient thyroid hormones.
Key Characteristics:
Occurs in populations , not just individuals (hence “endemic”)
Often associated with dietary insufficiency , especially in non-iodized salt–consuming regions
Can lead to hypothyroidism or impaired cognitive development (e.g., cretinism in severe cases)
Incorrect Answer Explanations:
Hashimoto’s thyroiditis
Incorrect
An autoimmune disease , not associated with geography or iodine deficiency.
More common in developed regions , not specifically mountainous areas.
Iatrogenic goiter
Incorrect
Refers to goiter caused by medical treatment (e.g., excessive iodine, drugs).
Not naturally occurring or associated with geography.
Myxedema
Incorrect
Refers to severe hypothyroidism (often in adults), usually autoimmune or iatrogenic .
It is a clinical manifestation , not a disease specific to iodine-deficient regions.
Graves disease
Incorrect
An autoimmune hyperthyroid condition .
Not linked to iodine deficiency or mountainous areas.
More common in iodine-sufficient populations .
When evaluating adrenal insufficiency, consider the dynamic test that directly challenges the adrenal glands’ ability to respond to a hormone signal from the pituitary, rather than relying on static hormone measurements alone.
77 / 232
Tags:
2019
Which of the following is a useful investigation for diagnosing Addison’s disease?
The ACTH (cosyntropin) stimulation test is the most useful and definitive test for diagnosing Addison’s disease (primary adrenal insufficiency).
How It Works:
Interpretation:
In normal individuals , cortisol levels rise in response to ACTH.
In Addison’s disease , the adrenal glands are damaged and cannot produce cortisol , so there’s no or minimal increase in cortisol levels.
Incorrect Answer Explanations:
Serum cortisol
Incorrect
While helpful as a screening test , a low baseline cortisol is not specific.
It does not differentiate between primary and secondary adrenal insufficiency .
Can be affected by stress, time of day , and other variables.
Serum electrolytes
Incorrect
May show hyponatremia , hyperkalemia , or hypoglycemia in Addison’s, but these are non-specific findings .
Cannot confirm the diagnosis.
Plasma ACTH level
Serum aldosterone
Incorrect
Often low in Addison’s due to zona glomerulosa dysfunction.
Not diagnostic on its own, and aldosterone levels may also vary in secondary adrenal insufficiency or with medications.
Consider the autoimmune condition that leads to gradual destruction of the thyroid gland, especially in middle-aged women, and is associated with antibodies against thyroid tissue.
78 / 232
Tags:
2019
Which of the following is the most common cause of hypothyroidism?
Hashimoto thyroiditis (also known as chronic lymphocytic thyroiditis ) is the most common cause of hypothyroidism in iodine-sufficient regions such as the U.S., Europe, and most developed countries.
Key Features:
Autoimmune destruction of the thyroid gland
Associated with anti-thyroid peroxidase (anti-TPO) and anti-thyroglobulin antibodies
Common in middle-aged women
Can initially present with transient hyperthyroidism due to follicular rupture, followed by long-term hypothyroidism
Histology shows Hurthle cells , lymphocytic infiltration, and germinal centers
Incorrect Answer Explanations:
Multinodular goiter
Follicular adenoma
Incorrect
A benign thyroid tumor , typically presents as a painless thyroid nodule.
It is non-functional in most cases and does not lead to hypothyroidism .
Graves disease
Incorrect
This is an autoimmune cause of hyperthyroidism , not hypothyroidism.
It involves TSH receptor-stimulating antibodies , increasing T3 and T4 levels.
None of these
In a state where the metabolic engine has slowed down, think about which byproducts might begin to accumulate in the bloodstream because they’re not being broken down or cleared efficiently.
79 / 232
Consider what essential element is required for the synthesis of thyroid hormones—and what would happen if a developing fetus doesn’t get enough of it from the mother during pregnancy.
80 / 232
Tags:
2019
Which of the following is the most common cause of congenital hypothyroidism worldwide?
Iodine deficiency is the most common cause of congenital hypothyroidism worldwide , especially in developing regions where dietary iodine intake is insufficient.
Why is iodine important?
Iodine is an essential element required for the synthesis of T3 (triiodothyronine) and T4 (thyroxine) .
During fetal development, maternal thyroid hormone (dependent on iodine availability) is critical for brain development and growth .
Deficiency leads to cretinism , characterized by:
Public Health Implication:
Incorrect Answer Explanations:
Hashimoto’s thyroiditis
Autoimmune destruction
Incorrect
Similar to Hashimoto’s, this may contribute in rare neonatal cases (e.g., transplacental antibodies), but is not the most prevalent global cause .
Cellular atrophy
Hypercortisolism
Think about which pituitary adenoma most commonly presents with hormonal symptoms like menstrual irregularities, galactorrhea, or infertility—and is often discovered early due to its noticeable endocrine effects rather than mass effect
81 / 232
Tags:
2019
Which of the following is the most hyperfunctioning pituitary tumor?
Prolactinoma is the most common and hyperfunctioning pituitary adenoma . It arises from lactotrophs (mammotropes) of the anterior pituitary and causes excessive secretion of prolactin .
Clinical Features:
Women: Amenorrhea, galactorrhea, infertility
Men: Decreased libido, impotence, gynecomastia
May cause mass effect symptoms if large (e.g., headaches, visual field defects)
Diagnosis:
Treatment:
Dopamine agonists (e.g., cabergoline, bromocriptine)
Surgery if medication fails
Incorrect Answer Explanations:
Growth cell adenoma (Somatotroph adenoma)
ACTH producing adenoma (Corticotroph adenoma)
FSH and LH producing adenoma (Gonadotroph adenoma)
None of these
Sometimes, weight loss isn’t a matter of diet—it’s a symptom of the body’s engine running in overdrive. Think: what endocrine organ, when overly active, speeds up nearly every system in the body?
82 / 232
Tags:
2018
A woman presented to the clinic with the complaint of weight loss despite having a good appetite, increased sweating, and oligomenorrhea. Her laboratory results show increased thyroid hormone levels in the body. Which of the following is the most likely cause for the woman’s symptoms?
🧠 Step 1: Interpret the Symptoms
The patient presents with:
Symptom
Clinical Significance
Weight loss with good appetite
Suggests increased metabolism → classic of hyperthyroidism
Increased sweating
Due to increased heat production (from high metabolic rate)
Oligomenorrhea
Seen in many endocrine disorders , but commonly associated with hyperthyroidism
Lab finding: Elevated thyroid hormones
Confirms the physiological diagnosis of hyperthyroidism
These features all point toward a hypermetabolic state , which is characteristic of hyperthyroidism .
🔍 Option-by-Option Breakdown
❌ Hypothyroidism
Opposite of what we see here.
Features: weight gain , cold intolerance , fatigue , constipation , menorrhagia .
Labs: ↓ T3/T4 , ↑ TSH (in primary hypothyroidism).
🚫 Not consistent with the case .
✅ Hyperthyroidism
Classic symptoms include:
Weight loss with increased appetite
Heat intolerance / sweating
Tachycardia, anxiety, tremors
Oligomenorrhea
Labs: ↑ T3/T4 , usually ↓ TSH (unless pituitary in origin).
✅ Perfect match with clinical presentation and labs.
❌ None of them
❌ Hashimoto thyroiditis
An autoimmune hypothyroidism (most common in developed countries).
Initial phase can cause transient hyperthyroidism (Hashitoxicosis), but typically progresses to hypothyroidism .
Patients usually present with weight gain, cold intolerance, dry skin .
Labs: ↑ TSH , ↓ T4 , positive anti-TPO antibodies .
🚫 Not consistent with sustained hyperthyroid symptoms.
❌ Hyperparathyroidism
Affects calcium metabolism , not directly related to thyroid hormones.
Features include: hypercalcemia , bone pain, kidney stones, abdominal pain, psychiatric symptoms .
Does not cause weight loss, sweating, or altered thyroid hormone levels.
🚫 Not relevant to this presentation.
🧠 Summary Table
Option
Consistent with Symptoms?
Hypothyroidism
❌ No — opposite symptoms
Hyperthyroidism
✅ Yes — fits all features
None of them
❌ No — hyperthyroidism fits
Hashimoto thyroiditis
❌ No — typically causes hypothyroidism
Hyperparathyroidism
❌ No — unrelated to thyroid hormones
When the body’s engine runs too hot, it shows certain signs. But if someone is always reaching for a sweater instead of fanning themselves, think carefully about whether the engine is truly overheating—or barely idling.
83 / 232
Tags:
2018
Which of the following is not a symptom of hyperthyroidism?
Hyperthyroidism is a condition where the thyroid gland produces too much thyroid hormone (T3 and T4) . This leads to a general increase in metabolic rate and affects multiple organ systems.
🔥 Key Clinical Features of Hyperthyroidism:
System Affected
Clinical Features
Metabolic
Heat intolerance, weight loss despite increased appetite
Cardiovascular
Tachycardia, palpitations, possibly atrial fibrillation
Neuromuscular
Fine tremors, hyperreflexia, muscle weakness
GI
Increased bowel movements or diarrhea
Skin
Warm, moist skin
Reproductive
Oligomenorrhea, decreased fertility
Psychological
Anxiety, restlessness, insomnia
🌡️ Let’s Analyze Each Option:
✅ Increase in body temperature
✅ Fine tremors
✅ Tachycardia
❌ Cold intolerance
This is a classic feature of hypothyroidism , not hyperthyroidism.
In hypothyroidism, metabolic rate is decreased , leading to reduced heat production, making the patient feel cold.
✅ Correct answer → this is NOT a symptom of hyperthyroidism.
❌ None of them
🧠 Summary Table:
Option
Hyperthyroidism Symptom?
Increase in temperature
✅ Yes
Fine tremors
✅ Yes
Tachycardia
✅ Yes
Cold intolerance
❌ No — seen in hypothyroidism
None of them
❌ No — there is one incorrect item
Some diseases arise not from outside attacks like infections or tumors, but from within—when the body’s own defense system turns against itself. Consider which causes might be region-dependent versus globally consistent.
84 / 232
Tags:
2018
What is the most common cause of Addison’s disease in the West?
🩺 What is Addison’s Disease?
Addison’s disease, also known as primary adrenal insufficiency , occurs when the adrenal cortex is damaged and fails to produce adequate cortisol (and often aldosterone).
Key hormones affected:
Cortisol → glucocorticoid (stress response, glucose metabolism)
Aldosterone → mineralocorticoid (salt balance)
Androgens → minor in adults (especially females)
🧪 Causes of Addison’s Disease
The causes vary depending on geographic location and socioeconomic conditions .
📍 In Western countries (like the US, UK, Europe):
Autoimmune adrenalitis is the most common cause .
It’s often part of Autoimmune Polyendocrine Syndrome (APS) .
The immune system attacks the adrenal cortex , leading to gradual destruction.
Antibodies to 21-hydroxylase enzyme are often present.
🌍 In developing countries :
Tuberculosis is a more common cause , especially in regions with high TB prevalence.
TB can infect and destroy adrenal glands .
This is less common now in high-income countries due to better TB control.
🔍 Option-by-Option Breakdown
Option
Why It’s Incorrect or Correct
Infection
❌ Too vague. Could mean TB, fungal, etc., but not the most common in the West.
Autoimmune adrenalitis
✅ Correct. Most common in the West due to prevalence of autoimmune diseases.
Tumor of adrenal gland
❌ Rare cause. Tumors can affect adrenal function but are not a common cause of Addison’s.
Tuberculosis
❌ More common in developing countries , not in the West.
Congenital absence of adrenal gland
❌ Extremely rare, and presents in neonates , not adults. Not a common cause anywhere.
Consider the inverse relationship between differentiation and proliferation.
85 / 232
Tags:
2018
Which of these is the least frequent thyroid tumor?
Thyroid tumors are broadly categorized into differentiated, medullary, and undifferentiated (anaplastic) types, with differentiated thyroid cancers being the most common. The incidence and characteristics of each type vary significantly.
Papillary Thyroid Carcinoma (PTC) :
Conclusion
Based on the prevalence rates, Anaplastic thyroid carcinoma is definitively the least frequent primary thyroid tumor among the choices provided.
GH is shouting, but the body isn’t listening
86 / 232
Tags:
2024
A young girl presents with short stature and other growth-related symptoms. Her growth hormone (GH) levels are normal , but IGF-1 levels are low , and a mutation affecting GH signaling is suspected. What is the most likely diagnosis ?
Lorain Dwarfism is a form of Laron Syndrome , a type of GH insensitivity caused by:
A mutation in the GH receptor
Normal or elevated GH levels ✅
Low IGF-1 levels ✅ (GH can’t stimulate its production)
Key clinical features:
❌ Why the Other Option Is Incorrect:
Cretinism ❌ Due to congenital hypothyroidism ❌ Features: mental retardation, coarse facial features, macroglossia, umbilical hernia ❌ Not related to GH or IGF-1 pathways
Bones are soft, labs are off — it’s adult rickets
87 / 232
Tags:
2024
A woman presents with muscle weakness , bone softening , and an increased frequency of fractures . Her lab investigations reveal:
What is the most likely diagnosis ?
Osteomalacia is the adult counterpart of rickets (which occurs in children). It’s caused by defective mineralization of osteoid due to vitamin D deficiency or phosphate metabolism disorders.
Typical lab findings in osteomalacia:
Clinical signs:
❌ Why the Other Options Are Incorrect:
Osteoporosis ❌ Normal calcium, phosphate, and ALP levels. It’s due to decreased bone mass , not defective mineralization.
Paget’s disease ❌ Typically has elevated ALP , but normal calcium and phosphate . Bones are thickened, not softened.
Primary hyperthyroidism ❌ Doesn’t fit lab findings. Might cause hypercalcemia , not low calcium .
Rickets ❌ Same pathology as osteomalacia but occurs in children , not adults.
If a pituitary tumor is causing hormone excess that leads to symptoms of cortisol overload, what hormone must it be producing too much of?
88 / 232
Tags:
2020
Which of the following involves excessive secretion of adrenocorticotropic hormone (ACTH)?
Cushing disease is a specific condition where there is excessive secretion of ACTH from the pituitary gland — usually due to a pituitary adenoma (benign tumor) .
Here’s what happens:
Excess ACTH stimulates the adrenal cortex
This causes overproduction of cortisol
The result is Cushing syndrome (the collection of signs and symptoms due to high cortisol), but since it’s caused by high ACTH from the pituitary , it’s specifically called Cushing disease
🧬 Key features include:
Moon face
Central obesity
Hypertension
Muscle weakness
Skin thinning and striae
❌ Why the Other Options Are Wrong (Simple):
Addison disease ❌ This is primary adrenal insufficiency , where the adrenal glands fail.
ACTH may be high (due to low cortisol), but it’s a response , not the cause.
The issue is not ACTH overproduction , but adrenal failure .
Thyrotoxicosis ❌ Involves excess thyroid hormone (T₃/T₄) — not related to ACTH at all.
Myxedema ❌ This is severe hypothyroidism , involving low thyroid hormones , again unrelated to ACTH.
Pheochromocytoma ❌ A tumor of the adrenal medulla , producing excess catecholamines (adrenaline and noradrenaline) — ACTH has no role here.
If the body cannot make enough cortisol even though the brain is shouting for it by increasing ACTH, where must the real failure be?
89 / 232
Tags:
2020
Which of the following leads to Addison disease?
Addison disease is caused by the failure of the adrenal cortex itself — meaning the adrenal glands cannot produce enough cortisol , and often aldosterone too.
This is called primary adrenal insufficiency , or primary adrenocortical hypofunction .
Common causes include:
Autoimmune destruction (most common in developed countries)
Tuberculosis (common in developing countries)
Infections, cancer, or adrenal hemorrhage
The key point is: 👉 The problem starts in the adrenal gland itself.
Because the adrenal cortex fails:
Cortisol and aldosterone levels drop
ACTH levels rise (trying to stimulate the failing gland)
This leads to symptoms like:
❌ Why the Other Options Are Wrong (Simple):
Secondary adrenal hypofunction ❌ This is due to pituitary failure (low ACTH), so it’s not Addison disease — Addison is primary .
Tertiary disorder of the cortisol hormone system ❌ Tertiary refers to hypothalamic failure (low CRH → low ACTH → low cortisol). Again, this is not primary , so not Addison .
Abnormal growth of the thyroid gland ❌ That relates to thyroid disease , not the adrenal glands.
Functional disability of the hormone cortisol ❌ Cortisol dysfunction might be a result , but Addison disease refers to adrenal gland failure , not cortisol resistan
When thinking about diseases of a small but critical endocrine gland, which condition—often slow-growing and usually benign—is the one you’re most likely to find on imaging or hormonal workup?
90 / 232
When a woman develops hormone deficiency symptoms shortly after a complicated delivery with severe blood loss , ask yourself: could a critical gland have been damaged due to lack of blood?
91 / 232
Tags:
2020
A lady arrives at the clinic for the delivery of her baby. After the delivery, the lady has significant bleeding and exhibits post-partum symptoms of poor lactation, loss of pubic hair, and fatigue. Which of the following describes the condition the patient is suffering from?
Let’s look at the key clues in this woman’s case:
All of these suggest that the woman is suffering from a condition where her pituitary gland has stopped working properly after childbirth.
🩸 What happens in Sheehan syndrome ?
During pregnancy, the pituitary gland grows bigger , but it doesn’t get extra blood supply.
If a woman has severe blood loss during delivery , it can lead to pituitary infarction (death of the gland tissue due to low blood flow).
This causes hypopituitarism (loss of pituitary hormones).
🧠 That explains all her symptoms:
No prolactin → poor lactation
No LH/FSH → loss of pubic hair (sex hormones drop)
No ACTH or TSH → fatigue due to low cortisol and thyroid hormone
❌ Why the Other Options Are Wrong (Simple):
Pituitary adenoma
Usually causes too much hormone , not too little .
Also, symptoms don’t appear right after delivery with bleeding.
Prolactinoma
Empty sella syndrome
A condition where the sella turcica (pituitary space) looks empty on imaging.
It may cause mild pituitary dysfunction , but it’s not linked to childbirth or bleeding .
Acromegaly
Caused by excess growth hormone in adults.
Leads to large hands, jaw, and feet , not the symptoms seen here.
When a patient presents with signs of hormone excess and imaging shows a brain mass, consider not just the symptoms — but which hormone might be overproduced due to a central source.
92 / 232
Tags:
2020
A 42-year-old woman presents to the clinic complaining of excessive facial hair and irregular menstrual cycles. On physical exam, the patient has truncal obesity and a characteristic ‘moon-facies’. CT of the head shows a mass in the pituitary gland. What is the most likely diagnosis?
This woman has several signs that point toward excess cortisol in the body:
Excess facial hair (hirsutism)
Irregular menstrual cycles
Truncal obesity
Moon facies (rounded face)
These are hallmark features of Cushing syndrome , which is a general term for any condition with too much cortisol .
However, the CT scan shows a pituitary mass — and that’s the key clue .
This makes the diagnosis Cushing disease , which is:
Cushing syndrome caused by a pituitary tumor that secretes too much ACTH , which stimulates the adrenal glands to produce too much cortisol .
❌ Why the Other Options Are Wrong (Simple):
If a young person presents with physical changes that match the effects of long-term hormone imbalance — like altered fat distribution, hypertension, and weakened muscles — ask yourself what kind of endocrine overactivity could be driving it.
93 / 232
Tags:
2020
A 15-year-old boy presents to the outpatient department with backache, moon face, and truncal obesity. On examination, the child has elevated blood pressure and delayed reflexes. The laboratory results show increased cortisol levels. What is the most likely disease from which the boy is suffering?
The boy is showing classic signs of Cushing syndrome , which is caused by too much cortisol in the body. Let’s go through the key clues in the question:
🧩 Symptoms:
Backache → Suggests muscle weakness or bone problems (cortisol causes bone breakdown).
Moon face & truncal obesity → Very characteristic signs of excess cortisol .
High blood pressure → Cortisol increases blood pressure.
Delayed reflexes → Seen in cortisol excess or associated electrolyte issues.
High cortisol levels in labs → Confirms hypercortisolism.
Now, the question is: What is the cause of this excess cortisol?
🧠 What is Cushing disease ?
Cushing disease is a specific cause of Cushing syndrome — it means the pituitary gland is producing too much ACTH , which then stimulates the adrenal glands to produce too much cortisol .
Common in adolescents , especially with the pattern of moon face , central obesity , and elevated cortisol .
So, this is the most likely diagnosis based on age, symptoms, and labs .
❌ Why the Other Options Are Wrong (Simple):
Pheochromocytoma
A tumor of the adrenal medulla (not cortex) that makes adrenaline , not cortisol.
It causes sudden high blood pressure , palpitations, and sweating — not moon face or obesity .
Addison disease
This is adrenal insufficiency , meaning low cortisol , not high.
It causes weight loss, low blood pressure, hyperpigmentation — opposite of what we see here.
Adrenal cortex tumor
This can cause high cortisol, but it’s rare in children and usually does not cause elevated ACTH (it’s ACTH-independent).
Cushing disease is much more common in this age group and fits better.
Pseudo-Cushing syndrome
Happens in alcoholism, depression, or severe obesity , where cortisol might be temporarily elevated , but symptoms are not as classic or consistent .
This child has clear Cushingoid features , so this is unlikely.
When the thyroid goes under the knife, the parathyroids panic — or vanish.
94 / 232
Tags:
2024
Which of the following is the most common cause of hypoparathyroidism ?
The most common cause of hypoparathyroidism is accidental removal or damage to the parathyroid glands during thyroidectomy or other neck surgeries.
This leads to low PTH , which results in:
❌ Why “Adenoma” Is Incorrect:
Parathyroid adenoma is the most common cause of hyperparathyroidism , not hypo.
It leads to increased PTH , hypercalcemia , and bone resorption.
f the eyes are popping out like headlights — it’s not just a delayed blink.
95 / 232
Tags:
2024
A patient presents with bulging, outwardly protruding eyes , often seen in association with Graves’ disease. What is the correct medical term for this clinical finding?
Term
Meaning
Common Cause
Exophthalmos ✅
Protrusion of the eyeballs beyond the orbit
Graves’ disease (thyroid eye disease)
Lid lag ❌
Upper eyelid lags behind when looking downward
Hyperthyroidism, but not protrusion
Exophthalmos is due to inflammatory enlargement of extraocular muscles and orbital fat , pushing the eye forward.
Seen classically in Graves’ orbitopathy , and can be measured with an exophthalmometer .
Lid lag is a different clinical sign — the upper eyelid fails to follow the eyeball promptly when the patient looks downward, but the eye itself remains in the socket.
❌ Why Lid Lag Is Incorrect:
Lid lag is commonly present in hyperthyroidism , but it is a neuromuscular response , not a structural protrusion .
You may have lid lag without exophthalmos .
When the body holds water but not salt, the sodium drowns in dilution.
96 / 232
Tags:
2024
A hospitalized patient is found to have excessive secretion of antidiuretic hormone (ADH), leading to water retention without appropriate sodium retention. Based on this pathophysiology, which of the following electrolyte imbalances is most commonly seen in SIADH?
In SIADH (Syndrome of Inappropriate Antidiuretic Hormone secretion) :
ADH (vasopressin) promotes water reabsorption from the kidneys, especially in the collecting ducts.
This leads to water retention without proportional sodium retention.
The result is a dilutional hyponatremia : total body water increases, but total body sodium stays the same or decreases → plasma sodium concentration drops.
Patients are often euvolemic (normal body fluid volume) because of compensatory natriuresis (kidneys excrete sodium to get rid of some water).
❌ Why Other Options Are Incorrect:
Hypernatremia ❌ Seen in diabetes insipidus (ADH deficiency), not SIADH.
Hypokalemia ❌ Not characteristic of SIADH. More common in conditions with excessive aldosterone.
Hyperkalemia ❌ No direct link with SIADH. Seen in renal failure, hypoaldosteronism, etc.
Hypochloremia ❌ Might occur secondarily but not the defining feature . Hyponatremia is the key lab abnormality.
Consider the only thyroid condition where the immune system affects not only the gland but also tissues behind the eyes, leading to a very striking facial appearance.
97 / 232
Tags:
2022
A 50-year-old lady presents to the emergency department with an enlarged thyroid, protruded eyes, severe palpitations, sweating, and complaints of fatigue. What is the diagnosis?
The clinical picture described is classic for Graves disease , the most common cause of hyperthyroidism , particularly in middle-aged women.
🔍 Key Clinical Features in This Case:
Enlarged thyroid → indicates goiter
Protruded eyes (exophthalmos) → hallmark of Graves disease due to autoimmune orbital fibroblast stimulation
Severe palpitations and sweating → signs of increased sympathetic activity
Fatigue → paradoxically seen in hyperthyroidism due to muscle catabolism and increased metabolic rate
🦠 Pathophysiology:
Graves disease is an autoimmune disorder where TSI (thyroid-stimulating immunoglobulins) mimic TSH and stimulate the thyroid gland excessively, leading to overproduction of T3 and T4 .
❌ Why the Other Options Are Incorrect:
Hashimoto’s thyroiditis : Causes hypothyroidism , not hyperthyroidism; commonly presents with fatigue, weight gain, and cold intolerance .
Multinodular goiter : Can lead to hyperthyroidism (toxic variety), but does not cause exophthalmos .
Myxedema : Severe hypothyroidism with features like puffy face, bradycardia, and dry skin , not palpitations and sweating.
Endemic goiter : Related to iodine deficiency ; presents with thyroid enlargement , but no exophthalmos or classic hyperthyroid signs .
Which volatile substance, commonly used in nail polish remover, can also make its way into your breath when your body burns fat instead of sugar?
98 / 232
Tags:
2022
What is the name of the sweet odor breath that is often associated with uncontrolled diabetes?
In uncontrolled diabetes mellitus , especially Type 1 diabetes , the body cannot utilize glucose properly due to a lack of insulin. As a result, it switches to fat metabolism for energy. This process leads to the production of ketone bodies , including:
Acetone is exhaled via the lungs and gives a characteristic fruity or sweet odor to the breath. This symptom is particularly associated with diabetic ketoacidosis (DKA) —a life-threatening complication of uncontrolled diabetes.
❌ Why the Other Options Are Incorrect:
Methane breath : Associated with bacterial overgrowth or digestive disorders , not diabetes.
Halitosis : A general term for bad breath , without specifying cause—could be dental, GI, or respiratory in origin.
Garlic breath : Caused by consumption of garlic or related sulfur-containing compounds, not linked to diabetes .
Ammonia breath : Seen in kidney failure due to urea breakdown, not in diabetic ketoacidosis .
In a patient with suspected hormone deficiency and dangerously low pressure, what should you restore before trying to name the disease?
99 / 232
Tags:
2022
A 45-year-old man presents to the emergency department with a 2-week history of fatigue, weakness, and weight loss. He also complains of nausea, vomiting, and dizziness. He has a history of chronic corticosteroid use for rheumatoid arthritis. On examination, he is hypotensive and has hyperpigmentation of the skin. His laboratory results show low sodium, glucose, and high potassium levels. Serum cortisol is low. What is the next best step in management?
This patient is presenting with acute adrenal insufficiency (also known as adrenal crisis ), a life-threatening emergency . The symptoms and lab findings are classic:
🔍 Key Clinical Clues:
History of chronic corticosteroid use → risk of adrenal suppression
Fatigue, weight loss, hypotension, nausea, vomiting → signs of adrenal crisis
Hyperpigmentation → suggests elevated ACTH (primary insufficiency may be unmasked)
Labs :
↓ Sodium
↑ Potassium
↓ Glucose
↓ Cortisol
This triad is classic for adrenal insufficiency , especially primary , though it may be mixed with secondary if steroid withdrawal is involved.
🩺 Immediate priority is not diagnosis, but survival .
➡️ Start empiric treatment immediately :
Once stabilized, you can pursue confirmatory diagnostic tests later (e.g., ACTH level, ACTH stimulation test, CT scan, etc.).
❌ Why the Other Options Are Incorrect Now:
Morning ACTH levels : Important for diagnosis later , but delay in treatment risks death.
Dexamethasone suppression test : For Cushing’s diagnosis, not useful here.
Saline challenge test : Not a diagnostic step for adrenal insufficiency.
Abdominal CT scan : Can help detect adrenal pathology but is not urgent in acute crisis.
If a hormone is causing your blood pressure to stay high despite treatment, where should you look to find the source?
100 / 232
Tags:
2022
A 55-year-old female presents to her primary care physician with uncontrolled hypertension. She reports no symptoms but her blood pressure is found to be consistently elevated in her recent visits. She has been on multiple anti-hypertensive medications without success. Laboratory tests reveal low plasma renin activity and high aldosterone levels. What is the next best investigation?
This patient’s uncontrolled hypertension , despite multiple medications, along with high aldosterone and low plasma renin activity (PRA) , is highly suggestive of primary hyperaldosteronism —most commonly caused by:
Once a biochemical diagnosis (↑ aldosterone + ↓ renin) is made, the next step is to localize the source of excess aldosterone production .
🖥️ Best next test :
➡️ CT scan of the abdomen , focused on the adrenal glands .
This will help detect:
❌ Why the Other Options Are Incorrect:
Renal biopsy : Not indicated here. There’s no evidence of glomerular disease or nephritic/nephrotic syndrome.
Plasma metanephrines : Used to evaluate for pheochromocytoma , which would present with episodic hypertension, palpitations, headaches, and sweating , not isolated high aldosterone.
Doppler ultrasound of abdomen : Good for renal artery stenosis , which causes secondary hyperaldosteronism (both aldosterone and renin elevated ), not what we see here.
Brain MRI : Unrelated unless neurological symptoms are present or you’re investigating secondary hypertension from central causes , which this case doesn’t suggest.
Which condition slows down every system in the body—including heart rate, reflexes, and metabolism—and often hides behind vague symptoms like fatigue and weight gain?
101 / 232
Tags:
2022
A 30-year-old female patient, obese, nondiabetic, presented with significant weight gain, easy fatiguability, and lethargy. On examination, she looked pale, obese, and slightly confused. Her blood pressure is 170/90 mmHg, and her pulse rate is 52 beats/min. She was unable to stand from a sitting position without support and her deep tendon reflexes were diminished. With the given scenario, what is the probable diagnosis?
This 30-year-old obese female presents with a constellation of symptoms that point strongly toward hypothyroidism :
🔍 Key Clinical Features:
Weight gain & lethargy → Common in hypothyroidism due to slowed metabolism
Pallor & confusion → Suggest reduced metabolic activity and possibly anemia
Bradycardia (52 bpm) → Classic sign of reduced thyroid hormone activity on the heart
Hypertension (170/90 mmHg) → Due to increased peripheral vascular resistance in hypothyroidism
Inability to rise from sitting → Suggests proximal muscle weakness or myopathy
Diminished deep tendon reflexes (especially delayed relaxation) → Very characteristic of hypothyroidism
🧪 What labs would support this?
↓ Free T₄ , ↑ TSH (in primary hypothyroidism)
Possible normocytic or macrocytic anemia
Hyperlipidemia may also be present
❌ Why the Other Options Are Incorrect:
Cushing syndrome : Causes weight gain and hypertension, but typically features central obesity, purple striae, facial rounding (moon face) , and muscle weakness without bradycardia or delayed reflexes.
Obstructive sleep apnea (OSA) : May cause fatigue and hypertension but not bradycardia, reflex changes, or muscle weakness .
Polycystic ovarian disease (PCOS) : Usually presents with menstrual irregularity, hirsutism, acne , and infertility —not bradycardia or delayed reflexes.
Obesity : May explain weight gain and fatigue but does not explain bradycardia, reflex changes, or confusion .
Which molecules rise during chronic stress and inflammation, quietly damaging blood vessels and increasing the risk of heart attacks—especially in overweight, sleep-deprived individuals?
102 / 232
Tags:
2022
A 35-year-old young male working in a marketing company was referred to you by a cardiologist after a minor angina attack. He complained of decreased sleep and pain in neck muscles. He works almost 10 to 12 hours a day and rides motorbikes to meet his clients in the city. He gained weight of 15 Kg in the last two months, having a body mass index of 35 Kg/m^2. What could be the possible reason for his recent attack?
This patient is a 35-year-old male with:
Among these factors, the most biochemically and clinically significant contributor to a cardiac event in such a young individual is chronic stress , which triggers low-grade systemic inflammation .
Stress activates the hypothalamic-pituitary-adrenal (HPA) axis and sympathetic nervous system , leading to the release of:
IL-6, in turn, stimulates the liver to produce C-reactive protein (CRP) , a key inflammatory marker strongly associated with:
This inflammatory state can accelerate coronary artery disease (CAD) and trigger acute coronary syndromes even in younger individuals, especially those with visceral obesity and poor sleep .
❌ Why the Other Options Are Incorrect:
Riding a motorbike : Increases musculoskeletal strain, but is not a direct cardiac risk factor .
Nature of job : Contributes to stress but is too vague ; the inflammatory response is more directly causative.
Age factor : 35 is young for typical angina; age alone is not the main issue here.
Decreased sleep : A risk factor for cardiovascular disease but mediated through inflammatory markers like IL-6 and CRP.
Which syndrome causes rapid adrenal failure due to hemorrhage, often triggered by a rapidly progressing bacterial infection in the bloodstream?
103 / 232
Tags:
2022
A clinical study is performed involving subjects who developed Addison’s disease. They were recorded to have laboratory studies with hyponatremia, hyperkalemia, hypoglycemia, and decreased plasma cortisol. They became hypotensive. In some subjects, this disease had an acute onset in less than 2 days. Which of the following conditions is most likely to produce this acute course?
Addison’s disease refers to primary adrenal insufficiency , where the adrenal cortex fails to produce cortisol, aldosterone, and androgens . The symptoms described—hyponatremia, hyperkalemia, hypoglycemia, low cortisol, and hypotension —are classic signs of acute adrenal insufficiency .
Among the options, the only condition that causes such a sudden, life-threatening onset of Addisonian crisis is Waterhouse-Friderichsen syndrome , which involves:
Bilateral adrenal hemorrhage
Typically secondary to septicemia , especially Neisseria meningitidis
Rapid onset of hypotension, shock, DIC , and acute adrenal failure
This syndrome can develop in less than 2 days , making it the best match for the described acute course .
❌ Why the Other Options Are Incorrect:
Reactive systemic amyloidosis : Causes chronic adrenal damage; not sudden in onset.
Disseminated TB : Can cause Addison’s but typically does so gradually over time , not acutely.
Metastatic small cell carcinoma : Can invade adrenals, but the process is gradual , not explosive.
Blunt force abdominal trauma : Rarely causes bilateral adrenal hemorrhage —and even when it does, it’s not a classic cause of acute Addisonian crisis.
What condition arises when there’s no insulin to suppress fat breakdown, leading to acid build-up, dehydration, and coma—even if the blood sugar is extremely high?
104 / 232
Tags:
2022
A 57-year-old man is found comatose. On physical examination, he has decreased skin turgor. Laboratory studies show a blood glucose of 780 mg/ dL. Urinalysis reveals ketosis and proteinuria along with 4+ glycosuria. Which of the following is the most likely diagnosis?
This 57-year-old man presents comatose with:
Severe hyperglycemia (glucose: 780 mg/dL)
Ketosis
Dehydration (evident by decreased skin turgor)
Glycosuria and proteinuria
These findings are most consistent with Diabetic Ketoacidosis (DKA) —a serious complication typically seen in Type I diabetes mellitus .
In Type I diabetes :
There’s absolute insulin deficiency due to autoimmune destruction of pancreatic beta cells .
Without insulin, glucose cannot enter cells → hyperglycemia
Fat breakdown is increased → ketone production
Ketosis leads to metabolic acidosis , dehydration, and can progress to coma
Although the patient is older than typical onset, latent autoimmune diabetes in adults (LADA) can present similarly.
❌ Why the Other Options Are Incorrect:
Cushing syndrome : Causes hyperglycemia due to cortisol excess but not ketosis or DKA .
Type II diabetes mellitus : DKA is rare in type II; more often associated with hyperosmolar hyperglycemic state (HHS) , which lacks ketosis .
Neuroendocrine tumor secreting glucagon (glucagonoma) : Can raise glucose but doesn’t cause DKA or glycosuria of this severity.
Ingestion of large quantity of sugar : May cause transient hyperglycemia and glycosuria but not ketosis or coma .
Which bone condition arises from too much parathyroid hormone eating away at bone—leaving behind cysts, hemorrhage, and pain?
105 / 232
Tags:
2022
A 49-year-old female experienced multiple attacks of abdominal pain last year. Three times she had passed stones in the urine during these episodes of pain. Now she is experiencing pain in her right middle finger for one month. On physical examination, her finger is tender. Laboratory reports show calcium 13.7 mg/ dL, phosphate 1.9 mg/ dL, and Creatinine 1.1mg/ dL, albumin 4.8 mg/ dL. Which of the following bone lesions is she most likely to have?
This patient presents with:
These are classic features of primary hyperparathyroidism , most often caused by a parathyroid adenoma .
In this condition:
Excess parathyroid hormone (PTH) increases bone resorption , releasing calcium and phosphate into the blood.
Kidneys excrete more phosphate , leading to hypophosphatemia
Increased calcium can result in nephrolithiasis (kidney stones)
The resulting bone lesion from prolonged PTH overactivity is osteitis fibrosa cystica . It is characterized by:
Subperiosteal bone resorption
Fibrous tissue replacement in bone
Formation of brown tumors (not true neoplasms, but reactive lesions due to bone turnover and hemorrhage)
❌ Why the Other Options Are Incorrect:
Osteoma : A benign, slow-growing bone tumor; not associated with hypercalcemia or systemic symptoms.
Osteochondroma : A benign cartilage-capped bony projection; unrelated to calcium/phosphate imbalances.
Osteoid : Not a disease—refers to unmineralized bone matrix, not a lesion type.
Osteomyelitis : Bone infection, typically with fever, elevated inflammatory markers—not linked to hypercalcemia or kidney stones.
Which mineral drops dangerously if parathyroid glands are damaged—causing the nerves and muscles to become overly excitable?
106 / 232
Tags:
2022
After undergoing total thyroidectomy for follicular carcinoma of the thyroid a 50-year-old man develops a tingling sensation and neuromuscular instability post-operatively. Which of the following laboratory test should be carried out on an urgent basis to determine the further proceedings of the therapy?
The patient is showing classic signs of hypocalcemia —tingling, neuromuscular irritability, and instability —which are common complications after total thyroidectomy . During surgery, the parathyroid glands (located on the posterior surface of the thyroid) can be:
This leads to decreased parathyroid hormone (PTH) levels, which results in hypocalcemia . Therefore, the urgent test needed is serum calcium to confirm and guide prompt calcium replacement therapy .
❌ Why the Other Options Are Incorrect:
Thyroid stimulating hormone (TSH) : Important for long-term monitoring, but not urgent post-op and does not explain neuromuscular symptoms .
Serotonin : Unrelated to calcium metabolism or thyroid surgery.
Iodine : Involved in thyroid hormone synthesis but not relevant post-thyroidectomy or for current symptoms.
Parathormone (PTH) : While helpful, PTH levels take longer to return and are not the immediate concern —serum calcium is faster and directly confirms the clinical problem.
Which hormone, when chronically elevated, tells the body to make more glucose and at the same time makes muscles ignore insulin?
107 / 232
Tags:
2022
A 45-year-old male was diagnosed with adrenal diabetes as a result of high serum levels of cortisol due to hypersecretion of the adrenocorticotrophic hormone. Which of the following is associated with the above mentioned condition?
The condition described is adrenal diabetes , which results from chronic excess cortisol , often due to ACTH hypersecretion (e.g., Cushing disease ).
Cortisol is a glucocorticoid hormone that:
Promotes gluconeogenesis in the liver
Reduces glucose uptake in skeletal muscle and adipose tissue
Leads to insulin resistance —cells don’t respond to insulin as effectively
As a result:
Blood glucose levels rise
Skeletal muscle cells become less sensitive to insulin
This can mimic or contribute to the development of type 2 diabetes
This explains why high cortisol levels can lead to hyperglycemia , termed “adrenal diabetes.”
❌ Why the Other Options Are Incorrect:
Enhanced deposition of fatty acids in adipose tissue : Cortisol actually promotes lipolysis , especially in limbs, though it causes central fat deposition .
Absence of beta cells of pancreas : Seen in type 1 diabetes , not adrenal diabetes.
Decrease in blood glucose level : Cortisol raises , not lowers, blood glucose.
Enhanced utilization of glucose by cells : The opposite occurs—glucose uptake is impaired .
If TSH can be boosted by giving TRH, where must the original defect be—in the gland that produces TSH or the one that stimulates it?
108 / 232
Tags:
2022
Lab investigations of a 35-year-old woman revealed low serum levels of thyroxine, and serum level of thyroid-stimulating hormones was also down but a rapid increase in TSH level was seen when given a synthetic thyroid releasing factor. Which of the following can be the reason for her hypothyroidism?
This patient has low levels of both thyroxine (T₄) and thyroid-stimulating hormone (TSH) , which suggests that the thyroid gland itself is not the primary problem . Instead, the issue lies upstream —in the hypothalamic-pituitary axis .
The key diagnostic clue here is the rapid increase in TSH after administration of synthetic thyrotropin-releasing hormone (TRH) . This indicates that:
This points to a hypothalamic origin of the hypothyroidism—a condition called tertiary hypothyroidism .
❌ Why the Other Options Are Incorrect:
Posterior pituitary : Releases ADH and oxytocin , not involved in thyroid regulation.
Thyroid receptor : A defect here would cause resistance to thyroid hormone , not low T₄ with low TSH.
Thyroid follicles : Affected in primary hypothyroidism , where TSH would be high due to lack of feedback inhibition—not the case here.
Anterior pituitary : If it were the problem (secondary hypothyroidism), it would not respond to TRH stimulation —but here it does, so it’s functioning properly.
Which hormone is produced by the pituitary and acts like a thermostat, adjusting up or down based on circulating thyroid hormone levels?
109 / 232
Tags:
2022
A 29-year-old man complains of weight gain, constipation, decreased energy, lethargy, and dry skin for the past 6 months. He was diagnosed with hypothyroidism and was put on synthetic thyroid hormone therapy. A decrease in which of the following hormonal levels should be observed to find if the treatment is effective?
In primary hypothyroidism , the thyroid gland underproduces thyroid hormones (T₃ and T₄), which leads to elevated TSH levels due to feedback from the anterior pituitary trying to stimulate the thyroid.
When a patient is treated with synthetic thyroid hormone (usually levothyroxine, a form of T₄) :
T₄ levels rise in the blood
This feeds back to the pituitary
Resulting in a decrease in TSH levels
Therefore, TSH is the most sensitive and reliable marker for monitoring treatment response in primary hypothyroidism.
❌ Why the Other Options Are Incorrect:
Vitamin A : Unrelated to thyroid function or therapy monitoring.
Free T₄ : Although it should rise with therapy, TSH is more reliable for long-term monitoring.
Plasma iron : May be affected by metabolism, but not specific to thyroid therapy response.
Plasma cholesterol : Often elevated in hypothyroidism and may improve, but it’s indirect and variable .
Free T₃ : Less reliable and fluctuates more ; not the preferred marker for monitoring therapy.
If the thyroid gland isn’t responding properly, how would the pituitary gland try to correct the hormone imbalance?
110 / 232
Tags:
2022
A person is diagnosed with hypothyroidism and goiter. Her laboratory tests most likely show which of the following?
In primary hypothyroidism , the thyroid gland fails to produce enough thyroid hormones (T₃ and T₄) . As a result, the pituitary gland compensates by increasing secretion of thyroid-stimulating hormone (TSH) to stimulate the thyroid.
This combination—low T₃/T₄ and high TSH —is characteristic of primary hypothyroidism , and if prolonged, the persistent stimulation by TSH leads to goiter formation (thyroid gland enlargement).
❌ Why the Other Options Are Incorrect:
Decreased TSH and decreased T₃, T₄ : Suggests secondary hypothyroidism (problem in the pituitary), not the most common form associated with goiter.
Increased TSH and increased T₃, T₄ : Would be seen in TSH-secreting pituitary adenoma , not hypothyroidism.
Decreased TSH and increased T₃, T₄ : Seen in hyperthyroidism , the opposite condition.
None of these : Incorrect—there is a clearly correct pattern here.
What hormone secreted by the pituitary gives you the quickest, most reliable window into how well the thyroid gland is functioning?
111 / 232
Tags:
2022
A 54-year-old, married woman para-2 comes to the clinic with the complaint of weight gain and lethargy for the past 3 months. She had her last menstrual period 6 years back. Her blood pressure is 150/90 mmHg, her pulse is 65/min, and is regular bilaterally. Her body mass index (BMI) falls in the obese range. Her eyes are puffy, she complains of breathlessness, and cold intolerance, her skin is coarse and her abdomen has white striations present. What is the best diagnostic test for this woman?
This woman presents with classic signs of hypothyroidism :
Weight gain , lethargy , cold intolerance
Bradycardia (pulse of 65/min), coarse skin , puffy eyes
Menopausal status with metabolic slowing
Hypertension , obesity , and breathlessness (possible due to decreased metabolism or myopathy)
These features strongly point to primary hypothyroidism , and the best initial and most sensitive test to confirm this is the serum TSH level . In primary hypothyroidism, TSH will be elevated , as the pituitary tries to stimulate a failing thyroid.
If the TSH is abnormal, it can be followed by a free T4 test to assess the degree of hormone deficiency.
❌ Why the Other Options Are Incorrect:
Oral glucose tolerance test : Not first-line here. While she may be at risk of diabetes due to obesity, her symptoms are better explained by hypothyroidism.
Serum cortisol : Tests for adrenal insufficiency (e.g., Addison’s), which doesn’t match her presentation.
Serum cortisol and TSH : Unnecessary unless there’s suspicion of multiple endocrine disorders; TSH alone is sufficient initially.
Oral glucose tolerance test and TSH : Again, adds extra testing—TSH alone is the focused and best first test .
Which complication involves damage to major blood vessels and is the primary reason for increased risk of heart attacks and strokes in people with long-standing diabetes?
112 / 232
Tags:
2022
What is the marked macrovascular complication of type 2 diabetes mellitus?
In type 2 diabetes mellitus , chronic high blood sugar damages blood vessels throughout the body. Complications can be broadly divided into:
Microvascular (small vessel): Retinopathy, nephropathy, neuropathy
Macrovascular (large vessel): Atherosclerosis , leading to cardiovascular disease
The most prominent macrovascular complication is accelerated atherosclerosis , which increases the risk of:
Coronary artery disease (heart attacks)
Cerebrovascular disease (strokes)
Peripheral artery disease (limb ischemia)
Diabetes promotes inflammation, endothelial dysfunction, and lipid abnormalities , all of which speed up plaque formation in arteries, making cardiovascular disease the leading cause of death in diabetics.
❌ Why the Other Options Are Incorrect:
Retinal vein occlusion : Can occur in diabetes, but it is not the hallmark macrovascular complication ; it is more of a microvascular event .
Tunnel vision : Seen in advanced glaucoma or retinitis pigmentosa , not specific to diabetes.
Diminishing vision : A symptom of diabetic retinopathy , which is a microvascular (not macrovascular) complication.
Accelerated arthritis : Not a recognized complication of type 2 diabetes.
When a patient with an overactive thyroid also presents with eye discomfort and a noticeable change in eye appearance, what underlying immune-related condition should you consider?
113 / 232
Tags:
2022
A 30-year-old woman presents to the clinic with palpitations, weight loss, and excessive sweating for almost 5-6 months. She cannot concentrate on carrying out her daily activities especially looking after her children. Recently, she also developed pain in both eyes and her friend told her that one of her eyes has become more prominent than the other. What is the most likely cause of her condition?
This 30-year-old woman presents with classic signs of hyperthyroidism —including palpitations , weight loss , heat intolerance (excessive sweating) , and difficulty concentrating —but what makes this case distinctive is her eye symptoms : pain and proptosis (bulging of the eye) . These features are strongly suggestive of Graves’ disease , which is the only type of hyperthyroidism commonly associated with ophthalmopathy .
Graves’ disease is an autoimmune condition where TSH receptor antibodies (TRAb) stimulate the thyroid gland, causing excess hormone production. These antibodies can also stimulate fibroblasts and immune cells in the orbital tissues, leading to Graves’ orbitopathy (eye pain, swelling, redness, and protrusion).
❌ Why the Other Options Are Incorrect:
Toxic nodular goiter : Causes hyperthyroidism, but not associated with eye symptoms . It’s more common in older individuals and has a more gradual onset.
Generalized anxiety disorder : May cause palpitations and restlessness, but does not explain the weight loss, sweating, or eye findings .
Hyperthyroidism : A general term for excess thyroid hormone—it describes the condition but not the specific cause , which in this case is Graves’.
Hashimoto’s thyroiditis : Typically leads to hypothyroidism , not hyperthyroidism. In early stages, it may transiently cause hyperthyroid symptoms, but does not cause eye changes .
If infections are well controlled and rare, what other internal process could quietly destroy the glands without any external threat?
114 / 232
Tags:
2022
What is the most common cause of Addison’s disease in the West?
In Western countries, the most common cause of Addison’s disease is autoimmune adrenalitis . This means the body’s immune system attacks and destroys its own adrenal glands, leading to a shortage of important hormones like cortisol and aldosterone. It often occurs with other autoimmune conditions like type 1 diabetes or thyroid problems.
❌ Why the Other Options Are Wrong:
Tumor of adrenal gland : More likely to cause extra hormone production, not hormone loss.
Congenital absence of adrenal gland : Very rare, and mostly affects newborns—not adults.
Infection : Can damage the adrenal glands, but it’s not the top cause in the West.
Tuberculosis : A major cause in developing countries, but much less common in the West today.
When a hormone causes the kidneys to retain sodium and excrete potassium and hydrogen ions , what happens to blood pH? Consider the systemic effect of prolonged hydrogen ion loss and how that shifts acid–base balance.
115 / 232
Tags:
2017
Which metabolic derangement is seen in Conn syndrome?
Conn syndrome , also known as primary hyperaldosteronism , is caused by excess secretion of aldosterone , typically due to an adrenal adenoma or adrenal hyperplasia.
Physiological Role of Aldosterone:
Aldosterone acts on the distal nephron (especially the collecting ducts) of the kidneys to:
Increase sodium reabsorption (→ water retention → hypertension)
Increase potassium excretion (→ hypokalemia)
Increase hydrogen ion excretion (→ metabolic alkalosis)
Why Metabolic Alkalosis?
Incorrect Answer Explanations:
Hyperkalemia
Hypercalcemia
Metabolic acidosis
Incorrect : This would occur if hydrogen ions were retained , such as in renal failure or type 1 renal tubular acidosis .
In Conn syndrome, H⁺ is excessively excreted , leading to alkalosis.
Respiratory acidosis
Incorrect : This is due to hypoventilation , causing CO₂ retention (e.g., in COPD or CNS depression).
Conn syndrome is a metabolic , not respiratory, disorder.
If the brain’s “thermostat” is trying to cool things down, but the system keeps heating up anyway, where do you think the problem originates—the controller or the machinery?
116 / 232
Tags:
2022
A 35-year-old female went to see her doctor because she had been feeling hot lately and was sweating at night. Her T3 and T4 levels were found to be high, whereas her TSH level was low. What is the most likely diagnosis made by the doctor?
This patient shows classic signs of hyperthyroidism —feeling hot, night sweats, and likely other symptoms like weight loss, palpitations, or anxiety. Her lab results reveal:
This pattern tells us that the thyroid gland itself is overactive , producing too much hormone. The pituitary gland senses this excess and reduces TSH production to try to bring hormone levels down—this is a normal negative feedback response .
This is called primary hyperthyroidism , because the problem starts in the thyroid gland itself .
❌ Why the Other Options Are Incorrect:
Hypothyroidism : ❌ In this condition, T3 and T4 would be low , not high.
Myxedema : ❌ This is a severe form of hypothyroidism , often with very low thyroid hormone levels and high TSH —not consistent with the patient’s findings.
Pituitary adenoma : ❌ A tumor in the pituitary that secretes TSH would cause high TSH along with high T3 and T4—this patient has low TSH , so this isn’t the cause.
Secondary hyperthyroidism : ❌ This means the pituitary gland is causing the problem , typically with elevated TSH , which then stimulates the thyroid. But here, TSH is low , ruling this out.
Consider the direct physical effects of fat accumulation in the bloodstream . When certain lipid levels become extremely high, they don’t just affect vessels—they can trigger acute organ damage through enzymatic or metabolic complications. Think about which organ deals with digestion and how excess fat might overwhelm its normal function.
117 / 232
Tags:
2017
Hypertriglyceridemia is associated with which of the following condition?
When Triglycerides Are Dangerously High (> 1000 mg/dL):
Pancreatitis becomes a significant risk.
The excess triglycerides are broken down by pancreatic lipase into free fatty acids (FFAs) in the pancreatic capillaries.
FFAs are toxic at high concentrations , leading to pancreatic inflammation, ischemia, and cellular injury .
Clinical Relevance:
Hypertriglyceridemia is one of the top three causes of acute pancreatitis , along with gallstones and alcohol abuse .
It is particularly common in patients with uncontrolled diabetes, obesity, and familial lipid disorders .
Incorrect Answer Explanations:
Diabetes Type 1
Type 1 Diabetes Mellitus involves autoimmune destruction of pancreatic beta cells , leading to insulin deficiency .
While diabetic ketoacidosis (DKA) may transiently elevate triglycerides, severe hypertriglyceridemia is more typical of Type 2 diabetes or metabolic syndrome , not Type 1.
Not directly linked to pancreatitis unless DKA-induced hypertriglyceridemia occurs, which is rare.
Retinopathy
Diabetic retinopathy is a microvascular complication of diabetes , primarily related to chronic hyperglycemia , not hypertriglyceridemia.
Lipid abnormalities may worsen retinal outcomes but retinopathy is not directly caused by hypertriglyceridemia .
Diabetic Foot
Diabetic foot ulcers are caused by neuropathy, ischemia, and infection .
There is no direct association with hypertriglyceridemia .
Appendicitis
Appendicitis is due to inflammation of the appendix , commonly from luminal obstruction (like fecaliths or lymphoid hyperplasia).
There is no link between triglyceride levels and appendicitis .
Consider whether the condition affects glucose metabolism or insulin regulation , or if it’s primarily a structural disorder of an organ unrelated to metabolic control. Does this condition involve cysts in the kidneys , or does it involve vascular, cardiac, or metabolic dysfunction ?
118 / 232
Tags:
2017
Which of the following is not associated with diabetes mellitus?
Polycystic kidney disease (PKD) is a genetic disorder that leads to the formation of multiple fluid-filled cysts in the kidneys .
It primarily affects kidney structure and function , leading to:
**PKD is not directly associated with diabetes mellitus or its pathogenesis.
It does not affect insulin production, insulin resistance, or glucose metabolism .
Conditions Associated with Diabetes Mellitus:
Condition
Association with Diabetes
Obesity
✅ Major risk factor for type 2 diabetes due to insulin resistance .
Genetic predisposition
✅ Family history is a key risk factor , especially for type 2 diabetes .
Heart disease
✅ Cardiovascular disease is a major complication of diabetes .
Atherosclerosis
✅ Diabetes accelerates atherosclerosis , increasing the risk of stroke, MI, and PAD .
Why the Other Options Are Incorrect:
Option
Why It’s Incorrect
Obesity
❌ Strongly linked to insulin resistance and type 2 diabetes .
Genetic predisposition
❌ Family history is a well-established risk factor .
Heart disease
❌ Diabetes is a major cause of cardiovascular disease .
Atherosclerosis
❌ Diabetes accelerates atherosclerosis , leading to vascular complications.
Summary:
Polycystic kidney disease is not associated with diabetes mellitus , whereas obesity, genetics, heart disease, and atherosclerosis are closely linked to diabetes and its complications.
Consider the type of thyroiditis that occurs after a viral infection like mumps. This condition typically causes thyroid pain, inflammation, and transient thyroid dysfunction . Does the primary problem involve autoimmunity, infection, or inflammation , and what would be the most logical way to reduce the inflammatory process ?
119 / 232
Tags:
2017
A woman developed severe inflammation of the thyroid gland after being infected by mumps. What is the treatment of choice?
Subacute thyroiditis is also called De Quervain’s thyroiditis or granulomatous thyroiditis .
Often follows viral infections such as mumps, influenza, or coxsackievirus .
Characterized by:
Painful, tender thyroid gland
Transient hyperthyroidism (due to release of preformed thyroid hormones)
Possible hypothyroid phase before recovery
Treatment of Choice:
The main goal is to reduce pain and inflammation .
First-line therapy :
Non-steroidal anti-inflammatory drugs (NSAIDs) for mild cases
Glucocorticoids (e.g., prednisone) if NSAIDs are not sufficient or if symptoms are severe
Beta blockers can be used for symptomatic control of hyperthyroidism (e.g., palpitations) but do not treat the inflammation.
Levothyroxine is not required initially , unless the patient becomes hypothyroid long-term .
Anti-thyroid drugs (e.g., methimazole) are not indicated , because the hyperthyroidism is due to hormone leakage , not overproduction.
Why the Other Options Are Incorrect:
Option
Why It’s Incorrect
Levothyroxine
❌ Not needed unless there’s long-term hypothyroidism after the inflammation subsides.
Beta blockers
❌ Help control symptoms of hyperthyroidism but do not treat the underlying inflammation .
Anti-thyroid drugs
❌ Ineffective because thyroid hormone is being released from damaged cells , not overproduced.
NSAIDs alone
❌ May be tried first for mild cases, but in severe cases like this (post-mumps thyroiditis) , steroids are often needed .
Summary:
The treatment of subacute (De Quervain’s) thyroiditis is NSAIDs and steroids , with steroids reserved for severe or refractory cases .
If a child’s bones are younger than they should be for her age — and she’s growing very slowly — what hormone might be missing from the equation?
120 / 232
Tags:
2023
A 9-year-old girl is brought to the pediatrician due to concerns about her growth. Her parents report that she has consistently been below the 3rd percentile for height and they have noticed a lack of height gain compared to her classmates. Otherwise, she is good at studies. The girl’s medical history is unremarkable and she has not experienced any significant illnesses. On examination, her growth velocity is significantly below the expected range for her age, and her bone age is delayed for his height age. What is the most likely cause of the patient’s short stature?
This 9-year-old girl presents with:
Height below the 3rd percentile
Significantly reduced growth velocity
Delayed bone age (bone maturation is behind her chronological age)
Normal intellect and no other systemic symptoms
These features strongly point toward Growth Hormone (GH) deficiency , which results in:
Poor linear growth
Delayed skeletal maturation (bone age)
Normal proportions (unlike some syndromic causes)
Often normal birth weight and height , but decline in growth by 2–3 years of age
GH deficiency can be congenital or acquired and should be confirmed by GH stimulation testing and IGF-1 levels.
❌ Why the Other Options Are Incorrect
Familial short stature Patients are short but have normal growth velocity and bone age matches chronological age .
Constitutional delay Presents with delayed bone age but normal growth velocity — “late bloomers” who catch up eventually.
Turner syndrome Can cause short stature in girls, but usually also presents with other features (webbed neck, wide-spaced nipples, primary amenorrhea). Also, genetic testing would confirm 45,X karyotype.
Hypothyroidism Would also cause poor growth , but often includes fatigue, constipation, weight gain, and dry skin — not seen here.
When calcium is high and phosphate is low, which hormone is most likely disrupting the balance — and how do we measure it?
121 / 232
Tags:
2023
A 60-year-old lady presented in the clinic with complaints of irritability and lethargy. Her lab values showed Ca=11.2 mg/dL, phosphate levels were low and ALP was raised. What is the best next step to reach the diagnosis?
This 60-year-old woman presents with:
Irritability and lethargy
Hypercalcemia (Ca²⁺ = 11.2 mg/dL)
Low phosphate
Elevated ALP (alkaline phosphatase)
This pattern is classic for primary hyperparathyroidism , where:
PTH is inappropriately elevated despite high calcium
PTH increases calcium by stimulating bone resorption, increasing renal reabsorption, and activating vitamin D
Phosphate is low because PTH promotes phosphate excretion
ALP rises due to increased bone turnover
➡️ Measuring serum PTH is the most specific next step to confirm the diagnosis.
❌ Why the Other Options Are Incorrect
Bone X-ray May show subperiosteal bone resorption or osteitis fibrosa cystica, but it is not the first diagnostic step .
LFTs ALP can be liver-derived, but elevated calcium + low phosphate points to bone/parathyroid etiology.
UCE (Urea, Creatinine, Electrolytes) Helpful in assessing renal function, but won’t confirm parathyroid pathology.
TSH Thyroid dysfunction doesn’t explain this calcium-phosphate pattern .
When thyroid hormones are leaking due to inflammation, would the gland take up more iodine — or less?
122 / 232
Tags:
2023
Which of the following is true regarding De Quervain thyroiditis?
De Quervain thyroiditis (also called subacute granulomatous thyroiditis ) is a self-limited, painful thyroid inflammation , often following a viral upper respiratory infection . It causes:
Painful, tender thyroid
Transient hyperthyroidism (from release of preformed T3/T4)
Low TSH, high T3/T4 initially
Followed by hypothyroidism , then recovery
The radioactive iodine uptake (RAIU) is decreased because the thyroid is not actively synthesizing hormones — it is merely leaking preformed hormone due to inflammation and follicular destruction .
❌ Why the Other Options Are Incorrect
It is painless ❌ Incorrect — painful thyroid is a hallmark feature (unlike silent thyroiditis or Hashimoto’s).
Anti-thyroid drugs are effective ❌ Incorrect — The hyperthyroidism is due to hormone leakage , not overproduction; anti-thyroid drugs (like methimazole) are not effective .
It follows bacterial infection ❌ Incorrect — It usually follows a viral infection (e.g., Coxsackievirus, mumps, adenovirus).
Increased RAIU ❌ Incorrect — RAIU is decreased , because the thyroid is not taking up iodine during the inflammatory phase.
If a thyroid mass is pressing on the trachea and causing airway compromise, what treatment will relieve the pressure — medication or removal?
123 / 232
Tags:
2023
A 60-year-old lady with NKCM presented in the outpatient department (OPD) with complaints of dyspnea and stridor. Examination revealed a large retrosternal goiter. Her thyroid profile was normal. Which of the following would be the best management?
This 60-year-old woman has:
A retrosternal goiter (thyroid enlargement extending behind the sternum)
Dyspnea (difficulty breathing) and stridor (upper airway obstruction sound)
A normal thyroid function test (euthyroid)
The presence of compressive symptoms (e.g., airway obstruction, stridor) in a non-toxic multinodular goiter (NKCM) is an absolute indication for surgery , even if thyroid function is normal.
➡️ The definitive treatment is total thyroidectomy , which:
Relieves tracheal compression
Prevents further growth or future complications
Eliminates recurrence risk in multinodular goiter
❌ Why the Other Options Are Incorrect
Radioactive iodine (RAI) Not effective for large or retrosternal goiters , especially with compressive symptoms . RAI is more suitable for toxic goiters or hyperthyroidism .
Anti-thyroid drugs Used for hyperthyroidism , not for structurally compressive, euthyroid goiters.
Steroids May reduce inflammation , not goiter size. Not appropriate for mechanical obstruction.
Beta-blockers Provide symptomatic relief in hyperthyroidism (e.g., palpitations), but irrelevant here .
What fluid do we always start with to restore circulatory volume and sodium in dehydrated, acidotic patients?
124 / 232
Tags:
2023
A patient presented in the emergency department (ER) with Diabetic Ketoacidosis (DKA). His serum sodium levels were 106 mmol/L. Which of the following fluids would be administered?
In Diabetic Ketoacidosis (DKA) , patients typically present with:
Severe dehydration
Electrolyte imbalances
Metabolic acidosis
Marked hyperglycemia
In this case, the patient has profound hyponatremia (Na⁺ = 106 mmol/L), which requires careful correction .
The first-line fluid in DKA is always isotonic saline (0.9% NaCl or normal saline) because it:
Expands intravascular volume
Begins correcting sodium and fluid deficits
Supports renal perfusion , aiding in clearance of glucose and ketones
Once glucose drops to ~200 mg/dL, 5% dextrose is added to prevent hypoglycemia while continuing insulin.
❌ Why the Other Options Are Incorrect
3% saline Hypertonic and used only for symptomatic severe hyponatremia (e.g., seizures, coma). DKA management does not start with 3% saline.
5% Dextrose Not used initially — added later once blood glucose drops to prevent hypoglycemia.
Hypotonic saline Can worsen hyponatremia and lead to cerebral edema — contraindicated in this setting.
Ringer’s lactate Contains lactate , which can confound acidosis management in DKA.
When a woman misses her periods and has a high prolactin level — what’s the most common , natural cause you must exclude before suspecting disease?
125 / 232
Tags:
2023
A woman presented to the clinic with her husband with complaints of amenorrhea for 4 months. The couple has 2 kids and the husband has azoospermia. The woman has prolactin levels of 400 ng/ml. What will be the next best step?
The first and most essential step in evaluating amenorrhea , especially in a woman of reproductive age, is to rule out pregnancy — regardless of the history.
Even though the husband reportedly has azoospermia , the couple already has two children , meaning prior fertility , and laboratory errors or misdiagnosis in azoospermia are possible.
Additionally, prolactin levels can rise during pregnancy — and a level of 400 ng/mL is within the typical range seen in pregnancy , though also elevated in prolactinomas .
But before jumping to imaging or invasive workup, the most logical and cost-effective step is to check for pregnancy .
❌ Why the Other Options Are Incorrect at This Stage
Repeat husband’s semen analysis Not relevant in the acute evaluation of the woman’s amenorrhea — and she has already conceived before.
MRI abdomen and pelvis Not indicated unless you suspect structural abnormalities (e.g., masses); not first-line for amenorrhea.
MRI pituitary Could be needed if pregnancy is excluded and prolactin remains high — but not before ruling out the most common cause of elevated prolactin: pregnancy .
Ultrasound abdomen and pelvis May help assess uterus and ovaries, but urine/blood pregnancy test is simpler and more direct .
If the pituitary gland isn’t working properly, what happens to the hormones it produces — do they go up, down, or stay the same?
126 / 232
Tags:
2023
What are the lab findings in hypopituitarism?
Hypopituitarism is the deficiency of one or more anterior pituitary hormones , which includes:
When the pituitary fails , it leads to low levels of trophic hormones , including FSH and LH , resulting in:
So, in hypopituitarism , FSH and LH are both low , along with other pituitary hormones.
❌ Why the Other Options Are Incorrect
No change in prolactin levels Prolactin can decrease in hypopituitarism, especially with global pituitary failure.
High FSH and LH Seen in primary gonadal failure , not pituitary failure.
Increased ACTH ACTH is typically low , leading to secondary adrenal insufficiency .
Low FSH, high LH This pattern is not seen in hypopituitarism — it would suggest some form of partial gonadotropin dysregulation , not pan-hypopituitarism.
When the body is in an acidotic crisis due to excess ketones, what test tells you how urgently you need to correct the blood’s pH?
127 / 232
Tags:
2023
A patient presented in the emergency department (ER) with diabetic ketoacidosis. What is the immediate investigation performed for the management?
In diabetic ketoacidosis (DKA) , the most immediate concern is the patient’s acid-base balance and electrolyte status — not long-term glucose control.
Arterial Blood Gases (ABGs) provide rapid and crucial information on:
This guides fluid resuscitation , insulin therapy , and electrolyte replacement , especially potassium .
❌ Why the Other Options Are Incorrect Initially
CBC May be useful later to rule out infection, but not urgent for DKA management.
UCE (Urea, Creatinine, Electrolytes) Important but not as immediate as ABG — typically ordered alongside ABG.
HbA1c Reflects long-term glucose control , not useful in acute settings.
Random Blood Sugar (RBS) Confirms hyperglycemia, but diagnosis of DKA also requires acidosis and ketones — ABG is essential.
When blood becomes “milky” due to very high fat levels, which organ—responsible for fat digestion—might be at risk of self-digestion?
128 / 232
Tags:
2023
Hypertriglyceridemia is associated with which of the following condition?
Hypertriglyceridemia is a well-known and important cause of acute pancreatitis , especially when triglyceride levels exceed 1000 mg/dL .
Mechanism:
Excess triglycerides are broken down by pancreatic lipase into free fatty acids .
These free fatty acids are toxic to pancreatic acinar cells , causing inflammation, necrosis , and pancreatitis .
Hypertriglyceridemia is the third most common cause of acute pancreatitis after gallstones and alcohol .
❌ Why the Other Options Are Incorrect
Diabetic foot Related to neuropathy, poor circulation, and infection — not directly caused by high triglycerides.
Diabetes type 1 Can be associated with ketoacidosis and other complications, but hypertriglyceridemia is more common in type 2 diabetes or secondary to insulin resistance .
Appendicitis Caused by obstruction of the appendix , not related to lipid metabolism.
Retinopathy Complication of long-standing hyperglycemia , not directly linked to triglyceride levels.
When cortisol is high and ACTH is low, where in the endocrine system does the problem most likely originate — upstream or downstream?
129 / 232
Tags:
2023
A 51-year-old man has noted increasing weakness and weight gain over the past 5 months. He has experienced low back pain for the past week. On physical examination, vital signs include a temperature of 37.3°C, pulse of 80/min, respirations of 15/min, and blood pressure of 155/95 mm Hg. He has bilateral breast enlargement, testicular atrophy, and a prominent fat pad in the posterior neck and back. His serum ACTH level is low. A radiograph of the spine shows decreased bone density with a compression fracture at T9, Which of the following findings is most likely to be present in this patient?
This patient has classic signs of Cushing syndrome , including:
Weight gain , central obesity , buffalo hump
Hypertension
Proximal muscle weakness
Osteoporosis with vertebral fracture
Gynecomastia and testicular atrophy (due to hormonal imbalance)
Critically, his serum ACTH is low , which indicates that his Cushing syndrome is ACTH-independent — the source of excess cortisol is not the pituitary or ectopic ACTH , but from the adrenal gland itself .
The most likely cause is a cortisol-secreting adrenal adenoma or carcinoma , which:
A 10-cm adrenal mass is concerning for adrenal carcinoma , which can be functional and cause severe Cushing syndrome .
❌ Why the Other Options Are Incorrect
Decreased radionuclide uptake in a thyroid gland nodule Related to thyroid disorders like a cold nodule; not related to cortisol or ACTH.
Pulmonary 6-cm right hilar mass on chest radiograph Suggestive of small cell carcinoma producing ectopic ACTH — would cause high ACTH , not low.
Sella turcica enlargement with erosion on head CT scan Seen in pituitary adenomas (Cushing disease) — which are ACTH-dependent and would show elevated ACTH , not low.
Retroperitoneal 5-cm mass at the aortic bifurcation Suggestive of paraganglioma or lymphoma , not associated with cortisol production or Cushing syndrome.
Consider the adrenal glands’ role in maintaining blood pressure, electrolyte balance, and energy metabolism . When these glands fail, patients may present with hypotension, weight loss, and fatigue . Could the flank tenderness be related to the adrenal glands , which sit just above the kidneys?
130 / 232
Tags:
2017
A 16-year-old girl came to the clinic with the complaints of lethargy and weight loss since six months. She also has hypotension and altered mentation. She also has mild flank tenderness. What is the probable diagnosis?
What is Addison’s Disease?
Addison’s disease is primary adrenal insufficiency —the adrenal glands fail to produce cortisol and aldosterone .
Common causes include autoimmune destruction , infections (e.g., TB), or adrenal hemorrhage .
Key Clinical Features in This Case:
Symptom
Explanation
Lethargy, fatigue
Due to low cortisol levels
Weight loss
Due to catabolic state and anorexia
Hypotension
Due to aldosterone deficiency → salt loss → volume depletion
Altered mentation
Due to hypoglycemia or electrolyte imbalances (e.g., hyponatremia, hyperkalemia)
Flank tenderness
May be from enlarged or inflamed adrenal glands (e.g., TB, hemorrhage), though usually mild
Lab Findings Typically Seen:
Why the Other Options Are Incorrect:
Option
Why It’s Incorrect
Pyelonephritis
❌ Typically causes fever, urinary symptoms, and severe flank pain , not chronic weight loss, hypotension, or altered mentation .
Phaeochromocytoma
❌ Causes hypertension , not hypotension . Symptoms include headache, palpitations, and sweating , not weight loss with low BP.
Hypothyroidism
❌ Causes weight gain , bradycardia , and cold intolerance , not weight loss and hypotension .
Diabetic ketoacidosis (DKA)
❌ DKA presents acutely with polyuria, vomiting, abdominal pain, Kussmaul breathing , and high blood sugar —not chronic lethargy and weight loss.
Summary:
A 16-year-old girl with chronic weight loss, fatigue, hypotension, altered mental status, and flank tenderness most likely has Addison’s disease (primary adrenal insufficiency) .
After brain injury, which hormone—essential for concentrating urine and preserving body water —might be deficient, leading to dilute urine despite dehydration?
131 / 232
Tags:
2023
A 42-year-old man sees his physician because he has had polyuria and polydipsia for the last 4 months. His medical history shows that he fell off a ladder and hit his head just before the onset of these problems. On physical examination, there are no specific findings. Laboratory findings include serum Na+, 155 mmol/L; K+, 3.9 mmol/L; Cl-, 111 mmol/L; C02,27 mmol/L; glucose 84 mg/dL; creatinine, 1 mg/dL; and osmolality, 350 mOsm/mL. The specific gravity of urine is 1.002. This patient is most likely to have a deficiency of which of the following hormones?
This patient presents with:
Polyuria and polydipsia
History of head trauma
Hypernatremia (Na⁺ 155 mmol/L)
High serum osmolality (350 mOsm/kg)
Very dilute urine (specific gravity 1.002)
These findings strongly indicate diabetes insipidus (DI) — specifically central diabetes insipidus , which occurs due to deficiency of antidiuretic hormone (ADH) , often after head trauma or hypothalamic/pituitary injury.
ADH (vasopressin) is secreted by the posterior pituitary and acts on the collecting ducts of the kidney to promote water reabsorption . Without it, water is lost in urine, leading to:
❌ Why the Other Options Are Incorrect
Melatonin Regulates sleep-wake cycles; has no role in fluid balance.
Corticotrophin (ACTH) Regulates cortisol production — ACTH deficiency would cause hypotension, hyponatremia , not hypernatremia or dilute urine.
Prolactin Affects lactation; unrelated to sodium or water balance.
Oxytocin Involved in uterine contractions and milk let-down ; not related to polyuria or osmolality.
If a hormone from the anterior pituitary suppresses the reproductive axis, what essential life function is likely to be disrupted?
132 / 232
Tags:
2023
A 25-year-old woman has noted breast secretions for the last 1 month. She is not breastfeeding and has never been pregnant. She has not menstruated for the past 5 months. Physical examination yields no abnormal findings. MRI of the brain shows a 0.7-cm mass in the adenohypophysis. Which of the following additional complications is most likely to be present in this patient?
This woman presents with:
Galactorrhea (breast secretions)
Amenorrhea (5 months without menstruation)
A 0.7 cm pituitary mass (microadenoma)
These findings strongly suggest a prolactinoma , a prolactin-secreting pituitary adenoma .
Prolactin inhibits GnRH (gonadotropin-releasing hormone), which decreases LH and FSH secretion, leading to:
So, infertility is the most likely additional complication.
❌ Why the Other Options Are Incorrect
Hyperthyroidism Would present with weight loss, heat intolerance, tremors, tachycardia — none are seen here. Also, prolactin is not elevated in hyperthyroidism.
Acromegaly Caused by GH-secreting pituitary adenoma , not prolactin. Presents with enlarged hands, coarse facial features , etc.
SIADH (Syndrome of Inappropriate Antidiuretic Hormone Secretion) Leads to hyponatremia and water retention , not galactorrhea or amenorrhea. It’s usually associated with hypothalamic or ectopic ADH secretion, not pituitary adenoma.
Cushing’s disease Caused by ACTH-secreting pituitary adenoma , resulting in hypercortisolism — features like central obesity, striae, moon face would be present.
Think about syndromes that involve tumors in multiple endocrine glands . One of them is known as the “3 Ps ” syndrome because it typically affects the Parathyroid, Pancreas, and Pituitary glands . Could this explain a combination of hypoglycemia, kidney stones, and hormonal dysfunction ?
133 / 232
Tags:
2017
A young male came to the clinic with hypoglycemia and loose stools. He also has renal stones and erectile dysfunction. He does not have diabetes. What is the most likely diagnosis?
What is MEN1?
Multiple Endocrine Neoplasia Type 1 (MEN1) is a hereditary syndrome characterized by tumors in:
Organ
Common Tumor
Parathyroid glands
Parathyroid hyperplasia/adenoma → Hypercalcemia → Renal stones
Pancreas (or GI tract)
Gastrinomas, Insulinomas, VIPomas → Hypoglycemia, diarrhea/loose stools
Pituitary gland
Pituitary adenomas → May cause prolactinoma → erectile dysfunction
Symptoms in This Case:
Hypoglycemia → Suggests insulinoma (pancreatic islet cell tumor)
Loose stools → May be due to VIPoma or gastrinoma (also seen in MEN1)
Renal stones → Due to hyperparathyroidism causing hypercalcemia
Erectile dysfunction → Could be from pituitary tumor (prolactinoma) affecting gonadal function
Why the Other Options Are Incorrect:
Option
Why It’s Incorrect
MEN2 (MEN2A/MEN2B)
❌ Involves medullary thyroid carcinoma, pheochromocytoma, and parathyroid hyperplasia (MEN2A). Hypoglycemia and diarrhea are not typical .
MEN3 (MEN2B)
❌ Characterized by mucosal neuromas, marfanoid habitus, pheochromocytoma, and medullary thyroid cancer . No hypoglycemia or renal stones.
Schmidt’s syndrome
❌ This is autoimmune polyglandular syndrome type 2 , involving Addison’s disease, thyroid disease, and type 1 diabetes , not endocrine tumors.
Autoimmune polyendocrinopathy type 1
❌ Involves Addison’s disease, hypoparathyroidism, and mucocutaneous candidiasis , not tumors or hyperparathyroidism.
Summary:
The combination of hypoglycemia, diarrhea, renal stones, and erectile dysfunction is characteristic of MEN1 , which involves tumors of the parathyroid, pancreas, and pituitary .
If the adrenal glands have been underused for years due to external steroid therapy, what would happen to their structure over time?
134 / 232
Tags:
2023
A 32-year-old woman with systemic lupus erythematosus has been treated with corticosteroid therapy for several years, because of recurrent lupus nephritis. While on vacation, she undergoes an emergency appendectomy for acute appendicitis. On postoperative day 2, she becomes somnolent and develops severe nausea and vomiting. She then becomes hypotensive. Blood cultures are negative, and laboratory studies now show Na+ of 128 mmol/L, K+ of 4.9 mmol/L, Cl- of 89 mmol/L, CO2 of 19 mmol/L, glucose of 52 mg/dL, and creatinine of 1.3 mg/dL. Which of the following morphologic findings in the adrenal glands is most likely to be present in this patient?
This patient with SLE on long-term corticosteroid therapy undergoes surgery and develops acute adrenal crisis postoperatively — presenting with:
Hypotension
Hyponatremia (Na⁺ 128)
Hypoglycemia (glucose 52 mg/dL)
Low CO₂ (metabolic acidosis)
Normal-to-high K⁺ (4.9 mmol/L)
These are classic signs of acute adrenal insufficiency .
Long-term exogenous corticosteroids suppress ACTH secretion through negative feedback. This leads to adrenal cortical atrophy over time. In a stressful situation like surgery, the atrophied adrenals can’t mount a cortisol response , resulting in acute adrenal crisis .
❌ Why the Other Options Are Incorrect
Bilateral cortical nodular hyperplasia Seen in ACTH-dependent Cushing disease (e.g., pituitary adenoma), not in steroid-suppressed patients.
Solitary 1-cm adenoma without contralateral atrophy Suggests nonfunctioning or mildly active tumor — not relevant here. Adenomas that produce cortisol usually suppress the other adrenal (causing unilateral atrophy), not both.
Bilateral hemorrhagic necrosis Characteristic of Waterhouse-Friderichsen syndrome — sudden adrenal failure due to sepsis (e.g., meningococcemia), which this patient does not have.
Bilateral caseating granulomas Found in tuberculosis — a chronic cause of adrenal insufficiency, not acute post-op failure in a steroid-treated SLE patient.
When evaluating thyroid conditions, pay close attention to the onset and nature of the goiter (painful vs. painless, acute vs. gradual, diffuse vs. nodular) and the patterns of TSH and thyroid hormone levels (T3, T4) .
135 / 232
Think of a fast-moving, deadly bacteremia in young, otherwise healthy individuals — especially one known for its link to meningitis and adrenal failure.
136 / 232
Tags:
2023
A 33-year-old, previously healthy woman dies suddenly after complaining of a mild sore throat the previous day, At autopsy, her adrenal glands are enlarged and there are extensive bilateral cortical hemorrhages. Which of the following organisms is responsible for the death of the patient?
This patient’s sudden death following a mild illness, combined with bilateral adrenal cortical hemorrhage , is characteristic of Waterhouse-Friderichsen syndrome — a rare but fatal complication of septicemia .
The most common cause of Waterhouse-Friderichsen syndrome is Neisseria meningitidis , a gram-negative diplococcus that can cause:
The presence of enlarged, hemorrhagic adrenal glands bilaterally at autopsy is a pathognomonic finding.
❌ Why the Other Options Are Incorrect
Cytomegalovirus Can cause adrenal involvement in immunocompromised patients , but not sudden death with bilateral adrenal hemorrhage.
Histoplasma capsulatum Fungal infection that can involve adrenals chronically , but not associated with sudden hemorrhage.
Mycobacterium tuberculosis Causes chronic adrenal destruction (Addison’s disease), not acute hemorrhage or sudden death.
Streptococcus pneumoniae Can cause meningitis , but rarely leads to Waterhouse-Friderichsen syndrome or adrenal hemorrhage.
If calcium is high, the body should naturally suppress the hormone that raises calcium. If that hormone is still elevated, where might the problem lie?
137 / 232
Tags:
2023
A 40-year-old man visits the physician due to weakness and easy fatiguability for 2 months. On physical examination, there are no remarkable findings. Laboratory studies show serum calcium of 11.5 mg/dl, inorganic phosphorus of 2.4 mg/dl, and parathyroid hormone of 54 pg/ml, which is near the top of the reference range. Radionuclear bone scan fails to show any uptake. What is the most likely cause of these findings?
This patient presents with:
Elevated serum calcium (11.5 mg/dL)
Low-normal phosphate (2.4 mg/dL)
High-normal PTH (54 pg/mL)
These findings are classic for primary hyperparathyroidism , where PTH is inappropriately elevated for the level of calcium . Normally, high calcium should suppress PTH — so a “normal-high” PTH in the context of hypercalcemia is abnormal .
The most common cause of primary hyperparathyroidism is a parathyroid adenoma — a benign tumor of one parathyroid gland.
The radionuclear bone scan not showing uptake also suggests no widespread skeletal disease or metastatic cancer.
❌ Why the Other Options Are Incorrect
Chronic renal failure Causes secondary hyperparathyroidism , with low calcium , high phosphate , and very high PTH due to phosphate retention and vitamin D deficiency.
Hypervitaminosis D Causes hypercalcemia , but with low PTH due to feedback suppression. Also tends to cause high phosphate , not low.
Parathyroid hyperplasia A cause of primary hyperparathyroidism, but usually all four glands are enlarged. It’s less common than adenoma and often part of MEN syndromes .
Parathyroid carcinoma Rare; usually presents with severe hypercalcemia , very high PTH , and more aggressive symptoms like bone pain, fractures, and renal damage .
Think about what happens when poor blood flow , nerve damage , and high blood sugar all affect the same limb for years — what’s the worst outcome?
138 / 232
Tags:
2023
A 52-year-old man has been concerned about a gradual weight gain for many years. He is 174cm (5 ft 7 inches) tall and weighs 91 kg (body mass index 30 kg/m^2). He is taking no medications. On physical examination, he has decreased sensation to pinprick and light touch over the lower extremities. Laboratory studies show glucose of 169 mg/dL, creatinine of 1.9 mg/dL, total cholesterol of 220 mg/dL, HDL cholesterol of 27 mg/dL, and triglycerides of 261 mg/dL. A chest radiograph shows mild cardiomegaly. Five years later, he has claudication in the lower extremities when he exercises. Based on these findings, which of the following complications in lower limbs can occur in this patient?
This patient has long-standing type 2 diabetes mellitus with multiple associated complications:
Obesity (BMI = 30)
Hyperglycemia (glucose 169 mg/dL)
Diabetic nephropathy (elevated creatinine)
Peripheral neuropathy (reduced pinprick/light touch sensation)
Dyslipidemia (high triglycerides, low HDL)
Cardiomegaly , indicating possible diabetic cardiomyopathy
Now he has claudication — pain in the legs during exertion, which suggests peripheral artery disease (PAD)
Over time, PAD + neuropathy + poor glycemic control can lead to:
This is a classic and devastating complication of advanced diabetes .
❌ Why the Other Options Are Incorrect
Chronic pancreatitis More common in alcoholics and causes malabsorption and chronic pain , not linked directly to this diabetic progression.
Systemic amyloidosis Can occur in chronic inflammatory or plasma cell diseases — but this patient has no signs of such conditions.
Ketoacidosis More common in type 1 diabetes ; this patient likely has type 2 , and there’s no acute decompensation or metabolic acidosis mentioned.
Hypoglycemic coma This patient is not on medications , especially insulin or sulfonylureas , which are the main causes of hypoglycemia. His glucose is actually elevated.
If one adrenal is producing excessive hormone on its own, what would happen to the pituitary’s ACTH output — and how would the other adrenal gland respond?
139 / 232
Tags:
2023
A 30-year-old man visits the hospital because he has had a headache, weakness, and a 5-kg weight gain over the last 4 months. On physical examination, his face is puffy. His temperature is 36.9°C, pulse is 79/min, and blood pressure is 160/75 mm Hg while lying down. He has cutaneous striae over the lower abdomen and ecchymoses scattered over the extremities. A radiograph of the spine shows a compressed fracture of T11. Laboratory findings show a fasting plasma glucose level of 200 mg/dL, the plasma cortisol level is 38 pg/dL at 8:00 am and 37 pg/dL at 6:00 pm. Which of the following conditions is most likely to be present in this patient?
This patient presents with Cushing syndrome , suggested by:
Weight gain , puffy face , abdominal striae , hypertension , glucose intolerance/diabetes , muscle weakness , easy bruising , and osteoporotic fractures
Cortisol levels remain high throughout the day (normally, they decline by evening), indicating loss of diurnal variation — a hallmark of Cushing syndrome
To identify the cause , we differentiate between:
ACTH-dependent causes : Pituitary adenoma (Cushing disease), ectopic ACTH (e.g., small-cell lung cancer)
ACTH-independent causes : Adrenal adenoma, adrenal carcinoma, exogenous steroids
In this case:
Persistent high cortisol
Features consistent with Cushing syndrome
No mention of elevated ACTH , and symptoms are not rapidly progressive or malignant-appearing (as in ectopic ACTH)
An adrenal adenoma secreting cortisol would:
❌ Why the Other Options Are Incorrect
Adenoma of the right adrenal cortex without atrophy of contralateral adrenal cortex This would not happen — ACTH suppression causes atrophy in the non-functioning adrenal .
Corticotroph adenoma of the anterior pituitary, medullary carcinoma of thyroid, and bilateral nodular hyperplasia of the adrenal cortex Medullary thyroid carcinoma is associated with MEN syndromes , not Cushing syndrome.
Corticotroph adenoma with bilateral hyperplasia Seen in Cushing disease , but would usually have elevated ACTH , not suppressed levels.
Small-cell carcinoma of lung with bilateral hyperplasia Ectopic ACTH production leads to rapid onset, severe symptoms , and marked ACTH elevation — not consistent with this patient’s subacute presentation.
This condition causes the thyroid to work overtime because it’s being stimulated, not destroyed . Think about how that might change the structure of the follicles and the immune presence around them.
140 / 232
Tags:
2023
A 40-year-old man comes into the outpatient department because of weight loss, increased appetite, and diplopia. On physical examination, his temperature is 37.7° C, pulse is 106/min, respiration rate is 15/min, and blood pressure is 140/80 mmHg. A fine tremor is observed in his outstretched hand. He has bilateral proptosis. The serum laboratory findings include a serum TSH level of 0.1 pU/mL. A radioiodine scan indicates increased diffuse uptake throughout the thyroid, Which of the following best describes the microscopic appearance of the patient’s thyroid gland?
This patient presents with classic signs of hyperthyroidism :
Weight loss , heat intolerance , increased appetite , tremor , tachycardia , and proptosis (exophthalmos)
His TSH is suppressed , and radioiodine uptake is diffusely increased , indicating Graves disease
Graves disease is an autoimmune hyperthyroidism caused by TSH receptor–stimulating antibodies (TRAb) . These antibodies stimulate the thyroid diffusely, causing:
Hyperplasia of follicular epithelium
Scalloped colloid due to increased uptake
Papillary infoldings into follicular lumen
Lymphoid aggregates in the stroma due to the autoimmune nature
❌ Why the Other Options Are Incorrect
Nodules with nests of cells separated by hyaline stroma that stains with Congo red → Classic for medullary thyroid carcinoma with amyloid deposition , not Graves disease.
Destruction of follicles, lymphoid aggregates, and Hurthle cell metaplasia → Seen in Hashimoto thyroiditis , a hypothyroid condition, not hyperthyroidism.
Follicular destruction with inflammatory infiltrates containing giant cells → Suggestive of subacute granulomatous thyroiditis (De Quervain) — painful thyroid, often after viral illness.
Enlarged thyroid follicles lined by flattened epithelial cells → Seen in colloid goiter , a non-toxic, euthyroid condition — not associated with autoimmune hyperthyroidism.
Think about a hormone that plays a key role in neural development , muscle tone , and gut motility — and whose deficiency in newborns can cause silent but dangerous developmental delays.
141 / 232
Tags:
2023
The emergency room receives a 2-month-old boy with worsening “floppiness” and poor eating. The baby was born vaginally to a 38-year-old lady. The parents call the baby a “good baby” who rarely cries and sleeps through the night but has been hard to wake for breastfeeding. Stools are now tiny and pellet-like. Examination reveals a hypotonic child with a big tongue, anterior fontanelle, and reducible umbilical hernia. Which of the following conditions is the infant suffering from?
This infant presents with floppiness , poor feeding , constipation , hypotonia , macroglossia , umbilical hernia , and lethargy — all classic signs of congenital hypothyroidism .
Thyroid hormone is essential for brain development , muscle tone , and metabolism . Deficiency in infancy causes:
Lethargy , minimal crying (“good baby”)
Hypotonia (“floppy baby”)
Constipation (tiny pellet stools)
Macroglossia (big tongue)
Delayed milestones , umbilical hernia
Prolonged jaundice (often present earlier)
Early diagnosis is critical , as untreated congenital hypothyroidism leads to irreversible intellectual disability . That’s why newborn screening is done universally in many countries.
❌ Why the Other Options Are Incorrect
Down syndrome May have hypotonia and umbilical hernia, but wouldn’t explain constipation, sleepiness, macroglossia, or feeding difficulty at this level. Also, there’s no mention of dysmorphic facial features , simian crease, or poor Moro reflex.
Addison’s disease Rare in infants and presents with hyperpigmentation, vomiting, low sodium, low BP — not a “good baby” or macroglossia.
Botulism Causes floppy baby syndrome, but typically in the U.S. it’s associated with honey exposure or contaminated food. The onset is acute , not slow and progressive, and doesn’t cause macroglossia or hernia .
Galactosemia Presents in newborns with vomiting, jaundice, hepatomegaly , and E. coli sepsis — not hypotonia with macroglossia and hernia.
Consider the effects of severe postpartum hemorrhage on the pituitary gland , which is highly vascular and vulnerable to ischemia after blood loss . Why would a woman stop menstruating and be unable to breastfeed after delivery?
142 / 232
Tags:
2017
A 28-year-old woman who gave birth recently came to the outpatient department with the complaints of amenorrhea and inability to breastfeed her child. She gives a past history of postpartum haemorrhage. What is your diagnosis based on her complaints?
Sheehan’s syndrome is postpartum hypopituitarism caused by ischemic necrosis of the anterior pituitary due to severe blood loss during or after childbirth (postpartum hemorrhage).
The anterior pituitary enlarges during pregnancy , making it more susceptible to ischemia in the event of hypotension or hemorrhage .
Clinical Features:
Failure to lactate (due to prolactin deficiency )
Amenorrhea or oligomenorrhea (due to gonadotropin deficiency → low LH/FSH )
Fatigue, weakness, cold intolerance (due to TSH and ACTH deficiency )
Loss of pubic and axillary hair (optional finding)
Why the Other Options Are Incorrect:
Option
Why It’s Incorrect
Adrenal insufficiency
May cause fatigue, but would not explain inability to breastfeed or amenorrhea after postpartum hemorrhage directly.
Prolactinoma
Causes galactorrhea , not inability to lactate. Prolactin would be high , not low.
Hypothyroidism
Can cause amenorrhea , but not typically failure of lactation immediately postpartum due to pituitary damage.
Hyperparathyroidism
Involves calcium regulation , unrelated to lactation or menstrual cycles.
Summary:
A postpartum woman with amenorrhea, inability to lactate, and a history of postpartum hemorrhage is classic for Sheehan’s syndrome (postpartum pituitary necrosis) .
Consider which common endocrine disorder in young women presents with menstrual irregularities, hirsutism, and weight gain . It is also frequently associated with insulin resistance and metabolic syndrome . What condition fits this triad?
143 / 232
Tags:
2017
A 21-year-old unmarried girl presented to the gynecology outpatient department with complaints of oligomenorrhea, excessive hair growth, and significant weight gain for the past year. What is the most probable diagnosis based on the presenting complaints?
Why PCOS is the Most Probable Diagnosis:
Polycystic Ovarian Syndrome (PCOS) is the most common endocrine disorder in reproductive-aged women , typically presenting with:
Oligomenorrhea or amenorrhea (due to anovulation)
Hirsutism (excess hair growth) from androgen excess
Weight gain/obesity , often linked to insulin resistance
Polycystic ovaries seen on ultrasound (multiple peripheral follicles)
Pathophysiology:
Increased LH:FSH ratio , leading to increased androgen production by the ovaries
Impaired follicle maturation → anovulation
Insulin resistance worsens hormonal imbalance
Why the Other Options Are Incorrect:
Option
Why It’s Incorrect
Acromegaly
Presents with coarse facial features, enlarged hands/feet, and jaw growth , not primarily with menstrual issues and hirsutism .
Hypothyroidism
Can cause weight gain and menstrual problems , but hirsutism is not a typical feature .
Hyperprolactinemia
Usually presents with amenorrhea and galactorrhea , not hirsutism or weight gain .
Congenital adrenal hyperplasia (CAH)
CAH can cause hirsutism and menstrual irregularities , but usually presents earlier in life and often with ambiguous genitalia or salt-wasting in severe forms .
Summary:
The combination of oligomenorrhea, hirsutism, and weight gain in a young woman is most consistent with Polycystic Ovarian Syndrome (PCOS).
Consider the feedback loop between PTH and calcium . Normally, high calcium levels suppress PTH secretion . If PTH remains elevated despite high calcium , where is the problem likely located—in the parathyroid glands themselves , or in a compensatory response to another condition?
144 / 232
Tags:
2017
High PTH, high plasma calcium, and normal levels of creatinine and vitamin D are consistent with which of the following parathyroid states?
Understanding the Lab Findings:
Parameter
Finding
PTH (Parathyroid Hormone)
High
Plasma Calcium
High
Creatinine
Normal
Vitamin D
Normal
Primary Hyperparathyroidism:
Caused by parathyroid adenoma (most common) , parathyroid hyperplasia , or rarely parathyroid carcinoma .
In this condition, the parathyroid glands secrete PTH autonomously , ignoring normal feedback mechanisms .
High PTH leads to:
Increased calcium reabsorption from bone and kidney
Increased activation of Vitamin D (but normal Vitamin D here suggests early stage or regulated synthesis)
Increased intestinal absorption of calcium (via Vitamin D)
Result: High calcium, high PTH , and normal renal function (normal creatinine)
Why the Other Options Are Incorrect:
Option
Explanation
Primary hypoparathyroidism
❌ Would cause low PTH and low calcium , not high levels.
Secondary hyperparathyroidism
❌ PTH is high, but calcium is usually low or normal , due to chronic conditions like renal failure or vitamin D deficiency .
Tertiary hyperparathyroidism
❌ Occurs in chronic renal failure , with high PTH and high calcium , but typically with elevated creatinine (renal dysfunction), which is normal here .
Secondary hypoparathyroidism
❌ Would cause low PTH due to external suppression (e.g., surgery, radiation), not high PTH.
Summary:
High PTH + High Calcium + Normal Creatinine and Vitamin D is classic for Primary Hyperparathyroidism .
Think about what happens when the body is under severe stress to produce more red blood cells , especially in hemolytic diseases . Does the bone marrow have time to let all the cells mature fully before releasing them, or does it push out immature forms ?
145 / 232
Tags:
2017
What is released from the bone marrow in excess amounts in erythroblastosis fetalis?
Erythroblastosis fetalis (also known as hemolytic disease of the newborn ) occurs when the maternal immune system destroys fetal red blood cells , often due to Rh incompatibility .
The fetal body responds by overproducing red blood cell precursors to replace the destroyed cells.
Bone Marrow Response:
Normally, the bone marrow releases mature red blood cells (RBCs) .
In erythroblastosis fetalis , there is massive hemolysis , so the bone marrow becomes hyperactive .
It starts to release immature red blood cell precursors (blast cells) , specifically erythroblasts , into the bloodstream.
This is why the condition is called erythroblastosis (“erythro-” = red blood cells, “-blastosis” = immature cell proliferation).
Why the Other Options Are Incorrect:
Option
Explanation
Red blood cells
Mature RBCs are normally released , but in erythroblastosis fetalis, immature forms (blasts) are released in excess .
Megakaryocytes
These are platelet precursors , unrelated to erythrocyte production.
Lymphocytes
White blood cells , not involved in the compensatory response to hemolysis.
Macrophages
Part of the immune system , not directly produced in response to anemia or hemolysis.
Summary:
In erythroblastosis fetalis, the bone marrow releases blast cells (erythroblasts) into the circulation in excess to compensate for severe red cell destruction.
Consider which anterior pituitary cell type is responsible for producing prolactin . A prolactinoma is a type of pituitary adenoma , not a tumor of the target organ. Which specific pituitary cells secrete prolactin?
146 / 232
Tags:
2017
Prolactinoma is a tumor of which of the following?
A prolactinoma is the most common type of pituitary adenoma .
It arises from mammotropes (also called lactotrophs) , the prolactin-secreting cells of the anterior pituitary .
Effects of a Prolactinoma:
Hyperprolactinemia → leads to:
Galactorrhea (inappropriate milk production)
Amenorrhea (in women)
Infertility (in both sexes)
Hypogonadism (due to prolactin’s inhibitory effect on GnRH)
Mass effect symptoms if large (headache, visual field defects from optic chiasm compression)
Why the Other Options Are Incorrect:
Option
Reason
Gonadotropes
Secrete LH and FSH , not prolactin.
Breast
The breast is a target organ of prolactin, but a prolactinoma originates in the pituitary , not in breast tissue.
Hypothalamus
The hypothalamus regulates prolactin via dopamine (inhibitory control) , but prolactin is produced by the anterior pituitary .
Somatotropes
Secrete growth hormone (GH) , not prolactin.
Summary:
A prolactinoma is a tumor of the mammotropes (lactotrophs) in the anterior pituitary , which produce prolactin .
Consider the timing of the hemorrhage in relation to childbirth. Does Sheehan syndrome typically occur before delivery , or is it more commonly related to events that happen during or after childbirth ?
147 / 232
Tags:
2017
Which of the following is wrong regarding Sheehan syndrome?
Sheehan syndrome is a condition resulting from ischemic necrosis of the anterior pituitary , typically following severe postpartum hemorrhage (PPH) .
Pathophysiology:
During pregnancy, the anterior pituitary enlarges , mainly due to increased prolactin-secreting cells to prepare for lactation .
Despite the enlargement, the blood supply does not increase proportionally , making the pituitary vulnerable to ischemia .
If a woman experiences massive postpartum hemorrhage , the sudden drop in blood pressure can cause hypoperfusion of the anterior pituitary , leading to necrosis and hypopituitarism .
Key Clinical Features:
Failure to lactate (due to prolactin deficiency)
Amenorrhea or oligomenorrhea (due to gonadotropin deficiency)
Hypothyroidism and adrenal insufficiency may also develop over time due to TSH and ACTH deficiencies .
Why the Other Options Are Correct (True Statements):
Option
Explanation
It is a result of ischemic necrosis of pituitary
✅ True. This is the fundamental cause of Sheehan syndrome.
It is associated with infarction of the anterior lobe
✅ True. The anterior pituitary is mainly affected; the posterior pituitary is usually spared because it has a direct arterial blood supply.
It is associated with postpartum hemorrhage
✅ True. Massive postpartum hemorrhage is the classic trigger .
During pregnancy anterior pituitary enlarges to twice its size
✅ True. The anterior pituitary enlarges, mostly due to prolactin cell hyperplasia , increasing its vulnerability to ischemia.
Why “It is associated with antepartum hemorrhage” is Incorrect:
Antepartum hemorrhage occurs before delivery (e.g., placenta previa, placental abruption).
**Sheehan syndrome is specifically associated with postpartum hemorrhage , meaning bleeding after delivery .
Therefore, the statement about antepartum hemorrhage is wrong .
Think adrenal destruction due to chronic infection.
148 / 232
Tags:
2017
A person with pulmonary tuberculosis undergoing treatment for 3 months is likely to develop which pathology?
Pulmonary tuberculosis (TB) can spread to the adrenal glands via hematogenous routes, especially in long-standing or poorly managed cases. Over time, this can destroy adrenal tissue , leading to primary adrenal insufficiency , also known as Addison disease . Patients may develop symptoms like hypotension, fatigue, hyponatremia, and hyperpigmentation due to increased ACTH production. TB used to be a leading cause of Addison disease worldwide, especially in developing countries.
Answer breakdown:
✅ Addison disease – Correct; due to TB destroying adrenal cortex
Cushing syndrome – Incorrect; excess cortisol, not linked to TB
Conn syndrome – Incorrect; primary hyperaldosteronism
Hypertension – Incorrect; Addison disease causes hypo tension
Cushing disease – Incorrect; pituitary ACTH excess, not related to TB
This syndrome results from excess cortisol — think of what cortisol increases and stores .
149 / 232
Tags:
2017
Which of the following is not seen in Cushing’s syndrome?
Cushing’s syndrome is caused by prolonged exposure to high levels of cortisol , whether endogenous (e.g. ACTH-producing tumor) or exogenous (e.g. long-term steroid therapy). Cortisol promotes gluconeogenesis, lipogenesis, and protein breakdown , leading to classic features like:
Central obesity and weight gain
Hyperglycemia and insulin resistance
Muscle wasting and thin limbs
Purple abdominal striae from skin thinning
Hirsutism due to excess adrenal androgensWeight loss is not typical in Cushing’s — the opposite occurs due to fat accumulation and fluid retention.
Answer Breakdown:
Hirsutism → Common due to androgen excess
Weight gain → Classic symptom
Hyperglycemia → Due to cortisol’s metabolic effects
Purple striae → Due to collagen breakdown and skin thinning
Weight loss ✅ → Not seen; thus the correct answer
In a condition where the thyroid is overactive , producing excess hormone without the pituitary’s instruction , what happens to the hormone that normally tells the thyroid what to do?
150 / 232
Tags:
2017
What is the test for Graves disease?
Graves disease is the most common cause of hyperthyroidism . It’s an autoimmune disorder in which the body produces TSH receptor antibodies (TRAb) . These antibodies mimic TSH , stimulating the thyroid gland to secrete excess T3 and T4 , even though TSH from the pituitary is not needed .
🧪 What happens to TSH levels?
This makes TSH concentration the best initial test when evaluating a patient with suspected thyroid dysfunction — both hypo- and hyperthyroidism .
If TSH is low , and symptoms point toward hyperthyroidism, the next steps may include:
🔍 Answer Breakdown:
To diagnose a condition characterized by excess of a hormone, would it make sense to check just one momentary value, or would a test that captures hormone production over a greater amount of time give a more reliable picture?
151 / 232
Tags:
2017
What is the best diagnostic test for Cushing syndrome?
Cushing syndrome is a clinical condition resulting from chronic exposure to excessive cortisol . It can be:
🎯 Diagnosing Cushing Syndrome: Why a 24-hour urine cortisol test?
Because cortisol levels fluctuate throughout the day — peaking in the early morning and dropping by night — a single blood or saliva measurement may miss the diagnosis. That’s why a test that measures total cortisol output over time is more reliable for initial screening .
The 24-hour urinary free cortisol test reflects the unbound, biologically active cortisol excreted in the urine. In Cushing syndrome , levels will be consistently elevated across the collection period.
This test is:
Non-invasive
Not affected by diurnal variation
Recommended as a first-line screening test by clinical guidelines (e.g., Endocrine Society)
🩺 What comes next?
Once Cushing syndrome is confirmed, further tests (e.g., ACTH levels, dexamethasone suppression tests, CRH stimulation) are used to determine the cause — whether it’s:
🔍 Answer Breakdown:
24-hour urine collection for free cortisol ✅
Adrenocorticotropic hormone (ACTH) levels test ❌
Serum cortisol levels test ❌
Cortisol is subject to diurnal variation . A one-time blood test may give a false-negative or false-positive .
Corticotropin-releasing hormone (CRH) stimulation test ❌
A secondary test , used to distinguish between pituitary and ectopic ACTH production . Not used initially.
High-dose dexamethasone suppression test ❌
Also a second-line test used to determine cause , not to confirm presence of Cushing syndrome.
Which hormonal imbalance leads to a metabolic slowdown that affects nearly every organ system — from gut motility and reproductive cycles to energy expenditure and fat storage?
152 / 232
Tags:
2018
Which of the following conditions causes amenorrhea, obesity, and constipation?
We’re given three key symptoms :
Amenorrhea = absence of menstruation
Obesity = weight gain
Constipation = slow bowel movements
Let’s analyze which endocrine disorder most logically links all three.
🔎 Option-by-option Analysis:
Hypothyroidism — Correct Answer
Why?
Hypothyroidism causes a general slowing down of metabolic functions due to decreased levels of thyroxine (T4) and triiodothyronine (T3) .
✅ Amenorrhea :
✅ Obesity :
✅ Constipation :
💡 Additional signs often seen: cold intolerance , dry skin , bradycardia , fatigue , depression , coarse hair .
Hyperpituitarism
Typically causes overproduction of pituitary hormones (e.g., GH , ACTH , prolactin ).
Amenorrhea can result from hyperprolactinemia , but:
It doesn’t typically cause obesity or constipation
Often associated with galactorrhea , visual field defects , acromegaly , or Cushingoid features depending on the hormone involved
Hyperparathyroidism
Characterized by elevated PTH → hypercalcemia
Symptoms = “bones, stones, groans, and psychiatric overtones” :
Not associated with amenorrhea or obesity
Constipation may occur (due to hypercalcemia ), but it doesn’t explain the full triad
Hyperthyroidism
None of them
In a patient who is weak, irritable, hypotensive, and shows a stress-induced blood sugar spike, what underlying hormone — essential for maintaining vascular tone and energy balance — would you check first to assess the body’s ability to handle physiologic stress?
153 / 232
Tags:
2018
A lean lady comes to the clinic with the complaint of fragility and irritability. On examination, her blood pressure is 90/60 mmHg. Her laboratory results show blood sugar 240 mg/dL. What test would you first suggest to be done?
🔍 Clinical Clues:
Lean build and irritability → May point toward hyperthyroidism or adrenal insufficiency
BP 90/60 mmHg → Hypotension , which is red flag for Addison’s disease (primary adrenal insufficiency)
Blood sugar 240 mg/dL → Hyperglycemia , unusual in Addison’s (which usually causes hypoglycemia ), but can be seen in stress or thyroid storms
So what are the most likely differential diagnoses ?
📌 Consider:
Adrenal insufficiency (Addison’s disease):
Hypotension
Weakness/fragility
Electrolyte imbalance (low Na⁺, high K⁺)
Often hypoglycemia , not hyperglycemia — but early or partial AI may vary
Cortisol deficiency is hallmark
Hyperthyroidism/Thyroid storm :
Irritability, weakness
Weight loss
May cause hyperglycemia , tachycardia , hypertension , anxiety
However, this patient has hypotension , which does not support thyrotoxicosis
Why it’s correct:
Cortisol is the primary hormone to check when suspecting adrenal insufficiency .
A low morning cortisol strongly suggests Addison’s disease , especially when paired with hypotension and general fatigue .
It’s the first-line test before ACTH or electrolyte panels.
If cortisol is low, then follow-up with ACTH can help differentiate:
Primary AI (Addison’s): high ACTH, low cortisol
Secondary AI (pituitary problem): low ACTH, low cortisol
❌ Why the Other Options Are Incorrect First-line Tests:
ACTH
Helpful for confirming cause of adrenal insufficiency, but not the first test .
Start with cortisol , then ACTH if cortisol is abnormal.
T4 / T3
Thyroid function tests are useful if hyperthyroidism is suspected.
But the BP is low , not high.
Thyroid storm or hyperthyroidism would usually show hypertension, tachycardia, and fever .
Her presentation fits adrenal insufficiency more than thyrotoxicosis.
Electrolytes
Definitely useful in adrenal insufficiency (looking for hyponatremia , hyperkalemia )
But not diagnostic on their own.
Cortisol is a more specific and primary test .
When the body’s defense system mistakenly targets more than one hormone-producing organ, a syndrome arises — particularly if both stress hormones and metabolic regulators are affected. Think: what condition reflects an autoimmune “double hit” to two major endocrine players?
154 / 232
Tags:
2018
What is referred to as a combination of Hashimoto thyroiditis and Addison’s disease?
To understand this properly, we must explore Autoimmune Polyglandular Syndromes (APS) — a group of rare disorders where multiple endocrine glands are attacked by the immune system .
There are three main types :
🔹 APS Type I (Whitaker Syndrome)
➡️ This is not the correct answer because Hashimoto’s thyroiditis is not typically part of APS I .
🔹 APS Type II (Schmidt Syndrome)
Onset: Adulthood (especially in middle-aged females)
Genetic basis: Polygenic, often associated with HLA-DR3 and HLA-DR4
Key features:
Addison’s disease (always present)
Autoimmune thyroid disease – Hashimoto’s thyroiditis or Graves’ disease
May also include Type 1 diabetes mellitus , pernicious anemia , etc.
➡️ When Addison’s disease + Hashimoto’s thyroiditis are found together, it’s classically called Schmidt Syndrome , a subtype of APS Type II .
Why it’s correct:
❌ Why the Other Options Are Incorrect:
Autoimmune Polyglandular Syndrome Type I
Involves Addison’s disease but not Hashimoto’s .
Features mucocutaneous candidiasis and hypoparathyroidism , not thyroiditis.
None of them
Pheochromocytoma
Tumor of the adrenal medulla , not cortex.
Causes catecholamine excess , not adrenal insufficiency.
Not autoimmune .
Pituitary adenoma
Involves central (secondary) endocrine disorders (e.g., ACTH deficiency), not primary autoimmune gland destruction .
Has nothing to do with Hashimoto’s or Addison’s disease.
Consider what level of average blood sugar over a few months would reflect the tipping point from normal metabolism to a state where chronic complications begin to develop — not too high, but high enough to trigger concern.
155 / 232
Tags:
2018
What is the HbA1c cut-off for diabetes mellitus diagnosis?
🧬 What is HbA1c?
HbA1c (Glycated Hemoglobin) reflects the average blood glucose levels over the past 2–3 months .
It is formed when glucose binds irreversibly to hemoglobin in red blood cells.
The higher the blood glucose, the more hemoglobin becomes glycated.
🩺 Diagnostic Cut-off for Diabetes Mellitus:
According to the American Diabetes Association (ADA) and World Health Organization (WHO) :
Test
Diagnostic Threshold for Diabetes
HbA1c
≥ 6.5%
Fasting plasma glucose
≥ 126 mg/dL (7.0 mmol/L)
2-hour OGTT (75g)
≥ 200 mg/dL (11.1 mmol/L)
Random glucose (with symptoms)
≥ 200 mg/dL
So, an HbA1c level >6.5% confirms a diagnosis of diabetes mellitus , provided the test is performed using standardized methods.
❌ Why the Other Options Are Incorrect:
Think about which disorder not only alters hormone levels but also causes immune cells to attack soft tissues . Could the eye be a victim in such a systemic autoimmune attack?
156 / 232
Tags:
2017
Exophthalmos is most strongly associated with which of the following pathologies?
Exophthalmos , also known as proptosis , refers to the protrusion of the eyeball from the orbit. While multiple conditions may cause it, Graves disease is the most strongly associated due to its unique autoimmune mechanism targeting tissues around the eye.
✅ Why “Graves disease” is correct:
Graves disease is an autoimmune hyperthyroid condition .
The body produces thyroid-stimulating immunoglobulins (TSIs) which stimulate the TSH receptor .
But here’s the key: These antibodies also affect fibroblasts in the orbital tissue , stimulating them to produce glycosaminoglycans (GAGs).
This leads to:
Result: the eye is pushed forward — classic exophthalmos .
This is not just due to hyperthyroidism, but due to immune-mediated infiltration of orbital tissues , making Graves disease the only thyroid disorder with this specific ocular manifestation.
❌ Why the other options are incorrect:
Hypothyroidism : May cause periorbital puffiness due to myxedema, but does not cause true exophthalmos .
Conn syndrome : A condition of primary hyperaldosteronism ; presents with hypertension, hypokalemia , but no eye involvement .
Hypoparathyroidism : Leads to hypocalcemia , causing neuromuscular symptoms (e.g., tetany), not orbital symptoms .
Hyperparathyroidism : Causes hypercalcemia , with symptoms like bone pain , renal stones , and psychiatric disturbances , but not exophthalmos
Which anterior pituitary hormone, when overproduced, suppresses reproductive hormones and initiates lactation even in non-pregnant women?
157 / 232
Tags:
2018
A woman presents to the emergency with amenorrhea and milk secretion for the past 2 months. On examination, it was found that the patient has pituitary adenoma. Which of the following hormone, released by the tumor, is responsible for the patient’s symptoms?
This case describes classic symptoms of a prolactin-secreting pituitary adenoma , also known as a prolactinoma :
Amenorrhea (absence of menstruation): Prolactin inhibits gonadotropin-releasing hormone (GnRH) , reducing LH and FSH secretion, disrupting the menstrual cycle.
Galactorrhea (milk secretion): Prolactin directly stimulates milk production in the mammary glands.
Prolactinomas are the most common type of functioning pituitary adenomas and frequently affect women of reproductive age .
❌ Why the Other Options Are Incorrect:
A. LH: Important for ovulation, but excess LH does not cause galactorrhea or amenorrhea.
B. FSH: Stimulates follicular development, but excess FSH is not responsible for these symptoms.
D. FSH and LH: Together, they regulate the menstrual cycle, but their overproduction doesn’t cause galactorrhea.
E. Testosterone: Is not secreted by pituitary tumors and is not relevant to this symptom complex in females.
Which anterior pituitary tumor often presents with symptoms even when small, due to its hormone’s effect on reproductive function?
158 / 232
Tags:
2018
Which of the following is the most common type of pituitary adenoma?
A prolactinoma is the most common type of pituitary adenoma , accounting for about 40–50% of all pituitary tumors. It arises from lactotroph cells in the anterior pituitary and causes excess secretion of prolactin .
Clinical Features:
Women : Amenorrhea, galactorrhea, infertility
Men : Decreased libido, erectile dysfunction, gynecomastia (less common)
Both sexes may experience headaches and visual disturbances if the tumor compresses surrounding structures.
Prolactinomas are often diagnosed early due to hormonal symptoms and are usually benign and treatable , often with dopamine agonists like cabergoline or bromocriptine.
❌ Why the Other Options Are Incorrect:
B. GH cell adenoma (somatotroph adenoma): Causes acromegaly or gigantism, but is less common than prolactinomas.
C. FSH-producing adenoma: Rare and often clinically silent.
D. ACTH cell adenoma: Causes Cushing disease , but much less frequent .
E. LH-producing adenoma: Also rare and often non-functioning.
Think about which part of the pituitary has increased metabolic demand during pregnancy and is more vulnerable to hypoperfusion.
159 / 232
Tags:
2018
Which of the following is correct regarding Sheehan syndrome?
Sheehan syndrome is a form of hypopituitarism caused by ischemic necrosis of the pituitary gland , typically following severe postpartum hemorrhage . Here’s what happens:
During pregnancy, the anterior pituitary (adenohypophysis) enlarges due to increased demand for prolactin but does not receive a proportional increase in blood supply .
If there’s a significant drop in blood pressure (e.g., from obstetric hemorrhage), the already vulnerable anterior pituitary undergoes infarction .
This results in deficiency of anterior pituitary hormones , leading to symptoms like inability to lactate, amenorrhea, fatigue, and hypotension .
❌ Why the Other Options Are Incorrect:
B. None of them: Incorrect because A is correct .
C. Prepartum necrosis of adenohypophysis: Sheehan syndrome occurs postpartum , not prepartum.
D. During pregnancy posterior pituitary enlarges twice its size: It’s the anterior pituitary that enlarges, not the posterior.
E. Infarction of posterior lobe: Sheehan syndrome affects the anterior lobe , while the posterior pituitary is usually spared.
Consider how crucial thyroid hormones are in the early stages of brain and body development . A deficiency at that stage affects not just growth, but also cognitive and neurological outcomes.
160 / 232
Tags:
2018
Cretinism is the clinical condition resulting from untreated congenital or early-onset hypothyroidism , especially during infancy or early childhood .
It leads to impaired physical growth and severe mental retardation , and is preventable with early diagnosis and treatment.
🔬 Causes of Cretinism:
Congenital absence or dysgenesis of the thyroid gland
Inborn errors of thyroid hormone synthesis (dyshormonogenesis)
Iodine deficiency during pregnancy (most common cause worldwide)
Maternal hypothyroidism
🧠 Key Clinical Features of Cretinism:
Severe mental retardation
Growth retardation / dwarfism
Pot-bellied , puffy-faced , and protruding tongue
Delayed skeletal maturation
Hoarse cry
Umbilical hernia
Constipation
Dry, coarse skin
Poor feeding and lethargy
📌 If not treated early (usually within the first few weeks of life), neurological damage becomes irreversible .
❌ Why the Other Options Are Incorrect
Exophthalmos – Incorrect
Hypoparathyroidism in children – Incorrect
A completely different condition involving low PTH , which leads to hypocalcemia , tetany , and neuromuscular irritability — not features of cretinism.
Hypothyroidism in adults – Incorrect
Hyperparathyroidism in adults – Incorrect
When evaluating a condition caused by excess of a hormone that promotes glucose production and fat storage, ask: Would the body be in a state of building up or breaking down its energy stores?
161 / 232
Tags:
2018
Which of the following is not seen in Cushing’s syndrome?
Cushing’s syndrome refers to the clinical condition caused by prolonged exposure to high levels of cortisol , either due to endogenous overproduction (e.g., pituitary adenoma, adrenal tumor) or exogenous steroid use.
Cortisol affects carbohydrate, protein, and fat metabolism , and has catabolic and anti-insulin effects , leading to several characteristic signs and symptoms.
🔍 Key Features of Cushing’s Syndrome:
Hyperglycemia – ✅ Seen
Weight gain – ✅ Seen
Purple striae – ✅ Seen
Hirsutism – ✅ Seen
❌ Weight loss – Not Seen in Cushing’s syndrome
This is the opposite of what’s typically observed.
Cushing’s syndrome causes weight gain , muscle wasting , and fat redistribution — not weight loss.
Weight loss is more associated with Addison’s disease (adrenal insufficiency), malignancy , or thyrotoxicosis .
If a tumor in a master gland disrupts reproductive hormones and causes lactation without pregnancy, which hormone should you check first to trace the root cause?
162 / 232
Tags:
2018
Which of the following is a diagnostic test for pituitary tumors?
Pituitary tumors (also known as pituitary adenomas ) are often functioning tumors , meaning they secrete excess hormones. The most common type of functioning pituitary adenoma is the prolactinoma — a tumor that secretes prolactin .
Therefore, measuring serum prolactin levels is often the first diagnostic test when a pituitary tumor is suspected, especially in cases of:
High prolactin levels often correlate with the size of the tumor, and very high levels (e.g., >200 ng/mL) are usually indicative of a macroadenoma .
🔍 Other Tests: When and Why They Matter
The other tests listed can be supportive in identifying other types of pituitary tumors, but prolactin is the most commonly tested first because prolactinomas are the most frequent functional tumors .
❌ Why the Other Options Are Incorrect
TSH plasma level – Incorrect
Used to assess thyroid function.
Rare TSH-secreting tumors exist, but they are very rare .
Elevated TSH + elevated T3/T4 suggests secondary hyperthyroidism , but not the first test you’d order for a pituitary mass.
ACTH plasma level – Incorrect
IGF-1 plasma level – Incorrect
Helps diagnose acromegaly due to GH-secreting pituitary adenoma .
Important for GH-related tumors, but less common than prolactinomas.
Visual field exam – Incorrect
Important for macroadenomas that compress the optic chiasm , causing bitemporal hemianopia .
Useful after a tumor is suspected to assess the extent, but not a primary diagnostic tool for identifying hormone-secreting tumors.
When a hormone is overproduced by a gland, ask yourself whether the cause is likely to be a benign overgrowth, an immune system misfire, or something more sinister — and consider which of these is most statistically likely.
163 / 232
Tags:
2018
What is the most common cause of hyperparathyroidism?
Hyperparathyroidism is a condition characterized by excessive secretion of parathyroid hormone (PTH) , resulting in hypercalcemia , hypophosphatemia , and bone resorption .
It is classified into:
Primary hyperparathyroidism – intrinsic problem of the parathyroid glands.
Secondary hyperparathyroidism – due to chronic hypocalcemia (e.g., chronic kidney disease).
Tertiary hyperparathyroidism – autonomous gland function after long-standing secondary hyperparathyroidism.
🔬 Most Common Cause of Primary Hyperparathyroidism
Solitary parathyroid adenoma accounts for ~85% of primary hyperparathyroidism cases.
These are benign tumors of a single parathyroid gland.
They cause increased PTH secretion , leading to classical features: “Bones, stones, groans, and psychiatric overtones ” (bone pain, kidney stones, abdominal discomfort, and mental status changes).
❌ Why the Other Options Are Incorrect
Infection – Incorrect
Hyperplasia – Incorrect
Parathyroid hyperplasia accounts for ~10–15% of cases and involves all four glands .
More commonly seen in familial syndromes (e.g., MEN 1, MEN 2A), but less common than adenoma .
Autoimmune – Incorrect
Carcinoma – Incorrect
Parathyroid carcinoma is extremely rare (~<1% of cases).
It should be considered when there are very high PTH levels and severe hypercalcemia , but it is not a common cause .
Think about medical procedures that involve the same anatomical region as the parathyroid glands — while these glands are small and tucked away, a common therapeutic intervention nearby can easily disrupt their function.
164 / 232
Tags:
2018
What is the most common cause of hypoparathyroidism?
Hypoparathyroidism is a condition characterized by deficient secretion of parathyroid hormone (PTH) , which leads to hypocalcemia and hyperphosphatemia . PTH is essential for maintaining serum calcium by acting on bones, kidneys, and indirectly on the intestines.
There are several causes of hypoparathyroidism, but the most common cause—especially in adults—is surgical injury or removal of the parathyroid glands.
🩺 Most Common Cause: Post-surgical Hypoparathyroidism
❌ Why the Other Options Are Incorrect
Genetic – Incorrect
Genetic forms (e.g., DiGeorge syndrome, CaSR mutations) are important but are much less common , especially in adults.
Idiopathic – Incorrect
Autoimmune – Incorrect
Congenital – Incorrect
For many enzyme deficiencies, consider whether a loss of function in both gene copies is necessary for symptoms — this often points to a specific inheritance pattern.
165 / 232
Tags:
2018
What is the mode of inheritance of congenital adrenal hyperplasia?
Congenital adrenal hyperplasia (CAH) refers to a group of inherited enzymatic disorders affecting cortisol synthesis in the adrenal cortex . The most common form is 21-hydroxylase deficiency , which accounts for over 90% of CAH cases.
🔹 Key Genetic Points:
CAH is inherited in an autosomal recessive pattern .
Both copies of the gene (one from each parent) must be defective for the disease to manifest.
The gene most commonly affected is CYP21A2 , located on chromosome 6p21.3 .
🔸 Carrier Parents:
❌ Why the Other Options Are Incorrect:
X-linked recessive / dominant → These involve genes located on the X chromosome , typically affecting males more severely — CAH is not X-linked .
Mitochondrial inheritance → Passed down only through mothers , affects both sexes — CAH is nuclear , not mitochondrial.
Autosomal dominant → A single mutated allele causes the condition — CAH requires two mutated alleles .
When the kidneys can’t activate vitamin D properly, what happens to calcium levels in the blood — and how might the parathyroid glands react to that?
166 / 232
Tags:
2018
Secondary hyperparathyroidism can arise due to which of the following?
Secondary hyperparathyroidism refers to increased parathyroid hormone (PTH) secretion in response to chronic hypocalcemia or low active vitamin D levels , not due to primary gland pathology.
🔬 What does 1α-hydroxylase do?
It’s a renal enzyme that converts 25-hydroxycholecalciferol (inactive vitamin D) into 1,25-dihydroxycholecalciferol (active vitamin D, also known as calcitriol) .
Calcitriol increases intestinal calcium absorption and helps suppress PTH secretion.
⛓️ In 1α-hydroxylase deficiency:
This condition is seen in:
❌ Why the Other Options Are Incorrect:
21α-hydroxylase deficiency → Affects adrenal steroidogenesis (cortisol and aldosterone synthesis), not calcium or PTH. ❌ Related to congenital adrenal hyperplasia , not parathyroid function.
Deficiency of both 1α-hydroxylase and 21α-hydroxylase → While 1α-hydroxylase deficiency is relevant, 21α-hydroxylase has no role in calcium/vitamin D metabolism . This distracts from the key cause.
17α-hydroxylase deficiency → Also an adrenal enzyme defect; affects sex steroid and cortisol production. ❌ No relation to calcium metabolism or parathyroid regulation.
1α-hydroxylase overexpression → Would lead to increased calcitriol , hypercalcemia , and suppressed PTH — possibly causing hypoparathyroidism , not secondary hyperparathyroidism. ❌ Opposite effect.
Think about which branchial cleft anomaly commonly presents as a painless neck mass without external or internal openings.
167 / 232
Tags:
2020
A boy was brought to the outpatient department with a swelling anterior to sternocleidomastoid muscle at the lower one-third of the neck. Which of the following could be the likely pathology?
A cervical cyst (also called a branchial cyst ) typically presents as a painless swelling located anterior to the sternocleidomastoid muscle, often in the lower one-third of the neck .
It is a congenital cyst arising from the incomplete obliteration of the second branchial cleft during embryological development.
The cyst is filled with fluid and may enlarge slowly.
Usually, there are no sinus tracts or fistulas unless infected or complicated.
❌ Why the Other Options Are Incorrect:
A. Piriform fistula: A tract arising from the piriform sinus (third or fourth branchial pouch anomaly), typically presenting near the thyroid region, not usually as a cystic neck swelling.
B. External cervical sinus: An opening on the skin surface due to incomplete branchial cleft closure but presents with discharge, not just swelling.
C. Cervical fistula: Has both an internal and external opening; would usually present with persistent discharge.
E. Internal cervical sinus: Opens into the pharynx and not usually palpable as an external swelling.
Consider which type of blood vessel disease predominantly causes poor blood supply and tissue death in diabetic limbs.
168 / 232
Tags:
2020
In diabetes mellitus gangrene of the lower extremity is caused by which of the following?
In diabetes mellitus , gangrene of the lower extremities typically results from macrovascular disease , which refers to atherosclerosis of large and medium-sized arteries supplying the limbs.
Macrovascular complications include peripheral arterial disease (PAD) , which reduces blood flow and oxygen delivery.
This ischemia leads to tissue necrosis and gangrene.
Although microvascular disease (affecting small vessels) contributes to other diabetic complications (like retinopathy and nephropathy), it is less responsible for gangrene.
❌ Why the Other Options Are Incorrect:
A. None of these: Incorrect; macrovascular disease is a well-established cause.
B. Microvascular disease: Primarily affects capillaries and small vessels, less commonly causing gangrene.
D. Skin disease: May complicate diabetes but is not the primary cause of gangrene.
E. Trauma: Can precipitate ulcers but underlying ischemia from macrovascular disease is the main cause.
Consider which substance in the connective tissue binds water and causes swelling in hypothyroid patients.
169 / 232
Tags:
2020
A patient presents with swelling in his right leg and edema. He is diagnosed with Hashimoto thyroiditis. Which of the following is true for the reason behind fluid accumulation in his leg?
Hashimoto thyroiditis is a common cause of hypothyroidism , which leads to a characteristic type of non-pitting edema called myxedema .
The edema in hypothyroidism is due to accumulation of mucopolysaccharides , especially hyaluronic acid and chondroitin sulfate , in the interstitial spaces.
These glycosaminoglycans are hydrophilic , attracting and trapping water, which results in fluid retention and swelling .
This is a hallmark feature of hypothyroid-related edema.
❌ Why the Other Options Are Incorrect:
B. Increase in acetic acid: No relation to edema in hypothyroidism.
C. Hives: Are allergic skin reactions, not related to hypothyroid edema.
D. Abscess: Localized infection with pus; unrelated.
E. Hyperglycemia: Characteristic of diabetes; not the cause of hypothyroid edema.
In endocrine feedback loops, what typically happens to the stimulating hormone when the end hormone is in excess?
170 / 232
Tags:
2020
Which of the following is true regarding the lab diagnosis of hyperthyroidism?
In primary hyperthyroidism , such as Graves’ disease or toxic multinodular goiter , the thyroid gland overproduces thyroid hormones (free T₃ and T₄ ). These elevated hormone levels exert negative feedback on the hypothalamus and anterior pituitary, leading to:
Thus, the hallmark lab pattern in hyperthyroidism is:
❌ Why the Other Options Are Incorrect:
A. TSH levels increased: Seen in hypothyroidism or secondary hyperthyroidism (rare).
B & C. Parathyroid hormone levels (PTH): Not directly related to thyroid hormone production; PTH regulates calcium levels.
D. Free T₃ and T₄ levels decreased: Suggestive of hypothyroidism , not hyperthyroidism.
If a disease affecting the pituitary gland can alter both vision and hormonal function, and is detected quite frequently on MRI scans, what kind of pathology would be most likely to cause this?
171 / 232
Tags:
2018
Which of the following is the most common pituitary disease?
The pituitary gland is subject to a variety of disorders, but by far the most common pathology affecting it is the pituitary adenoma — a benign tumor of the anterior pituitary cells.
🔬 Pituitary Adenoma:
Accounts for 10–15% of all intracranial tumors .
Arises from adenohypophyseal (anterior pituitary) cells.
Can be functional (hormone-producing) or non-functional .
Common types:
Prolactinomas (most common subtype)
Somatotroph adenomas (GH-producing)
Corticotroph adenomas (ACTH-producing)
Symptoms vary:
Hormonal excess (e.g., galactorrhea, acromegaly, Cushing’s disease)
Mass effect (e.g., headaches, bitemporal hemianopia from optic chiasm compression)
❌ Why the Other Options Are Incorrect:
Pituitary metastatic tumors → Rare. Though the pituitary can be a site for metastasis (especially breast/lung cancer), these are uncommon compared to primary adenomas.
Sarcoidosis → A multisystem granulomatous disease; involvement of the pituitary (neurosarcoidosis) is very rare .
Pituitary apoplexy → A sudden hemorrhage or infarction of a pituitary tumor. It’s a serious complication , but not a common baseline disease .
Empty sella syndrome → Condition where the sella turcica appears empty on imaging. It may be idiopathic or secondary, and often asymptomatic. While not rare, it’s not a primary pituitary disease per se and far less common in symptomatic presentations than adenomas.
Think about which chemical messengers prepare the body for “fight or flight”—and which rare tumor causes their overproduction.
172 / 232
Tags:
2020
What does pheochromocytoma, a rare tumor, secrete?
Pheochromocytoma is a rare, catecholamine-secreting tumor that arises from chromaffin cells of the adrenal medulla .
🔹 Clinical Effects:
The excess catecholamines lead to symptoms of sympathetic overactivity , such as:
❌ Why the Other Options Are Incorrect:
A. Tyrosinase: An enzyme involved in melanin synthesis; not secreted by pheochromocytomas.
B. Androgen: Secreted by adrenal cortex (zona reticularis) or gonads, not medullary tumors.
C. Insulin: Produced by pancreatic beta cells , not adrenal tumors.
D. Oxytocin: Produced by the hypothalamus and released by the posterior pituitary , unrelated to adrenal function.
Think about which enzyme essential for thyroid hormone production is targeted by autoantibodies in chronic autoimmune thyroiditis.
173 / 232
Tags:
2020
A 35-year-old lady is diagnosed with autoimmune hypothyroidism. Her physician is concerned about thyroid autoantibodies. Which of the following is the most appropriate antibody to detect autoimmune thyroid disease?
The most common and clinically significant autoantibodies in autoimmune hypothyroidism (Hashimoto’s thyroiditis) are:
❌ Why the Other Options Are Incorrect:
A. Antifollicular: Non-specific term; not commonly used clinically.
B. Antinuclear: Present in systemic autoimmune diseases like lupus, but not specific for thyroid autoimmunity.
D. Anticytoplasmic: More relevant to certain vasculitides (ANCA-associated), not thyroid.
E. Anti-membrane: Not a recognized antibody in thyroid autoimmune disease.
Consider the time frame: this syndrome develops after a dramatic event during childbirth. Does the problem arise before the baby is born, or after when blood loss may critically reduce pituitary perfusion?
174 / 232
Tags:
2018
Which of the following statements is wrong about the Sheehan syndrome?
Sheehan syndrome is a condition caused by ischemic necrosis of the pituitary gland , typically due to severe postpartum hemorrhage (PPH) . It is a form of hypopituitarism seen after childbirth .
🔬 Pathophysiology Overview:
During pregnancy, the anterior pituitary enlarges due to increased demand for hormone production.
This growth is not accompanied by a proportional increase in blood supply , making the gland more vulnerable to hypoperfusion .
In cases of massive blood loss during or after delivery , reduced perfusion leads to necrosis of the anterior pituitary → Sheehan syndrome .
✅ Correct Statements About Sheehan Syndrome:
Statement
Explanation
It leads to pituitary necrosis
✅ Yes — ischemia causes necrosis of anterior pituitary
It is associated with postpartum hemorrhage
✅ This is the primary risk factor
It can present as cold intolerance
✅ Due to secondary hypothyroidism from TSH deficiency
It results from ischemia to pituitary gland
✅ Central mechanism of the disease
❌ Why the Incorrect Option Is Wrong:
Option
Why It’s Incorrect
“It is associated with antepartum hemorrhage”
❌ Sheehan syndrome occurs due to postpartum hemorrhage , not bleeding before delivery
If hypertension is paroxysmal and accompanied by classic “fight-or-flight” symptoms, consider a tumor that mimics sympathetic overactivity.
175 / 232
Tags:
2020
A 34-year-old female comes to the outpatient department with uncontrolled hypertension. A computed tomography (CT) scan reveals bilateral enlargement of the suprarenal gland. What is the most probable cause?
Pheochromocytoma is a catecholamine-secreting tumor that arises from chromaffin cells of the adrenal medulla . It is a known cause of secondary hypertension .
While most pheochromocytomas are unilateral , bilateral pheochromocytomas can occur, especially in association with familial syndromes like:
Multiple Endocrine Neoplasia (MEN) type 2
Von Hippel–Lindau disease
Neurofibromatosis type 1 (NF1)
Symptoms include episodic hypertension , headaches, palpitations, sweating, and anxiety—due to surges in epinephrine and norepinephrine .
❌ Why the Other Options Are Less Likely:
A. Renal tumor: Can cause hypertension via renin secretion, but does not explain adrenal enlargement .
B. Hyperplasia of suprarenal medulla: Not a commonly diagnosed or well-defined clinical condition.
D. Renal cyst: Usually asymptomatic and not related to hypertension or adrenal changes.
E. Hyperplasia of suprarenal cortex: Causes cortical hormone excess (aldosterone, cortisol) but pheochromocytoma better explains medullary involvement with catecholamine-related symptoms.
If a patient presents with signs of hyperthyroidism and a non-tender thyroid swelling without nodularity or systemic signs of infection, what type of thyroid condition might quietly show up and resolve on its own over time?
176 / 232
Tags:
2018
A woman comes to the outpatient department with a complaint of being restless for the past 3 months. She has a single painless swelling in her neck. What is the most likely diagnosis?
This woman presents with:
Restlessness (a sign of thyrotoxicosis ),
A painless neck swelling , and
A duration of 3 months , suggesting a subacute , not acute condition.
These features are classic for subacute lymphocytic thyroiditis , also known as painless (or silent) thyroiditis — a self-limiting autoimmune thyroid disorder that causes transient hyperthyroidism followed by hypothyroidism , and eventually returns to euthyroid .
🔍 Key Clinical Features of Subacute Lymphocytic (Silent) Thyroiditis:
Painless goiter
Mild to moderate hyperthyroid symptoms (e.g., anxiety, restlessness, weight loss)
No tenderness or fever (distinguishes it from granulomatous thyroiditis)
Often occurs postpartum , or in women with a history of autoimmune disease
Thyroid is not diffusely enlarged or tender
❌ Why the Other Options Are Incorrect:
Option
Why Incorrect
Iodine deficiency
Causes goiter , but usually presents with hypothyroid symptoms , not hyperthyroid signs like restlessness
Toxic multinodular goiter
Typically presents with multiple nodules , usually in older patients , and with a longer history
Subacute granulomatous thyroiditis (De Quervain )
Presents with painful thyroid , fever, raised ESR — not a painless swelling
Multinodular goiter
Generally non-toxic unless specified as toxic , and develops over years , not months
When evaluating hormone disorders, would you begin testing the gland directly, or the signal that controls it from higher up in the axis? Think about what the “thermostat” of the endocrine system would be.
177 / 232
Tags:
2018
What is the test for Graves disease?
Graves disease is an autoimmune disorder and the most common cause of hyperthyroidism . It is caused by TSH receptor-stimulating autoantibodies (TRAb), which mimic TSH and overstimulate the thyroid gland to produce excess T₃ and T₄ .
🔬 Initial and Best Screening Test:
✅ TSH concentration is the first-line and most sensitive test for Graves disease and hyperthyroidism in general.
In Graves disease , TSH is suppressed (low or undetectable) due to negative feedback from elevated T₃ and T₄.
If TSH is low, free T₄ and T₃ levels are measured next to confirm hyperthyroidism.
To confirm Graves specifically , a TRAb (TSI) test or radioactive iodine uptake scan may be performed.
📈 Typical Lab Pattern in Graves Disease:
Test
Result
TSH
↓↓↓ (very low or undetectable)
Free T₄
↑↑
T₃
↑↑ (often more elevated than T₄)
TSH receptor antibodies (TRAb/TSI)
✅ Positive
❌ Why Other Options Are Incorrect as Initial Test:
Option
Reason
T₄ levels
May be elevated, but not as sensitive as TSH for screening
T₃ levels
Elevated in Graves, but T₃ toxicosis may be subtle and TSH is still preferred for diagnosis
CRH levels
Regulates ACTH , not involved in thyroid axis
ACTH levels
Related to adrenal function, not thyroid
When a hormone fluctuates throughout the day, is a one-time sample enough to capture the full picture? Or would a continuous measure over time better reveal excess production?
178 / 232
Tags:
2018
What is the best diagnostic test for Cushing syndrome?
Cushing syndrome is a condition caused by prolonged exposure to elevated levels of cortisol , whether from endogenous (e.g., adrenal tumor, pituitary adenoma) or exogenous (e.g., glucocorticoid therapy) sources.
To diagnose Cushing syndrome, the goal is to establish cortisol excess reliably and independently of its normal daily variation (diurnal rhythm).
🔍 Best Initial Diagnostic Test:
✅ 24-hour urine free cortisol (UFC) test
Measures unbound (active) cortisol excreted in urine over a full day
Reflects total daily cortisol production
Not affected by diurnal variations
A value >50–100 μg/day (or elevated above lab reference range) is suggestive of hypercortisolism , especially if confirmed on 2 or more collections .
❌ Why Other Options Are Incorrect Initially:
Test
Why It’s Not the Best Initial Diagnostic
ACTH levels
Useful after confirming cortisol excess to determine the cause (ACTH-dependent vs ACTH-independent)
CRH stimulation test
Used to differentiate Cushing disease (pituitary source) from ectopic ACTH source; not for initial screening
High-dose dexamethasone suppression test
A second-line test to determine if excess cortisol is suppressible (used after diagnosis is confirmed)
Serum cortisol
Varies significantly throughout the day (diurnal); not reliable alone to diagnose Cushing syndrome
Consider the anatomical positioning of the pituitary within the skull and how a tumor there could press on nearby visual pathways causing specific visual field defects.
179 / 232
Tags:
2020
A person suffers from bitemporal hemianopia, which arose because of the compression of the optic nerve by the pituitary adenoma. Where is the pituitary gland present in relation to the optic chiasma?
The pituitary gland sits in the sella turcica of the sphenoid bone, which is located just inferior to the optic chiasma .
The optic chiasma is the point where the optic nerves partially cross.
A pituitary adenoma , by expanding upward, compresses the optic chiasma from below , causing characteristic bitemporal hemianopia (loss of peripheral vision in both visual fields).
This is because the crossing nasal fibers in the chiasma, responsible for temporal visual fields, are affected.
❌ Why Other Positions Are Incorrect:
Anteriorly: The pituitary is not located in front of the optic chiasma.
Superiorly: The optic chiasma lies above the pituitary, not vice versa.
Laterally: The pituitary lies centrally, not on the sides of the chiasma.
Posteriorly: The pituitary lies in front and below the chiasma, not behind it.
If a condition affects not just metabolism but also causes the eyes to bulge outward due to immune activation in the orbit, what systemic process could link the two?
180 / 232
Tags:
2018
Exophthalmos is most strongly associated with which of the following pathologies?
Exophthalmos — also known as proptosis (forward protrusion of the eyeballs) — is a hallmark feature of Graves disease , which is an autoimmune form of hyperthyroidism .
🔍 Why It Happens in Graves Disease:
Graves disease is caused by thyroid-stimulating immunoglobulins (TSIs) that bind to TSH receptors in the thyroid and extra-thyroidal tissues , such as fibroblasts in the orbit .
Orbital fibroblasts stimulated by TSIs produce:
This leads to:
Swelling of orbital tissues
Extraocular muscle enlargement
Forward displacement of the eyeball (exophthalmos)
This is a unique autoimmune orbitopathy and is not seen in other thyroid conditions .
❌ Why the Other Options Are Incorrect:
Option
Why Incorrect
Conn syndrome
Aldosterone-producing adenoma → causes hypertension and hypokalemia , not eye symptoms
Hypothyroidism
May cause periorbital puffiness , but not true exophthalmos
Hyperparathyroidism
Associated with bone, kidney, and GI issues , not eye involvement
Hypoparathyroidism
Causes hypocalcemia , muscle cramps, Chvostek’s and Trousseau’s signs — no orbital involvement
🧬 Graves Orbitopathy (Thyroid Eye Disease) Key Signs:
Think about the common systemic illnesses that precede a painful thyroid inflammation and consider the most frequent infectious agents involved.
181 / 232
Tags:
2020
Thyroiditis is the inflammation of the thyroid gland encompassing a number of disorders that have some elements of inflammation. Subacute thyroiditis is believed to be triggered by which of the following insults?
Subacute thyroiditis , also known as de Quervain’s thyroiditis , is a self-limiting inflammatory disorder of the thyroid gland. It often follows a viral upper respiratory tract infection .
🔍 Key Features:
Patients typically present with a painful, tender thyroid gland , fever, and symptoms of hyperthyroidism followed by hypothyroidism.
Commonly implicated viruses include Coxsackievirus, mumps, adenovirus, influenza , and echovirus .
The inflammation leads to destruction of thyroid follicles and release of preformed thyroid hormone, causing transient thyrotoxicosis.
Unlike bacterial thyroiditis, subacute thyroiditis is not caused by bacterial or fungal infection .
❌ Why the Other Options Are Incorrect:
A. Trauma
B. Fungal infection
C. Bacterial infection
D. Iodine deficiency
When multiple organ systems slow down — digestion, heart, mind, and metabolism — think of a central hormonal controller that governs them all.
182 / 232
Tags:
2020
A middle-aged female presents to you with complaints of weight gain, swelling all over the body, constipation, and shortness of breath on mild to moderate exertion. Which of the following tests will you advise to diagnose this patient?
This case presents a classic picture of hypothyroidism , particularly autoimmune hypothyroidism (Hashimoto’s thyroiditis) , in a middle-aged woman .
🔍 Key Clinical Clues:
Symptom
Interpretation
Weight gain
Common in hypothyroidism due to slowed metabolism
Swelling (edema)
Likely myxedema — non-pitting edema due to mucopolysaccharide deposition
Constipation
Classic symptom of hypothyroidism
Shortness of breath on exertion
Due to pleural effusion , reduced respiratory drive , or cardiomyopathy in hypothyroid state
➡️ All point to thyroid dysfunction , especially hypothyroidism
🧪 Why Check TSH, Free T4, and Anti-TPO Antibodies?
TSH : Screening and diagnostic marker (elevated in primary hypothyroidism)
Free T4 : To confirm if TSH is abnormal and assess severity
Anti-TPO antibodies : Indicates autoimmune thyroiditis (Hashimoto’s) , the most common cause of hypothyroidism in middle-aged women
❌ Why Other Options Are Incorrect:
Fasting blood sugar, lipid profile, serum insulin, pelvic ultrasound – ❌
May help with metabolic syndrome or PCOS , but do not explain generalized swelling and constipation
Not first-line for these symptoms
Chest X-ray PA view – ❌
Echocardiography – ❌
Complete blood count with peripheral film – ❌
May show anemia of hypothyroidism (macrocytic or normocytic), but not diagnostic alone
Doesn’t explain all systemic signs
🧬 Hashimoto’s Thyroiditis (Autoimmune Hypothyroidism):
Most common cause of hypothyroidism in developed countries
Positive anti-TPO and/or anti-thyroglobulin antibodies
Common in middle-aged women
Symptoms:
Fatigue, weight gain, constipation
Cold intolerance, dry skin
Menstrual irregularities
Edema (myxedema)
Bradycardia, dyspnea
If a young girl has never menstruated, shows signs of estrogen deficiency, and her gonadotropin levels are high, where is the problem likely located — in the signal, or in the target organ?
183 / 232
Tags:
2020
A 20-year-old female presents to the outpatient department to be evaluated for short stature. Her history is significant for psychological problems, reduced IQ, altered bowel habits, and primary amenorrhea. Which of the following is the most likely cause of her short stature and clinical features?
This case describes a young female with:
Short stature
Primary amenorrhea
Low IQ
Altered bowel habits
Psychological issues
All of these are highly suggestive of a congenital or genetic syndrome , most notably Turner syndrome , which is a classic cause of hypergonadotropic hypogonadism .
🔍 Key Features Breakdown:
Feature
Interpretation
Short stature
Common in Turner syndrome due to SHOX gene haploinsufficiency
Primary amenorrhea
Due to streak ovaries → ovarian failure
Low IQ + psychological issues
Some Turner variants may show neurocognitive deficits (though IQ is often normal)
Altered bowel habits
Can be due to associated conditions (e.g., hypothyroidism or GI dysmotility)
Elevated FSH and LH
Seen in hypergonadotropic hypogonadism due to lack of estrogen feedback
➡️ Together, these features suggest ovarian failure due to gonadal dysgenesis (e.g., Turner syndrome or similar) — leading to high LH/FSH but low estrogen .
🧬 Hypergonadotropic Hypogonadism:
❌ Why Other Options Are Incorrect:
Pituitary failure / Sheehan’s syndrome – ❌
Occurs postpartum , not in adolescents
Causes hypogonadotropic hypogonadism , not primary amenorrhea
No short stature association
Hypogonadotropic hypogonadism – ❌
Would have low LH/FSH , not high
No elevated gonadotropins, and not typically associated with short stature
Polyglandular autoimmune syndrome – ❌
Often affects multiple glands (thyroid, adrenals, parathyroids)
May include hypogonadism , but not short stature or primary amenorrhea as isolated features
Celiac disease – ❌
Can cause short stature and delayed puberty , but not usually primary amenorrhea with high gonadotropins
Usually has malabsorption symptoms , not cognitive or psychological signs
When a young woman presents with a staring gaze, diffuse thyroid swelling, and signs of sympathetic overdrive, think about a condition that not only overstimulates the gland but also the tissues around the eyes.
184 / 232
Tags:
2020
A 35-year-old woman presents to the clinic with complaints of lethargy, palpitations, and weakness. She has had a profound weight loss of 12 Kg over the last 4 months. On examination, there are tremors of the hands and sweaty palms. Her eyes have a constant stare and she looks very frightened. There is a diffuse, firm goiter that moves with deglutition. Her thyroid function tests are as follows: thyroid stimulating hormone (TSH) – <0.001 (0.4-4.0), total T4 - 20.9 (5.0-11.2), free T4 - 1.9 (0.86-1.76). Which of the following is the most likely cause of her condition?
This case presents a classic picture of overt hyperthyroidism , and among the potential causes, Grave’s disease is by far the most consistent with all clinical and biochemical features described.
🔍 Key Clinical and Biochemical Features:
Finding
Interpretation
Lethargy, palpitations, tremors, weight loss
Classic symptoms of thyrotoxicosis
Sweaty palms, frightened stare, tremors
Signs of sympathetic overactivity from excess thyroid hormone
Diffuse, firm goiter that moves with deglutition
Typical of Grave’s disease
“Constant stare” / lid lag or stare
Suggestive of thyroid eye disease
TSH <0.001 (suppressed)
Primary hyperthyroidism (thyroid origin)
Total T4 = 20.9 (↑↑), Free T4 = 1.9 (↑)
Confirms thyrotoxicosis
🔬 Why It’s Grave’s Disease:
❌ Why the Other Options Are Incorrect:
Subacute thyroiditis – ❌
Often painful , associated with fever , elevated ESR , and tender thyroid
Thyroid is usually not firm and diffuse , and no eye signs
Factitious thyroiditis (thyrotoxicosis factitia) – ❌
Due to exogenous thyroid hormone intake
TSH suppressed, T4 elevated, but no goiter or eye signs
Thyroid uptake scans would show no uptake
Toxic multinodular goiter – ❌
Hyperthyroid phase of Hashimoto’s (Hashitoxicosis) – ❌
Usually transient and mild
Associated with autoimmune thyroiditis , often followed by hypothyroidism
No orbitopathy , no persistent severe hyperthyroid symptoms
🧪 Diagnosis Confirmation:
Think about which part of the adrenal gland produces the “fight or flight” hormones, and where a tumor causing episodic high blood pressure would originate.
185 / 232
Tags:
2020
Pheochromocytoma is a tumor of which of the following?
Pheochromocytoma is a tumor arising from the chromaffin cells of the adrenal medulla . These cells are neuroendocrine cells that produce catecholamines (mainly epinephrine and norepinephrine ).
🔍 Key points:
Tumor causes excess catecholamine secretion , leading to symptoms like episodic hypertension, headaches, sweating, and palpitations .
It is a neuroendocrine tumor originating specifically from the medullary portion of the adrenal gland , not the cortex.
Sometimes similar tumors arise outside the adrenal medulla (called paragangliomas ) but pheochromocytoma refers specifically to the adrenal medulla.
❌ Why the Other Options Are Incorrect:
B & C. Hepatic origin
D. Vascular origin
E. Adrenal cortex
The adrenal cortex produces steroid hormones (cortisol, aldosterone, androgens), and tumors here cause different syndromes (e.g., Cushing’s syndrome), not pheochromocytoma.
If a young woman stops menstruating and begins to show symptoms usually seen in postmenopausal women, but without a positive pregnancy test, what could be failing her endocrine system far too early?
186 / 232
Tags:
2020
A 31-year-old female presents to the medical outpatient department with complaints of amenorrhea for the past six months along with dyspareunia and bone pain all over the body. Investigations reveal negative urine pregnancy test, high concentrations of luteinizing hormone (LH) and follicle-stimulating hormone (FSH) along with low or low-normal estradiol. Which of the following is the most likely cause of amenorrhea in this patient?
This clinical scenario clearly describes primary ovarian insufficiency (POI) , also known as premature ovarian failure , which is a form of hypergonadotropic hypogonadism in young women under age 40 .
🔬 Key Clinical and Laboratory Clues:
Feature
Interpretation
Amenorrhea x 6 months
Secondary amenorrhea
Dyspareunia
Suggests estrogen deficiency
Bone pain
Due to reduced estrogen → low bone density
Negative pregnancy test
Rules out pregnancy
High LH and FSH
Indicates loss of negative feedback → hypergonadotropic hypogonadism
Low or low-normal estradiol
Indicates ovarian failure
🧬 What Is Premature Ovarian Failure (POF)?
❌ Why Other Options Are Incorrect:
Polycystic ovary syndrome (PCOS) – ❌
Hypergonadotropic hypogonadism – ❌
This is a mechanism , not a diagnosis
Premature ovarian failure is a cause of hypergonadotropic hypogonadism
Androgen-secreting tumor – ❌
Presents with virilization (deep voice, hirsutism, clitoromegaly), not estrogen deficiency
LH and FSH would be suppressed due to negative feedback
Pregnancy – ❌
🩺 Management Tip:
Confirm diagnosis with two FSH levels >40 IU/L at least 1 month apart
Rule out autoimmune and chromosomal causes
Consider hormone replacement therapy to prevent osteoporosis and manage symptoms
When a young woman presents with menstrual irregularity, hormonal suppression, and visual changes, think beyond the ovaries — what central structure might be responsible for both endocrine disruption and compressive symptoms?
187 / 232
Tags:
2020
A 36-year-old female presents to the clinic with complaints of secondary amenorrhea and headache for 6 months. She has 3 children, the last-born is 2 years old. She started oral contraceptives after the birth of her last born, but stopped 11 months ago. She has also noted weight gain during the same time period. On examination, Ferriman-Gallwey score is 8/32, secondary sexual characteristics are well developed and there is no galactorrhea. However, visual confrontation shows left inferior temporal field defects. Serum thyroid-stimulating hormone (TSH) is 2.4 (0.4-4.0), luteinizing hormone (LH) is 0.2 (1.2-4.0), follicular stimulating hormone (FSH) is 2.0 (2.0-8.0) and prolactin levels are 90 (10-23). What is the most likely cause of her presentation?
This case is a classic example of pituitary prolactinoma , presenting with features of hyperprolactinemia , secondary amenorrhea , and a mass effect on the optic chiasm.
📌 Key Clinical Clues from the Case:
Feature
Interpretation
Secondary amenorrhea
No menstruation for >6 months in a woman who previously had regular cycles
Headache + visual field defect (left inferior temporal field)
Suggests a pituitary mass compressing the optic chiasm (classically causes bitemporal hemianopia , but may present asymmetrically early on)
Prolactin = 90 ng/mL (↑)
Significantly elevated — normal is 10–23
Low LH (0.2) and borderline low FSH (2.0)
Indicates suppressed gonadotropins , suggesting hypogonadism
Ferriman-Gallwey score = 8/32
Mild hirsutism , may be a result of hormonal imbalance
No galactorrhea
Doesn’t rule out prolactinoma; not all patients exhibit galactorrhea
🔬 Mechanism of Symptoms in Prolactinoma:
↑ Prolactin inhibits GnRH from the hypothalamus → ↓ LH and FSH → Secondary hypogonadism → amenorrhea and infertility
Mass effect of large pituitary adenomas causes headache and visual field defects
Hirsutism may result from estrogen deficiency and mild androgen excess
❌ Why the Other Options Are Incorrect:
Lactational amenorrhea – ❌
She stopped breastfeeding and contraceptives 11 months ago
This would not explain prolactin of 90 ng/mL , visual changes , or low LH
Secondary hypogonadism – ❌
Technically correct in mechanism (↓LH/FSH), but not the root cause
Prolactinoma is the cause of the hypogonadism , so this is too broad/nonspecific
Polycystic Ovary Syndrome (PCOS) – ❌
PCOS features: chronic anovulation, normal or high LH , high androgens
This patient has low LH , no mention of polycystic ovaries, and very high prolactin , which is not typical of PCOS
Hypothyroidism – ❌
🧠 Diagnosis Recap:
If a patient with long-standing diabetes develops symmetrical sensory loss in the feet first, with slow progression upward and diminished reflexes, what pattern of nerve damage best fits this length-dependent progression?
188 / 232
Tags:
2020
A 60-year-old female presents to the clinic with complaints of pins and needles sensation along with burning feet over the last 8 to 9 months. She is a known case of diabetes and has been non-compliant with oral hypoglycemic therapy for the last 15 years. Clinical examination reveals sensory loss in a stocking pattern, absent ankle jerks, and subtle weakness of dorsiflexion of the toes. Which of the following is the dominant type of neuropathy in this patient?
This clinical scenario is a classic description of the most common form of diabetic neuropathy — distal symmetric polyneuropathy , also known as diabetic peripheral neuropathy (DPN) .
🔍 Key Features From the Case:
Clinical Feature
Interpretation
Long-standing, poorly controlled diabetes
Major risk factor for diabetic neuropathy
Pins and needles sensation + burning feet
Suggests sensory neuropathy
Sensory loss in stocking distribution
Length-dependent pattern of nerve damage
Absent ankle jerks
Common in peripheral neuropathy
Weak dorsiflexion of toes
Late motor involvement in distal neuropathy
➡️ These findings are hallmark features of distal symmetric (sensorimotor) polyneuropathy , the most common diabetic neuropathy.
📌 What Is Distal Symmetric Neuropathy?
Chronic, progressive neuropathy
Starts in the toes and feet , then progresses up the legs
May eventually involve the hands (“glove-and-stocking” distribution)
Affects:
Small fibers (burning, pain, temperature)
Large fibers (vibration, proprioception, reflexes)
Causes sensory, motor , and autonomic symptoms over time
❌ Why the Other Options Are Incorrect:
Small fiber neuropathy – ❌
Typically causes burning pain, allodynia, and temperature loss , but reflexes and strength are usually preserved
Does not explain the loss of ankle reflexes and motor weakness
Mononeuropathy multiplex – ❌
Involves multiple, non-contiguous peripheral nerves (e.g., right radial + left peroneal)
Asymmetric and multifocal , not symmetric stocking pattern
Entrapment neuropathy – ❌
Focal neuropathy (e.g., carpal tunnel)
Not symmetrical , and doesn’t explain stocking distribution
Amyotrophic neuropathy (Diabetic amyotrophy) – ❌
Usually affects older men with sudden asymmetric pain and weakness in proximal leg muscles (e.g., thigh)
Doesn’t match this chronic, distal presentation
When interpreting a scan with increased uptake confined to one area and clinical hyperthyroidism, think about which thyroid lesions actively produce hormone without waiting for signals.
189 / 232
Tags:
2020
An old lady with a diagnosis of hyperthyroidism undergoes a thyroid scan. The findings show increased uptake of radioactive iodine in a solitary circumscribed area. Which of the following is the most likely diagnosis?
This is a classic presentation of a toxic adenoma , also called a toxic solitary nodule . It is a benign, hyperfunctioning thyroid nodule that autonomously produces thyroid hormone , independent of TSH regulation.
🔬 Key Features:
Hyperthyroidism (clinical or subclinical): Due to excessive hormone production
Thyroid scan : Shows increased radioactive iodine uptake in one distinct, solitary area (“hot nodule “)
Suppression of surrounding tissue : Normal thyroid tissue shows reduced uptake due to TSH suppression by high circulating thyroid hormone
❌ Why the Other Options Are Incorrect:
A. Hurthle cell (oxyphil) adenoma
Typically nonfunctional (“cold” on scan)
Rarely causes hyperthyroidism
Often found incidentally or as a mass
B. Thyroiditis
Inflammation leads to release of preformed hormone , causing transient hyperthyroidism
Radioactive iodine uptake is low or absent (because synthesis is suppressed)
C. Colloid nodule
A benign, non-functioning lesion
Appears as a “cold” nodule on scan (low uptake)
Does not cause hyperthyroidism
D. Follicular adenoma
Usually nonfunctioning (cold)
Occasionally functioning, but not typically with the distinct hot spot seen in toxic adenoma
When testicular function is severely impaired from birth without a hormonal or obstructive cause, what subtle, inheritable genetic defect might silently prevent sperm development?
190 / 232
Tags:
2020
A 24-year-old man presents to the clinic for evaluation of primary infertility. His semen analysis on two separate occasions shows severely abnormal spermatogenesis. Which of the following is the cause of this abnormality?
📌 Why Y Chromosome Microdeletions?
Y chromosome contains critical genes for spermatogenesis , particularly in the AZF regions (Azoospermia Factor) .
Microdeletions in AZFa, AZFb, or AZFc → cause severe impairment of sperm production .
Most common genetic cause of non-obstructive azoospermia in men.
These men typically:
Have normal virilization
Normal testosterone levels
Normal FSH/LH or elevated depending on the extent of testicular failure
No sperm or very few in ejaculate
❌ Why the Other Options Are Incorrect:
Previous sexually transmitted infection – ❌
Can cause obstructive azoospermia , not abnormal spermatogenesis (testicular function is normal).
Here, the defect is sperm production , not transport.
Hypogonadotropic hypogonadism – ❌
Causes low FSH/LH → low testosterone and impaired spermatogenesis .
But would be associated with underdeveloped secondary sex characteristics and low libido , often reversible with hormone therapy.
Varicocele – ❌
Common cause of mild to moderate sperm abnormalities , not typically severe spermatogenic failure .
Also more common on the left side , and would show gradual onset , often treatable.
Hypergonadotropic hypogonadism – ❌
Refers to testicular failure with high FSH/LH and low testosterone .
While this may be seen secondarily in Y chromosome microdeletions , it is a descriptive category , not the root cause.
🧬 Clinical Tip:
If you see unexplained diabetes in a young person with sudden weight gain and skin changes such as deep stretch marks, think beyond primary metabolic causes. What hormone could be silently sabotaging glucose metabolism?
191 / 232
Tags:
2020
A 20-year-old man is referred to the diabetes clinic with newly diagnosed diabetes (fasting glucose on two occasions >289 mg/dL). He has had osmotic symptoms (polyuria and polydipsia) for 6 months, and his weight has recently increased by around 14 kg. On examination, there are purplish striae on the abdomen with central obesity. Which of the following is the most likely diagnosis?
Let’s examine each of the key findings :
Finding
Interpretation
Polyuria, polydipsia
Classic osmotic symptoms of hyperglycemia
High fasting glucose
Confirms diabetes mellitus
Purple striae
Strongly suggests Cushing syndrome
Central obesity + rapid weight gain
Seen in Cushing’s , not just metabolic syndrome
Young age (20 y/o)
Less typical for Type 2 DM/metabolic syndrome
📌 What is Cushing Syndrome?
Excess cortisol (endogenous or exogenous)
Cortisol has diabetogenic effects : increases gluconeogenesis, insulin resistance, and appetite
Classic signs include:
Purple striae (due to collagen breakdown)
Central obesity , buffalo hump , moon face
Weight gain
Hypertension
Proximal muscle weakness
Glucose intolerance or overt diabetes
❌ Why the Other Options Are Incorrect:
Primary hypothyroidism – ❌
Causes weight gain but not diabetes or purple striae
Often associated with cold intolerance , bradycardia , dry skin
Acromegaly – ❌
Causes insulin resistance, but also involves coarse facial features , large hands/feet , and prognathism — none of which are mentioned
Type 2 diabetes with metabolic syndrome – ❌
Central obesity and diabetes fit, but not purple striae
Also unusual in a 20-year-old without family history or long-standing obesity
Lipodystrophy – ❌
Associated with abnormal fat distribution , insulin resistance , and very low leptin
No mention of fat loss in limbs or face , which is characteristic
When the heart is beating chaotically and the blood pressure is crashing, should your first move be to suppress fluid or rhythm—or immediately restore organized contraction?
192 / 232
Tags:
2020
A 25-year-old woman admitted to the hospital for the treatment of thyrotoxicosis suddenly becomes severely dyspneic. Her vitals are as follows: blood pressure 60/40 mm Hg, heart rate 180/minute with irregularly regular pulse, and cold peripheries. ECG reveals fast atrial fibrillation; her ECG done one day earlier had only revealed tachycardia. Chest x-ray reveals frank pulmonary edema. Which of the following is the most appropriate treatment?
📌 Diagnosis:
➡️ This is a case of unstable AF — a cardiac emergency .
🩺 Management Principles for Unstable Atrial Fibrillation:
According to ACLS guidelines :
If the patient is hemodynamically unstable (hypotension, pulmonary edema, altered mentation, chest pain), and in AF :
➤ Immediate synchronized DC cardioversion is the treatment of choice.
❌ Why the Other Options Are Incorrect:
Carotid massage – ❌
IV furosemide and IV amiodarone – ❌
Furosemide may reduce preload, worsening hypotension.
Amiodarone is not first-line in unstable patients — it delays urgent correction.
IV verapamil – ❌
IV furosemide alone – ❌
⚡️ DC Cardioversion Is Life-Saving Here
When a patient has signs of both hormone excess and hormone deficiency, ask: could a single expanding lesion be overproducing one hormone while compressing others?
193 / 232
Tags:
2020
A 30-year-old man is referred to the clinic for evaluation of visual field defects. Additional complaints include fatigue, myalgia, headache, bilateral knee pain, and excessive perspiration. His blood pressure is 156/94 mm Hg, pulse 56/ minute, weight 105 kg, and height 6 feet 2 inches. His hands are thick and enlarged; his calcaneus could not be felt through his heel pad. Laboratory studies are as follows: free thryoxine index 3.6 mcg/dL (4-11 mcg/dL), TSH 0.7 U/L, testosterone 87 ng/dL (300 – 1000 ng/dL), LH 3.2 mIU/mL (2 – 28 mIU/mL), prolactin 5 ng/dL (< 8 ng/dL), 8 am cortisol 2 mcg/dL (6-24 mcg/dL), and fasting glucose 186 mg/dL. Which of the following diagnoses best explains the patient's signs and symptoms?
Let’s break this down systematically by symptoms and labs :
🔍 Clinical Clues in the Stem:
Feature
Interpretation
Visual field defects
Suggests optic chiasm compression (pituitary mass)
Thick, enlarged hands , heel pad
Classic signs of acromegaly
Fatigue, myalgia, bilateral knee pain
Often due to GH excess , joint hypertrophy
Height = 6’2” & weight = 105 kg
Large frame, possibly post-pubertal GH excess
Excessive perspiration
Common in acromegaly due to hypermetabolism
BP 156/94 mmHg & glucose 186 mg/dL
GH excess leads to insulin resistance and HTN
Free thyroxine index low, TSH normal
Suggests secondary hypothyroidism
Low testosterone, normal LH
Suggests secondary hypogonadism
Low 8 am cortisol
Suggests secondary adrenal insufficiency (ACTH↓)
🎯 Unifying Diagnosis: Acromegaly
Caused by GH-secreting pituitary adenoma .
Tumor mass can compress surrounding pituitary tissue , leading to:
↓ TSH → secondary hypothyroidism
↓ ACTH → secondary adrenal insufficiency
↓ LH/FSH → secondary hypogonadism
✅ These pan-hypopituitarism signs , along with GH excess signs (coarse features, thick hands, glucose intolerance), strongly support acromegaly due to a pituitary macroadenoma .
❌ Why Other Options Are Incorrect:
Secondary hyperthyroidism – ❌
Secondary hypogonadism – ❌
True, but only one part of the clinical picture ; doesn’t explain visual defects, glucose intolerance, or acromegalic features.
Hyperpituitarism – ❌
If a patient has a hyperactive thyroid along with eye changes, what autoimmune disease uniquely explains both the thyroid and ocular symptoms together?
194 / 232
Tags:
2020
A 30-year-old woman presents to the clinic with palpitations, weight loss, and excessive sweating for almost 5-6 months. She cannot concentrate on carrying out her daily activities especially looking after her children. Recently, she also developed pain in both eyes and her friend told her that one of her eyes has become more prominent than the other. What is the most likely cause of her condition?
📌 Key Clinical Clues:
Symptom/Sign
Interpretation
Palpitations, sweating, weight loss
Hypermetabolic state → thyrotoxicosis
Difficulty concentrating, agitation
CNS effects of excess thyroid hormone
Eye pain + unilateral proptosis
Suggests Graves’ orbitopathy (specific!)
🔍 Why Graves’ Disease?
Graves’ disease is:
The most common cause of hyperthyroidism in young women
Autoimmune : antibodies stimulate the TSH receptor , causing diffuse thyroid hyperactivity
Uniquely associated with extrathyroidal manifestations , particularly:
This eye involvement is specific to Graves’ disease, and rules out other causes of hyperthyroidism.
❌ Why the Other Options Are Incorrect:
Generalized Anxiety Disorder – ❌
Can mimic hyperthyroid symptoms (e.g. palpitations, anxiety), but does not cause proptosis , sweating, or weight loss to this extent.
Hashimoto’s Thyroiditis – ❌
Usually causes hypothyroidism , not hyperthyroidism.
May rarely have an initial hyperthyroid phase (“Hashitoxicosis”), but no eye findings .
Hyperthyroidism – ❌
This is a condition , not a cause.
The question asks for most likely cause — and Graves’ disease is a cause of hyperthyroidism .
Toxic Nodular Goiter – ❌
Typically occurs in older patients
No eye signs , and the thyroid gland is nodular (not diffusely enlarged)
If a gland isn’t producing enough hormone, which upstream regulator ramps up production in response—and would show abnormal values before the hormones it controls do?
195 / 232
Tags:
2020
Which of the following is the most sensitive marker for the diagnosis of hypothyroidism?
📌 Understanding the Thyroid Feedback Loop:
The hypothalamus secretes TRH → stimulates the anterior pituitary to release TSH → stimulates the thyroid gland to produce T3 and T4 .
T3 and T4 exert negative feedback on both the pituitary and hypothalamus.
🔍 Why TSH is the Most Sensitive Marker:
In primary hypothyroidism (thyroid gland failure), T3 and T4 drop , and in response:
Therefore, an elevated TSH is the earliest and most sensitive indicator of primary hypothyroidism.
In subclinical hypothyroidism , T3 and T4 may still be normal , but TSH is already elevated .
❌ Why the Other Options Are Incorrect:
T3 – ❌
T4 – ❌
Free T3 – ❌
Similar to total T3, it’s not a reliable standalone marker for hypothyroidism.
More useful in hyperthyroidism assessment.
ACTH – ❌
Adrenocorticotropic hormone , related to the adrenal axis , not the thyroid axis.
Not relevant to thyroid function testing.
Which test reflects how “sugar-coated” your red blood cells have become over time, rather than just offering a one-time snapshot of glucose levels?
196 / 232
Tags:
2020
Which of the following is the most appropriate investigation for long-term monitoring of glucose levels?
📌 What is HbA1c?
HbA1c (glycated hemoglobin) reflects the average blood glucose over the past 2–3 months .
It is formed when glucose irreversibly binds to hemoglobin in red blood cells.
Since RBCs live ~120 days, HbA1c gives a long-term picture of glycemic control.
✅ Clinical Use of HbA1c:
Ideal for monitoring chronic glycemic control in patients with diabetes mellitus .
Helps assess effectiveness of treatment and risk of complications (e.g., nephropathy, neuropathy).
Normal HbA1c
< 5.7%
Pre-diabetes
5.7–6.4%
Diabetes
≥ 6.5%
❌ Why the Other Options Are Incorrect:
Random blood glucose – ❌
Reflects glucose at a single moment .
Useful for initial screening , but not reliable for long-term monitoring .
Oral glucose tolerance test (OGTT) – ❌
Used for diagnosing diabetes , especially in gestational diabetes or early prediabetes.
Not practical or useful for ongoing monitoring .
Fasting blood glucose – ❌
Also used for diagnosis , gives a snapshot, not a chronic trend .
Can vary day to day depending on diet, stress, sleep.
None of them – ❌
If a medication leads to less bone being built and more bone being broken down, which process more directly contributes to acute structural weakness that can lead to fractures?
197 / 232
Tags:
2020
A lady was taking oral glucocorticoids as per her doctor’s recommendation. After that, she went for surgery due to a hip fracture. Which of the following would be a cause of bone fracture?
📌 Glucocorticoid-Induced Osteoporosis:
Prolonged use of oral glucocorticoids is a well-known risk factor for osteoporosis and fragility fractures, especially in weight-bearing bones like the hip and spine .
Glucocorticoids affect bone in two major ways:
✅ Increase bone resorption :
They stimulate osteoclast activity , leading to increased breakdown of bone .
Calcium is released from bones into the blood → weakened bone matrix .
✅ Decrease bone formation :
So both resorption and decreased formation are involved, but if we must choose one primary mechanism leading to fracture , resorption of calcium from bone directly contributes to bone fragility .
❌ Why the Other Options Are Incorrect:
None of them – ❌
Incorrect. There is a well-established mechanism of bone loss due to glucocorticoids.
Osteoarthritis – ❌
This is a degenerative joint disease , primarily affecting cartilage and joint surfaces.
It does not directly lead to bone fractures .
Osteochondroma – ❌
Decreased bone formation – ❌
Consider which maternal condition leads to elevated fetal insulin in utero and how this metabolic mismatch can create an environment of overgrowth before birth and dangerous drops in glucose after birth.
198 / 232
Tags:
2020
A newborn was diagnosed with multiple congenital anomalies and was having seizures right after it was born with a glucose level of 19 mg/dl. What would be the disease the mother is suffering from to cause these defects in the child?
🧬 Clues from the Case:
Newborn with congenital anomalies : Suggests an intrauterine metabolic disturbance.
Seizures at birth : Often due to neonatal hypoglycemia .
Glucose = 19 mg/dL : Critically low; normal in newborns is typically ≥ 45 mg/dL.
📌 Link to Maternal Condition:
In gestational diabetes , the mother experiences hyperglycemia during pregnancy .
Glucose crosses the placenta , but insulin does not .
Fetal pancreas responds by increasing insulin production (hyperinsulinemia) .
After birth, maternal glucose supply is suddenly cut , but fetal insulin remains high , leading to profound hypoglycemia , which can cause seizures .
💥 Associated Neonatal Complications:
Macrosomia (large birth weight)
Neonatal hypoglycemia
Congenital anomalies (especially of the heart, spine, CNS)
Respiratory distress syndrome
❌ Why the Other Options Are Incorrect:
Diabetes of the young – ❌
Refers to MODY (Maturity Onset Diabetes of the Young) , a monogenic form of diabetes. It doesn’t typically cause severe maternal hyperglycemia during pregnancy nor the same fetal effects as gestational diabetes.
Adrenal carcinoma – ❌
A rare tumor. It may produce excess hormones, but it’s not a maternal condition typically associated with fetal hypoglycemia or birth defects .
Pancreatic endocrine tumor – ❌
This could cause maternal hypoglycemia (e.g., insulinoma), but wouldn’t cause hyperglycemia in the fetus or lead to neonatal hyperinsulinemia and congenital anomalies.
Diabetes mellitus type 2 – ❌
It can cause similar effects if present during pregnancy , but the question specifically hints at a pregnancy-related condition . Gestational diabetes is more precise, especially in previously non-diabetic women.
When a young individual presents with sluggish metabolism, always consider what hormone modulates the body’s basal metabolic rate. Would a deficit of this hormone explain both physical and cognitive slowing?
199 / 232
Tags:
2020
An 18-year-old female presents with weight gain, thinning of hair, low pulse and blood pressure, cold intolerance, clammy skin that is cold, constipation, and difficulty to concentrate. What would be the most probable diagnosis?
🧬 Key Symptoms in the Case:
Let’s decode each symptom:
Symptom
Explanation
Weight gain
Slowed metabolism → hallmark of hypothyroidism
Thinning of hair
Common due to decreased protein synthesis and skin turnover
Low pulse and BP (bradycardia, hypotension)
Hypothyroid state depresses cardiac output and heart rate
Cold intolerance, cold clammy skin
Decreased thermogenesis → classic hypothyroid sign
Constipation
GI motility is slowed in hypothyroid states
Difficulty concentrating (brain fog)
A well-known cognitive symptom in hypothyroidism
🔬 Conclusion:
All signs strongly point toward hypothyroidism , particularly the combination of cold intolerance, weight gain, bradycardia, cognitive slowing , and constipation .
❌ Why the Other Options Are Incorrect:
Hyperparathyroidism – ❌
Causes hypercalcemia , symptoms include bone pain, kidney stones, abdominal pain , and psychiatric disturbances (“stones, bones, groans, and moans”).
No strong overlap with this case.
Cushing syndrome – ❌
Associated with weight gain , yes, but also:
Cold intolerance and bradycardia are not features.
Hyperthyroidism – ❌
Hypoparathyroidism – ❌
When evaluating a patient with signs of cortisol excess, consider whether the body is making too much—or if we are supplying it. In today’s medical landscape, think about which sources of hormones are most frequently introduced artificially.
200 / 232
Tags:
2020
Which of the following is the most common cause of Cushing’s syndrome?
📌 Understanding Cushing’s Syndrome:
Cushing’s syndrome is a clinical condition caused by prolonged exposure to high levels of glucocorticoids , particularly cortisol . It can be endogenous or exogenous .
📚 Classification of Causes:
🔹 Exogenous (Most Common Overall):
Glucocorticoid therapy (e.g., for autoimmune or inflammatory conditions like asthma, RA, lupus)
This is the most common cause of Cushing’s syndrome in clinical practice.
It suppresses ACTH and CRH via negative feedback.
🔹 Endogenous (Less Common):
Cushing’s disease → Pituitary adenoma secreting ACTH (about 60–70% of endogenous cases)
Adrenal adenoma/carcinoma → Autonomous cortisol production
Ectopic ACTH production → e.g., small cell lung carcinoma
❌ Why the Other Options Are Incorrect:
Iatrogenic cause – ❌
Technically correct, but too vague . “Iatrogenic” just means “caused by medical treatment.” Glucocorticoid therapy is the specific iatrogenic cause , making it the better choice.
Pituitary adenoma – ❌
Correct only for endogenous Cushing’s (called Cushing’s disease ), but not the most common overall cause .
Adrenal carcinoma – ❌
Rare and usually causes rapidly progressive Cushing’s. Not a common cause.
Adrenal cortex adenoma – ❌
More common than carcinoma but still not the leading cause overall . Causes ACTH-independent Cushing’s.
When thinking about immune-related causes of thyroid dysfunction, consider which autoantibodies are destructive versus stimulatory, and whether their net effect would lead to a slowing down or speeding up of metabolism.
201 / 232
Tags:
2020
Which of the following is the most common etiology of hypothyroidism?
🧬 Understanding Hypothyroidism:
Hypothyroidism is a clinical condition characterized by insufficient thyroid hormone production , leading to symptoms like fatigue, weight gain, cold intolerance, and bradycardia.
The most common cause of hypothyroidism in iodine-sufficient areas is autoimmune thyroiditis , specifically Hashimoto’s thyroiditis .
🔍 Key Mechanism: Hashimoto’s Thyroiditis
Hashimoto’s is an autoimmune condition where the immune system attacks the thyroid.
The hallmark autoantibodies include:
✅ Anti-thyroid peroxidase (TPO) antibodies → most common and sensitive marker .
Anti-thyroglobulin (Tg) antibodies → less sensitive, often co-exist with anti-TPO.
Over time, chronic inflammation and destruction of thyroid tissue leads to permanent hypothyroidism .
❌ Why the Other Options Are Incorrect:
Anti-TSH receptor antibodies – ❌
These are typically associated with Graves’ disease , an autoimmune hyperthyroidism , not hypothyroidism. They stimulate the TSH receptor, leading to increased hormone production.
Anti-thyroglobulin antibodies – ❌
These are also found in Hashimoto’s but are less common and less specific than anti-TPO antibodies.
None of them – ❌
Incorrect because anti-TPO antibodies are very much responsible and commonly present in the most frequent cause of hypothyroidism.
Radioactive thyroid scan – ❌
This is a diagnostic imaging tool , not a cause. It helps in evaluating thyroid nodules or function (e.g., cold/hot nodules), but it is not an etiology .
Which triad of symptoms results from a pituitary tumor that increases a hormone whose primary physiologic role is to prepare the body for lactation?
202 / 232
Which mode of inheritance typically requires both parents to be carriers, often presenting early in life when both enzyme copies are non-functional?
203 / 232
Tags:
2020
Which of the following is incorrect regarding congenital adrenal hyperplasia (CAH)?
🔹 What is Congenital Adrenal Hyperplasia (CAH)?
Congenital Adrenal Hyperplasia is a group of autosomal recessive genetic disorders affecting the enzymes involved in cortisol synthesis in the adrenal cortex — most commonly 21-hydroxylase deficiency .
🔹 Key Features of CAH:
Genetics:
Hormonal Imbalance:
Cortisol deficiency (due to enzyme defect)
Leads to increased ACTH secretion (due to lack of negative feedback)
Causes adrenal hyperplasia and increased androgen synthesis
Clinical Manifestations:
Virilization or masculinization in genetically female (XX) infants
Salt-wasting crisis in severe forms (if aldosterone is also deficient)
Precocious puberty in males
🔹 Why the Other Options Are Correct:
Option
True?
Explanation
Masculinization of female genitalia
✅ Yes
Due to excess adrenal androgens (e.g., DHEA)
Autosomal recessive disorder
✅ Yes
Classic inheritance pattern of CAH
Increases in adrenal androgens
✅ Yes
Shunting of precursors toward androgen synthesis
None of these
❌ No
One of the listed statements (autosomal dominant) is incorrect
🔹 Why “Autosomal Dominant Disorder” Is Incorrect:
CAH is not dominant ; both alleles must be mutated to express the disease.
Carriers (one defective allele) are usually asymptomatic.
Thus, calling CAH an autosomal dominant disorder is a factual error .
Which signaling molecule, when constitutively activated by a mutation, bypasses normal receptor regulation and leads to unchecked hormone secretion and cell growth?
204 / 232
Tags:
2020
Genetic abnormalities are associated with pituitary adenomas. Which of the following is the most common genetic abnormality in pituitary adenomas?
🔹 Understanding Pituitary Adenomas:
Pituitary adenomas are benign tumors of the anterior pituitary.
They are classified based on the type of hormone they secrete (e.g., prolactinomas, somatotroph adenomas causing acromegaly, etc.).
While often sporadic, many have identifiable genetic mutations that drive their formation and hormone overproduction.
🔹 Most Common Genetic Abnormality: G-Protein Mutation
The most common mutation in pituitary adenomas involves the Gs alpha subunit of the G-protein (encoded by the GNAS1 gene ).
This mutation causes constitutive activation of the Gs protein, leading to:
✅ This is especially common in somatotroph adenomas (GH-secreting tumors).
🔹 Why the Other Options Are Incorrect:
❌ Defect in retinoblastoma (Rb gene)
❌ cAMP formation mutation
❌ GDP formation mutation
GDP is involved in G-protein cycling, but there’s no known GDP synthesis mutation linked to pituitary adenomas.
Again, the problem is the Gs protein’s inability to hydrolyze GTP due to mutation.
❌ None of them
🔹 Summary Table:
Option
Involved in Pituitary Adenomas?
Correct?
Explanation
G-protein mutation
✅ Yes
✅ Yes
Most common genetic abnormality (GNAS1)
Defect in retinoblastoma
❌ No
❌ No
Seen in other tumors (e.g., retinoblastoma)
cAMP formation mutation
❌ Indirect
❌ No
cAMP is upregulated, but not mutated
GDP formation mutation
❌ No
❌ No
Not a recognized mechanism
None of them
❌ No
❌ No
G-protein mutation is the correct one
Ask yourself:Which organ system suffers the most lethal long-term consequences of chronic hyperglycemia, and what complications are the most irreversible? Focus on macrovascular complications for causes of mortality , and microvascular complications for morbidity .
205 / 232
Tags:
2016
What is the most common cause of death in patients with diabetes mellitus?
Diabetes mellitus — especially type 2 — is a major risk factor for cardiovascular disease, and cardiovascular complications are the leading cause of death in diabetic patients. The long-term effects of chronic hyperglycemia include damage to both small and large blood vessels (microvascular and macrovascular complications), but it’s the macrovascular disease that proves most fatal.
✅ Why “Heart disease” Is Correct:
Diabetics have 2–4 times higher risk of developing coronary artery disease (CAD) .
Long-standing diabetes leads to accelerated atherosclerosis , causing:
Many patients with diabetes also have co-existing hypertension , dyslipidemia , and obesity , which further compound cardiac risk.
Silent ischemia (MI without typical chest pain) is also more common in diabetics, leading to delayed diagnosis and worse outcomes.
Thus, ischemic heart disease is the most common cause of mortality in diabetics.
❌ Why the Other Options Are Incorrect:
Wound infection and sepsis : While diabetics are prone to poor wound healing and infections , especially foot ulcers , these are more commonly causes of morbidity , not the most common cause of death .
Stroke : Stroke risk is increased in diabetes, but heart disease still accounts for more deaths than stroke in this population.
Hypoglycemia : A possible acute complication (especially from insulin or sulfonylureas), but less common as a cause of death than chronic cardiovascular disease.
Renal failure : Diabetic nephropathy is a common cause of end-stage renal disease , but cardiac events remain the leading cause of death, even in patients on dialysis.
Ask yourself:
Does this complication primarily involve small capillaries or large arteries? That distinction will guide you to whether it’s microvascular or macrovascular .
206 / 232
Tags:
2016
Which of the following is not a microvascular complication of diabetes?
Diabetes mellitus leads to long-term complications due to chronic hyperglycemia , which damages blood vessels . These complications are broadly classified into:
Understanding the type of vessel affected helps you classify the complication correctly.
✅ Why “Stroke” Is Correct:
Stroke is a result of atherosclerosis and thromboembolism affecting large cerebral arteries (e.g., carotids, vertebrobasilar system).
Therefore, it is a macrovascular complication of diabetes.
It occurs due to accelerated atherosclerosis , often in combination with hypertension and dyslipidemia in diabetic patients.
❌ Why the Other Options Are Microvascular Complications:
Neuropathy :
Caused by ischemia of vasa nervorum (tiny blood vessels supplying nerves)
Includes peripheral, autonomic, and focal neuropathies
Retinopathy :
Results from damage to retinal capillaries
Leads to microaneurysms , hemorrhages , and neovascularization
Nephropathy :
Due to glomerular capillary damage
Leads to albuminuria , glomerulosclerosis , and ultimately chronic kidney disease
Cataracts :
While not purely vascular, they result from osmotic damage in lens proteins due to sorbitol accumulation (via the polyol pathway)
Often associated with chronic hyperglycemia , and considered a non-classic microvascular complication
When considering muscle weakness, ask: Could this be due to disrupted ion gradients?
207 / 232
Tags:
2016
Which of the following causes muscle weakness in Conn’s syndrome?
🔹 What Is Conn’s Syndrome?
Conn’s syndrome refers to primary hyperaldosteronism — an endocrine disorder characterized by excessive aldosterone production , most commonly from an adrenal adenoma .
Aldosterone acts on the distal nephron in the kidney to:
Increase Na⁺ reabsorption → hypertension
Promote K⁺ excretion → hypokalemia
Promote H⁺ excretion → metabolic alkalosis
✅ Why Hypokalemia Is Correct:
Hypokalemia is a classic feature of Conn’s syndrome.
Low potassium levels impair neuromuscular excitability , resulting in:
Potassium is essential for maintaining resting membrane potential , especially in skeletal and cardiac muscle.
❌ Why the Other Options Are Incorrect:
Hyperkalemia: This would cause increased excitability , not weakness — and is the opposite of what happens in Conn’s.
Acidosis: Conn’s syndrome actually causes alkalosis (due to H⁺ excretion). Acidosis is not a feature here.
Hyponatremia: You’d expect mild hypernatremia or normal sodium due to aldosterone’s sodium-retaining effect. Hyponatremia is not characteristic of Conn’s syndrome.
None of these: Incorrect because hypokalemia clearly contributes to muscle weakness .
When a question asks for the “most common” cause of an endocrine disorder, always consider: Is this typically a sporadic, benign condition or part of a rare genetic syndrome? Most commonly, it’s a single-gland adenoma .
208 / 232
Tags:
2016
Which of the following is the most common parathyroid tumor?
🔹 Primary Hyperparathyroidism:
This condition is caused by excessive secretion of parathyroid hormone (PTH) , leading to:
Hypercalcemia
Hypophosphatemia
Bone resorption
Renal stones, bone pain, abdominal discomfort (“bones, stones, groans, and psychiatric overtones”)
✅ Why Parathyroid Adenoma Is Correct:
Parathyroid adenoma is the most common cause of primary hyperparathyroidism , accounting for approximately 85–90% of cases.
It is a benign tumor of one parathyroid gland that causes unregulated PTH secretion .
Typically presents sporadically , but may also be seen in familial syndromes like MEN1 .
❌ Why the Other Options Are Incorrect:
Multiple Endocrine Neoplasia Type 1 (MEN1): Involves the parathyroid glands , pancreas, and pituitary. While MEN1 can cause parathyroid hyperplasia , it accounts for <5% of parathyroid tumors.
Multiple Endocrine Neoplasia Type 2 (MEN2): MEN2 typically involves medullary thyroid carcinoma, pheochromocytoma , and parathyroid hyperplasia , but parathyroid involvement is less frequent than in MEN1.
Parathyroid hyperplasia: Accounts for about 10–15% of cases of primary hyperparathyroidism — often involves all four glands and is sometimes familial (e.g., MEN syndromes), but less common than adenoma.
Parathyroid carcinoma: Very rare (less than 1% of cases). Though it may cause severe hypercalcemia , it’s not a common tumor.
If a disease has many rare genetic or autoimmune forms, always ask: What is the most frequent and preventable cause in a clinical setting? Often, the answer is iatrogenic (surgery-related).
209 / 232
Tags:
2016
Hypoparathyroidism most commonly occurs secondary to which of the following?
🔹 What Is Hypoparathyroidism?
Hypoparathyroidism is characterized by low parathyroid hormone (PTH) levels , leading to:
✅ Why Surgery-Induced Hypoparathyroidism Is Most Common:
The most frequent cause of hypoparathyroidism is accidental removal or damage to the parathyroid glands during neck surgeries , particularly:
Thyroidectomy
Parathyroidectomy
Radical neck dissection
This iatrogenic cause is far more common than genetic or autoimmune forms.
Surgeons now try to identify and preserve the parathyroid glands during surgery to avoid this complication.
❌ Why the Other Options Are Incorrect:
Congenital absence of the parathyroid glands: Occurs in rare syndromes like DiGeorge syndrome , but not the most common cause overall.
Familial isolated hypoparathyroidism: A rare genetic condition , not a common cause.
Autosomal dominant hypoparathyroidism: Another inherited form , seen in familial syndromes — again, rare .
Autoimmune hypoparathyroidism: Seen in conditions like Autoimmune Polyendocrine Syndrome Type 1 (APS-1) — important, but less common than post-surgical hypoparathyroidism.
When distinguishing endocrine emergencies, ask: Is this condition causing the body to “rev up” or “slow down”? Storms are always about hyperactivity — think heat, speed, and danger.
210 / 232
Tags:
2016
Thyroid storm is seen in which of the following diseases?
🔹 What is Thyroid Storm?
Thyroid storm is a life-threatening, hypermetabolic state caused by excessive thyroid hormone activity .
It is an extreme form of thyrotoxicosis and presents with:
High fever
Tachycardia or arrhythmias
Agitation, delirium, or coma
GI symptoms (nausea, vomiting, diarrhea)
It’s usually triggered by acute stress (infection, surgery, trauma) in someone with untreated or poorly controlled hyperthyroidism .
✅ Why Grave’s Disease Is the Correct Answer:
Grave’s disease is the most common cause of hyperthyroidism and therefore the most common underlying condition in thyroid storm .
It is an autoimmune disorder where TSH receptor antibodies stimulate the thyroid to produce excessive hormone.
In the setting of additional stress, this can precipitate thyroid storm .
❌ Why the Other Options Are Incorrect:
Hashimoto’s thyroiditis: This is a cause of hypothyroidism , not hyperthyroidism. No thyroid storm here — though early stages may have transient thyrotoxicosis.
Wolff-Chaikoff effect: This is a protective autoregulatory mechanism where excess iodine inhibits thyroid hormone synthesis — it actually reduces thyroid activity.
Myxedema coma: The opposite of thyroid storm — a severe hypothyroid emergency characterized by hypothermia, bradycardia, hypotension , and altered mental status.
Hypothyroidism: Chronic low thyroid function — does not cause storm; rather, untreated severe cases may lead to myxedema coma .
What lies just beneath the optic chiasm and can, when enlarged, create a constellation of visual and neurologic symptoms over weeks to months?
211 / 232
Tags:
2020
A 25-year-old male comes to the outpatient department with complaints of intermittent headache and nausea for 4 months. On examination of the eyes, he has slightly impaired vision and diplopia. What is the most appropriate diagnosis for this patient?
1. Key symptoms and localization:
Intermittent headache and nausea → suggest increased intracranial pressure or mass effect .
Impaired vision and diplopia → point toward optic pathway or cranial nerve compression .
These symptoms together strongly suggest a space-occupying lesion in the sellar/suprasellar region .
2. Why Pituitary Adenoma fits best:
Pituitary adenomas are tumors that arise from the anterior pituitary .
As they enlarge, especially macroadenomas (>10 mm), they compress surrounding structures:
Optic chiasm → causes bitemporal hemianopia or blurred vision.
Cavernous sinus → affects cranial nerves III, IV, VI , leading to diplopia .
Headaches occur due to stretching of the dura or pressure on adjacent tissues.
The slow progression of symptoms fits the gradual growth of a pituitary tumor.
✅ This matches the clinical picture very well.
3. Why the other options are incorrect:
🔹 Hypothalamic ischemia
Would present more acutely with autonomic , endocrine , or thermoregulatory dysfunction.
Not typically associated with diplopia or chronic headaches.
❌ Doesn’t match presentation.
🔹 Corneal infection
Causes eye pain, redness, photophobia , and possibly blurred vision , but not diplopia or headache .
❌ Local eye disease only.
🔹 Ophthalmic inflammation
Could cause visual changes but typically includes pain, redness, photophobia .
Doesn’t explain intracranial symptoms like nausea and headache.
❌ Doesn’t explain systemic features.
🔹 Retinal damage
Would cause visual field defects , floaters, or vision loss.
No diplopia or headache , and unrelated to nausea .
❌ Too localized and sudden.
Summary Table:
Condition
Headache + Nausea
Visual Symptoms
Diplopia
Fits Case?
Pituitary adenoma
✅ Yes
✅ Yes
✅ Yes
✅ Yes
Hypothalamic ischemia
⚠️ Rarely
❌ No
❌ No
❌ No
Corneal infection
❌ No
✅ (local only)
❌ No
❌ No
Ophthalmic inflammation
❌ No
✅ (local only)
❌ Rare
❌ No
Retinal damage
❌ No
✅ Yes
❌ No
❌ No
Which condition arises due to placental hormone-induced insulin resistance during pregnancy and can affect the fetus through increased nutrient delivery, leading to excessive fetal growth and postnatal complications?
212 / 232
Tags:
2020
A pregnant woman, with no previous history of diabetes, started developing hyperglycemia, coupled with polyuria and glycosuria in her 2nd trimester. She is brought to the emergency room on the 29th week of gestation and an emergency caesarian is performed. Her baby is born overweight and develops seizures immediately after birth. Which of the following is the most likely reason for the baby’s condition?
1. Key clinical clues:
No prior history of diabetes → suggests the diabetes developed during pregnancy .
Second trimester onset → classic for gestational diabetes mellitus (GDM) .
Overweight baby (macrosomia) → common fetal outcome due to maternal hyperglycemia .
Neonatal seizures → strongly suggest hypoglycemia after birth , a known complication of GDM.
2. Mechanism behind neonatal complications in GDM:
Maternal hyperglycemia → glucose crosses placenta → fetus develops hyperinsulinemia .
Insulin is a growth-promoting hormone → leads to macrosomia (large baby).
After birth, the maternal glucose supply stops, but fetal insulin remains high .
This causes rapid drop in blood glucose → neonatal hypoglycemia , leading to seizures .
✅ This is a hallmark presentation of complications from gestational diabetes .
3. Why the other options are incorrect:
🔹 Maturity Onset Diabetes of the Young (MODY)
A rare monogenic form of diabetes, typically diagnosed in adolescents or young adults .
Not typically related to pregnancy.
❌ Doesn’t fit mother’s or baby’s clinical profile.
🔹 Type 2 Diabetes Mellitus
Often diagnosed before pregnancy , not in mid-second trimester without prior symptoms.
Insulin resistance in pregnancy is physiological, and if exaggerated, leads to GDM , not classic T2DM.
❌ Incorrect because mother had no prior diabetes diagnosis.
🔹 Type 1 Diabetes Mellitus
Autoimmune disease with absolute insulin deficiency .
Usually presents in childhood or adolescence, not in second trimester of pregnancy.
❌ Doesn’t fit timing or presentation.
🔹 Neonatal diabetes
A rare condition where the baby has diabetes, either transient or permanent.
Presents as hyperglycemia in the newborn , not hypoglycemia or seizures.
❌ Does not explain macrosomia or seizures due to low blood sugar .
Summary Table:
Option
Fits maternal case?
Fits neonatal findings?
Most likely?
MODY
❌ No
❌ No
❌ No
Type 2 DM
❌ Unlikely
❌ No
❌ No
Type 1 DM
❌ No
❌ No
❌ No
Gestational diabetes
✅ Yes
✅ Yes
✅ Yes
Neonatal diabetes
❌ No
❌ No
❌ No
Which hormone reflects the pituitary’s real-time reaction to low thyroid hormone levels and becomes abnormal first, even when others remain in range?
213 / 232
Tags:
2020
Which of the following is the most sensitive test for myxedema?
1. What is myxedema?
Myxedema refers to severe hypothyroidism , often with systemic manifestations (fatigue, cold intolerance, constipation, weight gain, puffiness, bradycardia, etc.).
In primary hypothyroidism (thyroid failure), the pituitary responds by secreting more TSH to stimulate the underactive thyroid.
Therefore, TSH is elevated even before T3 or T4 levels drop significantly.
2. Why TSH is the most sensitive test:
TSH rises early , even with mild reductions in thyroid function, before free T3 or T4 drop below normal.
It is the first lab abnormality seen in subclinical hypothyroidism .
Used as a screening test and for monitoring treatment .
In myxedema (severe primary hypothyroidism), TSH is markedly elevated .
✅ Hence, TSH is the most sensitive and reliable marker .
3. Why the other options are incorrect or less sensitive:
🔹 Triiodothyronine (T3)
🔹 Thyrotropin-releasing hormone (TRH)
Produced by the hypothalamus .
TRH stimulation test is rarely used and not practical .
TSH is preferred over TRH for sensitivity and ease.
✅ ❌ Not routinely measured.
🔹 Thyroxine (T4)
Total T4 includes bound hormone, which can vary with protein levels.
May remain in normal range in early disease.
✅ ❌ Less sensitive than TSH.
🔹 Free T3
Summary Table:
Test
Measures
Sensitivity for Hypothyroidism
Most Sensitive?
TSH
Pituitary response
✅ Rises early
✅ Yes
T3
Active hormone
❌ Changes late
❌ No
TRH
Hypothalamic hormone
❌ Rarely used clinically
❌ No
T4
Prohormone
❌ Less sensitive
❌ No
Free T3
Unbound T3
❌ Poor early marker
❌ No
When antibodies are present that stimulate hormone production, consider how this differs from conditions where the gland is simply inflamed or damaged.
214 / 232
Tags:
2020
A 40-year-old woman presents with 5 kg weight loss over 4 months, heat intolerance, and palpitations. On examination, there is diffuse thyroid enlargement. Serology shows TSH receptor antibodies. Which of the following is the correct diagnosis?
1. Thyrotoxic symptoms:
Weight loss , heat intolerance , and palpitations are classic signs of hyperthyroidism (thyrotoxicosis).
The thyroid is diffusely enlarged — another common finding in Grave’s disease .
2. Key diagnostic clue:
TSH receptor antibodies are specific for Grave’s disease .
These stimulating autoantibodies activate the TSH receptor, causing increased production of thyroid hormones and gland growth.
✅ This lab result confirms the diagnosis of Grave’s disease .
3. Why other options are incorrect:
🔹 Hashimoto’s thyroiditis
An autoimmune hypothyroid condition.
Often presents with weight gain , cold intolerance, fatigue.
Positive antibodies: anti-TPO , anti-thyroglobulin , not TSH receptor antibodies.
✅ ❌ Not consistent with symptoms or labs.
🔹 De Quervain’s thyroiditis (Subacute granulomatous thyroiditis)
Painful, tender thyroid gland.
Often follows a viral illness.
Transient hyperthyroid phase possible, but no TSH receptor antibodies .
✅ ❌ Incorrect presentation.
🔹 Palpation thyroiditis
Mild, transient hyperthyroidism caused by repeated physical manipulation of the thyroid.
No autoimmune antibodies , and not associated with diffuse gland enlargement.
✅ ❌ Doesn’t fit serology or symptom pattern.
🔹 Lymphocytic thyroiditis
Summary Table:
Diagnosis
Key Features
TSH Receptor Antibodies
Fits Case?
Grave’s disease
Hyperthyroid symptoms, diffuse goiter, positive antibodies
✅ Yes
✅ Yes
Hashimoto’s thyroiditis
Hypothyroid symptoms, anti-TPO antibodies
❌ No
❌ No
De Quervain’s thyroiditis
Painful thyroid, viral prodrome, transient hyperthyroid
❌ No
❌ No
Palpation thyroiditis
Transient, mild hyperthyroid due to pressure
❌ No
❌ No
Lymphocytic thyroiditis
Painless, postpartum or autoimmune, transient
❌ No
❌ No
If a hormone-producing organ is under autoimmune attack, consider which enzymes or proteins essential to hormone synthesis might become immune targets.
215 / 232
Tags:
2021
A 35-year-old female comes to the clinic with symptoms of hypothyroidism. The physician is concerned about thyroid auto-antibodies. Which antibody is most commonly observed in Hashimoto thyroiditis?
🔬 Hashimoto Thyroiditis Overview:
Most common cause of hypothyroidism in iodine-sufficient areas
Autoimmune destruction of the thyroid gland
More common in females , typically between 30–50 years
Histology shows lymphocytic infiltration , Hurthle cells , and germinal centers
🧪 Key Autoantibodies in Hashimoto’s:
Anti-thyroid peroxidase (anti-TPO) antibodies
Target the TPO enzyme , which is essential for iodination of thyroglobulin
Most sensitive and most common marker for Hashimoto thyroiditis
Found in >90% of patients
Anti-thyroglobulin antibodies
Anti-follicular cell antibodies
❌ Why the Other Options Are Incorrect:
Anti-follicular antibodies : ❌ Non-specific; not commonly used in diagnosis
None of these : ❌ Incorrect—anti-TPO antibodies are a hallmark of the disease
Anti-cytoplasmic antibodies : ❌ Typically associated with vasculitides (e.g., ANCA), not Hashimoto’s
Anti-nuclear antibodies (ANA) : ❌ Seen in systemic autoimmune diseases (e.g., SLE); not specific to thyroid pathology
Consider the mode of inheritance and whether a disorder requires one or two mutated gene copies to manifest. How might this influence family patterns and genetic counseling?
216 / 232
Tags:
2021
Which one of these is an autosomal recessive condition?
1. Understanding inheritance patterns
2. Review of each disorder
🔹 MEN-2 syndrome (Multiple Endocrine Neoplasia type 2)
Inheritance: Autosomal dominant
Gene: RET proto-oncogene mutations
Features: Medullary thyroid carcinoma, pheochromocytoma, hyperparathyroidism (in MEN-2A)
❌ Not autosomal recessive.
🔹 Neurofibromatosis type 1
Inheritance: Autosomal dominant
Gene: NF1 gene mutation
Features: Café-au-lait spots, neurofibromas, Lisch nodules
❌ Not autosomal recessive.
🔹 MEN-1 syndrome
Inheritance: Autosomal dominant
Gene: MEN1 gene mutation
Features: Tumors of parathyroid, pancreatic islets, pituitary gland
❌ Not autosomal recessive.
🔹 Von Hippel-Lindau syndrome
Inheritance: Autosomal dominant
Gene: VHL gene mutation
Features: Hemangioblastomas, renal cell carcinoma, pheochromocytomas
❌ Not autosomal recessive.
🔹 Autoimmune polyglandular syndrome type I (APS I)
Inheritance: Autosomal recessive
Gene: AIRE gene mutation
Features: Chronic mucocutaneous candidiasis, hypoparathyroidism, adrenal insufficiency
✅ This is a classic autosomal recessive disorder.
Summary Table:
Disorder
Inheritance Pattern
Autosomal Recessive?
MEN-2 syndrome
Autosomal dominant
No
Neurofibromatosis type 1
Autosomal dominant
No
MEN-1 syndrome
Autosomal dominant
No
Von Hippel-Lindau syndrome
Autosomal dominant
No
Autoimmune polyglandular syndrome type I
Autosomal recessive
Yes
Think about which test provides definitive proof of the actual organism’s presence rather than just detecting its activity or associated changes.
217 / 232
Tags:
2021
Which of the following is the gold standard for diagnosing Helicobacter pylori?
1. What is meant by “gold standard”?
2. Analyzing each diagnostic test:
🔹 Microbiological culture
Involves growing H. pylori from gastric biopsy specimens on selective media.
Provides definitive proof of infection.
Allows antibiotic susceptibility testing , guiding treatment.
Highest specificity and sensitivity when done correctly.
Considered the gold standard , but it’s technically difficult, time-consuming, and requires special labs.
🔹 Histopathology
Biopsy tissue examined under a microscope after special staining.
Visualizes H. pylori and the associated gastritis.
High sensitivity and specificity but not as definitive as culture .
Cannot test antibiotic sensitivity.
🔹 Urea Breath Test (UBT)
Non-invasive, detects active infection by measuring labeled CO2 after ingestion of labeled urea.
Very sensitive and specific.
Widely used clinically for diagnosis and post-treatment follow-up.
However, it is an indirect test (detects enzyme activity, not the organism itself).
🔹 Urease test (Rapid urease test)
🔹 None of these
Summary Table:
Test
Direct/Indirect
Sensitivity/Specificity
Gold Standard?
Microbiological culture
Direct
Highest
✅ Yes
Histopathology
Direct
High
No
Urea breath test
Indirect
Very high
No
Urease test
Indirect
Good
No
None of these
—
—
No
Think about the age of onset and the fundamental defect in sugar metabolism for the most common adult-onset form of diabetes.
218 / 232
Tags:
2021
A 65-year-old woman comes into the outpatient department suffering from polyuria and polydipsia. She looks dehydrated and her skin has lost some turgor. Lab tests show glucosuria +4. What is the most important pathogenesis of this disease?
1. Clinical picture analysis:
The patient is elderly (65 years old).
Symptoms: polyuria, polydipsia, dehydration → classic diabetes mellitus symptoms .
Lab shows glucosuria +4 , indicating high blood glucose spilling into urine.
The age and presentation suggest Type 2 Diabetes Mellitus (T2DM) , which usually develops later in life.
2. Pathogenesis of diabetes types:
🔹 Type 1 Diabetes Mellitus (T1DM)
Autoimmune destruction of pancreatic beta cells (Insulitis).
Associated with HLA-DR3 and HLA-DR4 alleles.
Usually occurs in younger individuals.
Leads to absolute insulin deficiency .
Therefore, Insulitis and HLA-DR4/DR3 relate to T1DM.
🔹 Type 2 Diabetes Mellitus (T2DM)
Characterized primarily by insulin resistance in peripheral tissues (muscle, fat, liver).
There may be a relative insulin deficiency later due to beta-cell dysfunction.
Usually presents in adults over 40.
Strongly linked to obesity and metabolic syndrome.
Peripheral resistance is the key initial pathogenesis in T2DM.
🔹 Glucokinase gene mutation
Causes a rare form of diabetes called Maturity Onset Diabetes of the Young (MODY 2) .
Typically presents early in life.
Not common in elderly patients.
🔹 None of these
3. Why is Peripheral resistance correct?
The patient’s age and clinical features are typical of T2DM .
In T2DM, the most important initial defect is insulin resistance in peripheral tissues, meaning the body’s cells do not respond adequately to insulin.
This leads to hyperglycemia and symptoms like polyuria and polydipsia.
Summary Table:
Option
Disease Association
Correct for this Case?
Peripheral resistance
Type 2 DM
✅ Yes
Insulitis
Type 1 DM
❌ No
HLA-DR4 / HLA-DR3
Type 1 DM
❌ No
Glucokinase gene mutation
MODY 2
❌ No
None of these
—
❌ No
Which enzyme is central to both glucocorticoid and mineralocorticoid synthesis , and whose deficiency would cause both hormonal insufficiency and androgen excess ? Think about where a metabolic “traffic jam” would cause the biggest hormonal detour.
219 / 232
Tags:
2016
What is the most common enzyme defect in congenital adrenal hyperplasia?
🔹 What is Congenital Adrenal Hyperplasia (CAH)?
CAH is a group of autosomal recessive disorders involving defects in cortisol biosynthesis in the adrenal cortex. When cortisol synthesis is impaired, there’s a loss of negative feedback on the hypothalamic-pituitary-adrenal (HPA) axis. This leads to increased ACTH , causing adrenal hyperplasia and overproduction of other steroid hormones proximal to the enzymatic block .
🔹 21-Hydroxylase Deficiency — The Most Common Form:
🔸 Blocked Cortisol + Aldosterone Synthesis:
↓ Cortisol → ↑ ACTH → adrenal hyperplasia
↓ Aldosterone → salt-wasting , hypotension, dehydration
🔸 Shunting to Androgens:
✅ Why 21-Hydroxylase is the Correct Answer:
It’s the most frequently mutated enzyme in CAH.
Presents with salt-wasting, hypotension, and hyperandrogenism .
Confirmed by elevated 17-hydroxyprogesterone levels .
❌ Why the Other Options Are Incorrect:
17α-Hydroxylase:
Leads to ↓ cortisol and sex steroids , but ↑ mineralocorticoids .
Causes hypertension , hypokalemia , and sexual infantilism .
Rare form of CAH.
11β-Hydroxylase:
Leads to excess deoxycorticosterone , causing hypertension and virilization .
Second most common (~5-8%) but still less common than 21-hydroxylase .
17,20-lyase:
Involved in androgen synthesis , rare mutation.
Results in impaired sexual development , but not classic CAH.
18-Hydroxylase:
Involved in aldosterone synthesis .
Not typically classified under CAH; associated with hypoaldosteronism , not virilization or cortisol defects.
When a female neonate shows signs of masculinization, consider which step in steroid synthesis—if blocked—would divert the pathway toward androgen overload rather than cortisol completion.
220 / 232
Tags:
2021
Physical examination of a neonate shows a markedly virilized and hypertrophic clitoris with partial fusion of the labioscrotal folds in a female having 46, XX karyotype. Laboratory studies will most likely reveal a deficiency of which of the following?
🔬 21-Hydroxylase Deficiency:
In females:
Results in virilization of the external genitalia , despite normal internal organs (ovaries, uterus)
Ambiguous genitalia is the typical presentation at birth
❌ Why the Other Options Are Incorrect:
❌ Corticotropin (ACTH) :
❌ Prolactin :
❌ Progesterone :
An intermediate in steroid synthesis, but not typically deficient in CAH.
In fact, some CAH variants show elevated progesterone .
❌ Androstenedione :
In societies with abundant iodine and healthcare access, when the thyroid slows down, it’s often not from lack of resources—but from the body turning against itself.
221 / 232
Tags:
2021
Hypothyroidism is caused by structural or functional derangement that interferes with thyroid hormone production. Which of the following statements regarding hypothyroidism is correct?
Hypothyroidism is a clinical condition resulting from deficiency of thyroid hormones (T3 and T4) , which leads to a general slowing down of metabolic processes.
🔬 Common Causes :
🌍 Worldwide :
🌎 Developed countries :
❌ Why the Other Options Are Incorrect:
❌ “In infants, it results in myxedema” :
In infants , hypothyroidism leads to cretinism , not myxedema.
Myxedema refers to severe hypothyroidism in adults , presenting with non-pitting edema and characteristic facial puffiness .
❌ “Most common cause in iodine deficient areas of Pakistan is Graves’ disease” :
Graves’ disease is a cause of hyperthyroidism , not hypothyroidism.
In iodine-deficient regions like parts of Pakistan, the most common cause of hypothyroidism is endemic goiter due to iodine deficiency , not Graves’ .
❌ “Thyroid ophthalmopathy with proptosis is a common feature” :
❌ “Throughout the world, hypothyroidism is associated with zinc deficiency” :
Zinc deficiency may affect thyroid function , but it is not a primary global cause of hypothyroidism.
Iodine deficiency , autoimmune thyroiditis , and post-ablative causes (like surgery or radioactive iodine) are more significant.
When extreme hyperglycemia leads to dehydration and altered mental status but without the fruity smell of ketones or acidotic breathing, ask yourself: which diabetic crisis quietly disrupts the brain by pulling water out instead of acid in?
222 / 232
Tags:
2021
A 61-year-old diabetic comes into the emergency room. His laboratory results shows a random blood sugar of 890 mg/dL and serum osmolarity of 460 mOsm. The patient is dehydrated and confused. What is this condition known as?
🔬 What is HHNS (or HHS)?
Hyperosmolar Hyperglycemic Nonketotic Syndrome (HHS) (also called hyperosmolar hyperglycemic state ) is:
❌ Why the Other Options Are Incorrect:
❌ Renal tubular acidosis (RTA) :
❌ Diabetic nephropathy :
❌ Diabetic ketoacidosis (DKA) :
More common in Type 1 diabetics
Features include:
This patient has very high glucose and no signs of acidosis or ketosis , which rules out DKA
❌ Diabetic neuropathy :
If a hormone that normally responds to renin is being overproduced while renin itself is low, think of a scenario where the hormone’s gland has gone rogue — acting independently of the usual control system.
223 / 232
Tags:
2021
A 55-year-old man complains of muscle weakness and dizziness for 4 months. His blood pressure is 190/105 mmHg. Laboratory results show hypernatremia, hypokalemia, elevated serum aldosterone, and low renin and angiotensin. Blood urea nitrogen (BUN) is 26 mg/dL, and creatinine is 1.3 mg/dL. The attending physician rules out Cushing syndrome. Which of the following is the most likley cause of hypertension in this patient?
🧠 Case Breakdown:
👨⚕️ Patient profile :
55-year-old male
Symptoms : Muscle weakness, dizziness (suggestive of electrolyte imbalance)
Vitals : Hypertension (190/105 mmHg)
Labs :
Cushing syndrome ruled out
🔬 Physiological reasoning :
The hallmark of Conn syndrome (primary hyperaldosteronism) is:
💡 The triad of hypertension, hypokalemia, and low renin strongly points toward primary hyperaldosteronism (Conn syndrome) .
❌ Why the Other Options Are Incorrect:
❌ Pheochromocytoma :
Causes episodic hypertension , palpitations, headache, and sweating.
Doesn’t usually cause hypokalemia or hypernatremia .
Aldosterone is not elevated.
❌ Adrenogenital syndrome (Congenital adrenal hyperplasia):
More common in younger individuals.
Involves enzyme defects (like 21-hydroxylase deficiency), leading to excess androgens .
Not typically associated with elevated aldosterone or this patient’s age/symptoms.
❌ Chronic adrenal failure (Addison’s disease) :
Features hypotension , hyponatremia , hyperkalemia — opposite of this case.
Due to low aldosterone and cortisol , not high.
❌ Chronic renal failure :
Would show elevated BUN and creatinine far beyond what’s in this patient.
Often associated with secondary hyperaldosteronism (↑ renin), not suppressed RAAS.
When a tumor causes symptoms that mimic a full-blown “fight-or-flight” response, consider which part of the adrenal gland is tied directly to the sympathetic nervous system — and not the steroid-producing layers.
224 / 232
Tags:
2021
Pheochromocytoma is a medical condition in which the patient experiences tachycardia (increased heart rate), hypertension, excessive sweating, and headaches. It involves which following part of the adrenal gland?
Pheochromocytoma is a rare catecholamine-secreting tumor that arises from the chromaffin cells of the adrenal medulla .
🔹 The adrenal gland has two main parts :
Adrenal cortex — outer layer, with three zones :
Zona glomerulosa → secretes aldosterone
Zona fasciculata → secretes cortisol
Zona reticularis → secretes androgens
Adrenal medulla — inner part, made of chromaffin cells (modified postganglionic sympathetic neurons) that secrete:
Epinephrine
Norepinephrine
🔥 In pheochromocytoma , excessive catecholamine secretion leads to:
Tachycardia (increased heart rate)
Hypertension
Excessive sweating (diaphoresis)
Headaches
Anxiety, palpitations, pallor, and panic attacks
This constellation of symptoms is sometimes called the “5 P’s” :Pressure (hypertension), Pain (headache), Perspiration, Palpitations, Pallor
❌ Why the Other Options Are Incorrect:
❌ Zona fasciculata :
Secretes glucocorticoids (mainly cortisol) .
Involved in stress response , but not catecholamine production .
Tumors here may cause Cushing’s syndrome , not pheochromocytoma.
❌ Adrenal cortex :
❌ Zona pellucida :
❌ Zona glomerulosa :
Produces aldosterone , which regulates sodium and potassium.
Involved in blood pressure regulation , but not in catecholamine release or pheochromocytoma .
When a duct carrying digestive secretions is compressed by a mass, consider both its location and the consequences of backed-up contents. Think about where bile goes when it can’t enter the intestine—and which nearby structures could cause that blockage.
225 / 232
Tags:
2021
A 50-year-old male patient came into the outpatient department with yellowish pigmentation of the skin, itching, and was diagnosed as a case of obstructive jaundice. What is the most likely cause of this patient’s jaundice?
📌 Key Clinical Clues in the Question :
This presentation points toward a mechanical blockage in the biliary tree — not a hepatocellular or hemolytic issue , but an obstructive one .
🔍 Why Cancer in the Head of the Pancreas is Correct:
The common bile duct passes through or very close to the head of the pancreas before emptying into the duodenum.
A tumor in the head of the pancreas can compress the common bile duct , leading to:
This is a classic presentation of pancreatic head carcinoma — often painless jaundice with itching in an older adult.
❌ Why the Other Options Are Incorrect:
❌ Perforated ulcer of the stomach :
Typically causes peritonitis , acute abdomen , and shock , but not jaundice .
No direct involvement with the biliary tree.
❌ Perforation of the duodenum :
May affect surrounding structures (e.g. pancreas), but does not cause obstructive jaundice directly.
More likely to cause peritonitis , pain , and gas under the diaphragm , not itching or yellowing of the skin.
❌ Cancer in the body of pancreas :
The body is located more medially , away from the bile duct .
Tumors here usually don’t compress the bile duct → less likely to cause jaundice.
More often causes epigastric pain radiating to the back .
❌ Obstruction of the main pancreatic duct :
Leads to pancreatitis or pancreatic insufficiency , not jaundice .
Does not interfere with bile flow unless the tumor is in the head region compressing both ducts.
Which form of diabetes is characterized by an absolute lack of insulin due to immune system malfunction, often beginning in early life?
226 / 232
Tags:
2021
The autoimmune destruction of beta cells results in which of the following diseases?
✅ Correct Answer: D) Diabetes mellitus type-1
Type 1 Diabetes Mellitus (T1DM) is an autoimmune disease where the body’s immune system mistakenly attacks and destroys the insulin-producing beta cells of the islets of Langerhans in the pancreas .
🔬 Breakdown:
Beta cells reside in the pancreatic islets (Islets of Langerhans) .
Their main function is to produce insulin , which helps cells absorb glucose from the bloodstream.
In Type 1 diabetes , this autoimmune destruction leads to absolute insulin deficiency , meaning:
Type 1 diabetes usually develops in childhood or adolescence , but it can also appear in adults (called latent autoimmune diabetes in adults , or LADA).
❌ Why the other options are incorrect:
A) None of these : Incorrect—Type 1 diabetes is a listed and correct option.
B) Cushing disease : This involves excess ACTH secretion , usually due to a pituitary adenoma , leading to hypercortisolism . It’s unrelated to beta cells or insulin.
C) Diabetes mellitus type-2 : This is caused by insulin resistance and often a relative insulin deficiency , not autoimmune destruction . The beta cells are present, though may become dysfunctional over time.
E) Diabetes insipidus : This is a completely unrelated condition involving ADH (antidiuretic hormone) and water balance, not insulin or glucose
Think about which condition would cause the thyroid gland to fail , making the pituitary work harder
227 / 232
Tags:
2023
A 40-year-old man presents with complaints of chronic fatigue, diffuse muscle aches, and intermittent numbness in his fingers. On examination, there is modest weight gain but no palpable thyroid enlargement. Laboratory evaluation reveals a thyroid-stimulating hormone (TSH) level >10 μU/L (reference range: 0.5–5.0 μU/L) and a free thyroxine (T4) level that is low to low-normal.
These findings are most consistent with which of the following conditions?
High TSH + Low T4 = Primary hypothyroidism → thyroid gland is failing, so pituitary increases TSH to compensate.
Autoimmune thyroid disease (Hashimoto thyroiditis) is the most common cause of hypothyroidism in this setting — even without palpable thyroid.
Symptoms (fatigue, weight gain, myalgias, numbness) also fit hypothyroidism.
Why the other options are incorrect:
Hyperthyroidism → would show low TSH + high T4 — opposite of this case.
Iodine deficiency → rare in developed countries; patient has no signs of endemic goiter.
Hyperthyroidism secondary to hypothalamic-pituitary defect → hyperthyroidism would show low TSH + high T4 — not the case.
Hypothyroidism secondary to hypothalamic-pituitary defect → in central hypothyroidism , TSH would be low or inappropriately normal , not elevated.
Consider which acute abdominal complication is directly linked to severe hypertriglyceridemia, especially above 1000 mg/dL.
228 / 232
Tags:
2023
A 34-year-old woman presents to the outpatient clinic for routine follow-up, one year after being prescribed fenofibrate for a lipid disorder. She reports frequent non-adherence to the medication regimen. Additionally, she mentions a strong family history of lipid disorders, with several maternal uncles on lipid-lowering therapy. On examination, her height is 163 cm (5’4″), weight 82 kg (180 lb), with a BMI of 31.6 kg/m². Notable findings include bilateral yellow plaques on the eyelids and multiple firm, painless nodules over the extensor surfaces of the hands. Laboratory analysis reveals a triglyceride level of 1100 mg/dL (12.43 mmol/L).
This patient is at greatest risk of developing which of the following complications?
Severely elevated triglycerides (> 1000 mg/dL) greatly increase risk of acute pancreatitis .
This is due to excess chylomicrons impairing pancreatic blood flow and triggering inflammation.
Patients often show xanthomas when triglycerides are elevated — seen here.
Pancreatitis is the most immediate and life-threatening risk in this patient.
Why the other options are incorrect:
Coronary heart disease : Elevated LDL is more strongly associated with CHD; triglycerides contribute less directly. While this is a long-term risk, pancreatitis is more urgent here.
Cholelithiasis : Fibrates may increase gallstone risk, but no gallstone symptoms here; also, TG elevation more likely leads to pancreatitis first.
Appendicitis : No relationship to triglyceride levels or lipid metabolism.
Hepatitis : No mention of hepatotoxic drug use or liver enzyme abnormalities.
Think about the most chronic, slowly progressive pattern of neuropathy commonly seen in long-standing poorly controlled diabetes.
229 / 232
Tags:
2023
A 60-year-old female presents to the clinic with complaints of pins and needles sensation along with burning feet over the last 8 to 9 months. She is a known case of diabetes and has been non-compliant with oral hypoglycemic therapy for the last 15 years. Clinical examination reveals sensory loss in a stocking pattern, absent ankle jerks, and subtle weakness of dorsiflexion of the toes. Which of the following is the dominant type of neuropathy in this patient?
The most common type of diabetic neuropathy is distal symmetric polyneuropathy (also called distal symmetric neuropathy).
It typically presents with:
Bilateral symptoms starting in the feet (stocking pattern).
Burning, tingling, pins and needles sensations.
Loss of ankle jerks.
Mild weakness in distal muscles (toe dorsiflexion).
It progresses slowly over months to years — fits this case perfectly.
Why the other options are incorrect:
Amyotrophic neuropathy : Involves proximal muscle weakness and wasting, usually of thighs or pelvic girdle — not the distal, sensory-dominant pattern seen here.
Entrapment neuropathy : Localized, single nerve compression (e.g., carpal tunnel); not symmetric stocking pattern.
Mononeuropathy multiplex : Multiple single nerves affected in an asymmetric, patchy way — this patient has a symmetric pattern.
Small fiber neuropathy : Presents mostly with pain and temperature sensory issues without weakness or reflex loss — here weakness and absent reflexes suggest involvement of large fibers too.
When a woman develops unexplained hormonal deficiencies postpartum—especially failure to lactate—ask: was there any significant peripartum blood loss ? Which gland is most vulnerable to ischemia in this setting?
230 / 232
Tags:
2021
A young woman comes into the emergency room experiencing vertigo, nausea, bradycardia, failure to lactate, and loss of genital and axillary hair. She has given birth one month before and stayed in the hospital for 1 week postpartum. What is the most probable diagnosis?
Sheehan Syndrome is a form of postpartum hypopituitarism due to ischemic necrosis of the anterior pituitary gland . It typically occurs in women who experience severe blood loss or hypotension during or after childbirth , compromising pituitary blood flow.
🔬 Pathophysiology:
During pregnancy, the anterior pituitary enlarges due to estrogen stimulation (mainly from lactotroph hyperplasia).
It becomes more vulnerable to ischemia in the setting of postpartum hemorrhage .
Resulting necrosis leads to panhypopituitarism , or failure of multiple pituitary hormones.
✅ Key Clinical Features:
Failure to lactate → ↓ prolactin
Loss of pubic/axillary hair → ↓ LH/FSH
Bradycardia, hypotension, fatigue, nausea → ↓ ACTH → ↓ cortisol
Menstrual abnormalities or amenorrhea
May develop gradually or present acutely postpartum
❌ Why Other Options Are Incorrect:
When thinking of the most common cause of increased thyroid hormone production, focus on conditions where hormone synthesis is increased , not just released due to inflammation or destruction. Consider whether the process is autoimmune stimulation versus transient leakage of preformed hormone.
231 / 232
Tags:
2021
Which of the following is the most common cause of hyperthyroidism or thyrotoxicosis?
Graves disease is the most common cause of hyperthyroidism (thyrotoxicosis) , especially in young to middle-aged adults , with a strong female predominance .
🔬 Graves Disease Overview:
It is an autoimmune disorder where the body produces thyroid-stimulating immunoglobulins (TSIs) .
These TSIs bind to the TSH receptor on thyroid follicular cells, stimulating excess production of T3 and T4 .
This leads to diffuse toxic goiter and classical signs of thyrotoxicosis , including:
❌ Why the Other Options Are Incorrect:
Hashimoto thyroiditis : Most common cause of hypothyroidism , not hyperthyroidism. Occasionally presents with a transient thyrotoxic phase (Hashitoxicosis), but not the primary cause.
Granulomatous thyroiditis (De Quervain’s): Causes painful thyroid inflammation and transient thyrotoxicosis, but it is self-limited and much less common .
Subacute lymphocytic thyroiditis : A painless thyroiditis with transient hyperthyroidism, often post-partum. Again, not the most common cause overall.
None of these : Incorrect—Graves disease is the well-established leading cause.
When a pituitary tumor secretes hormones that affect reproductive function, think about how those hormones impact hypothalamic-pituitary-gonadal signaling . Does the excess promote or inhibit normal reproductive cycles?
232 / 232
Tags:
2021
Which of the following are seen with lactotroph adenoma?
A lactotroph adenoma (also called a prolactinoma ) is the most common type of functioning pituitary adenoma . It arises from lactotroph cells of the anterior pituitary and secretes excessive amounts of prolactin (PRL) .
🧬 Key Features of Prolactin Excess:
In women :
Galactorrhea – abnormal milk production unrelated to breastfeeding
Amenorrhea – prolactin inhibits GnRH → ↓ LH/FSH → menstrual irregularity
Infertility – anovulation due to suppressed gonadotropins
In men :
❌ Why Other Options Are Incorrect:
Galactorrhea, amenorrhea, increased libido : Increased libido is not a symptom—libido is usually decreased due to suppressed gonadal hormones.
None of these : Incorrect—classical triad exists.
Galactorrhea, polymenorrhea, increased libido : Polymenorrhea (frequent periods) and increased libido are not associated with hyperprolactinemia.
Galactorrhea, increased libido, infertility : Again, increased libido is inaccurate ; it’s typically decreased .
Your score is
The average score is 0%
Restart quiz
Anonymous feedback
Thank you for your feedback.